Download as pdf or txt
Download as pdf or txt
You are on page 1of 129

c.

Metoprolol
d. Misoprostol

Pharmacology 1_1
1. What is the interaction of a full agonist and a
competitive antagonist?
a. Reversible binding
b. Maximal dose of agonist will not be achieved
c. The opposite effect of agonist will happen is
competitive agonist is induced alone
d. All of the above
e. A and B only

2. True of Therapeutic Index


7. A 28 year old pregnant patient was having upper
a. The larger the TI, the more effective is the
respiratory infection. She decided to self medicate
drug
and bought from the pharmacy an anti-tussive
b. The larger the TI, the safer is the drug
medication. It says in the drug literature that the drug
c. The larger the TI, the more potent is the
is Pregnancy Category B. This means:
drug
a. Adequate and well controlled studies have
d. All of the above
failed to demonstrate a risk to the fetus in
e. A and B
the first trimester of pregnancy.
b. Animal reproduction studies have shown an
3. Which of the following is true
adverse effect on the fetus, and there are no
adequate and well-controlled studies in
humans, but potential benefits may warrant
use of the drug in pregnant women despite
potential risks.
c. Animal reproduction studies have failed
to demonstrate a risk to the fetus, and
there are no adequate and well-
controlled studies in pregnant women.
d. Positive evidence of human fetal risk based
a. Drug C is more efficacious than drug A on adverse reaction data from
b. Drug E is more potent than drug C investigational or marketing experience, and
c. Drug B is as efficacious as drug D the risks involved in the use of the drug in
d. All of the above pregnant women clearly outweigh potential
e. A and B benefits.
4. Binds basic drugs
a. Albumin
b. Alpha 1-glycoprotein
c. Alpha 2- macroglobulin
d. Beta 2-microglobulin
Albumin for acidic drugs

5. Refilling of an anti-seizure drug like phenobarbital is


prohibited if the date of issuance of the prescription
is more than
a. 2 weeks
b. 1 month
c. 3 months
d. 6 months

6. Which of the following drugs does NOT require a


second messenger to exert a pharmacologic action?
a. Atropine
b. Lidocaine

BK23 | TABLE OF CONTENTS


9. Meaning of A in SBAR
a. Achievement
b. Application
c. Act
d. Assessment

8. True of apoptosis
a. Does not elicit an inflammatory response
b. End result is phagocytosis by macrophage
c. Will activate in organelle damage
d. All of the above
e. A and B
10. Not a major proinflammatory cytokine
a. IL1
b. IL6
c. IL10
d. TNF-alpha

11. CNS drugs cross BBB barrier with what


pharmacokinetic property?
a. Hydrophilicity

BK23 | TABLE OF CONTENTS


b. Lipophilicity In the case of drugs that bind covalently to the receptor site, the effect
may persist until the drug-receptor complex is destroyed and
c. Low pKa new receptors or enzymes are synthesized, as described
d. High bioavailability previously for aspirin. Covalent bonds are very strong and in
many cases not reversible under biologic conditions.
12. True of maintenance dose:
a. Administered in repetitive doses 16. Strategy to improve patient compliance
b. Administered in continuous IV infusion a. Clinic secretary prints computerized
c. Directly proportional to bioavailability prescription and gives it to patient
d. A and B b. Doctor writes prescription and reads
e. All of the above instructions to nurse-patient
GG (p. 27) c. Doctor gives the patient his maintenance
In most clinical situations, drugs are administered in a series of
repetitive doses or as a continuous infusion to maintain a steady
good for 1 year
state concentration of drug associated with the therapeutic d. Doctor is friendly with family members
window.
Dosing rate = Target Cp * CL/F
Based on this equation, dosing rate is inversely proportional to the
bioavailability (F)

13. Which of the following schedule needs prescription


on refill at any circumstance
a. Schedule 2
b. Schedule 3
c. Schedule 4
d. Schedule 5
GG (Appendix 1, p. 1319)
Schedule 2: No prescription order may be refilled under any
circumstance. - highly regulated
Schedule 3 and 4: Refills may be issued either orally or in writing, not
to exceed 5 refills or 6 mos after the issue date, whichever
comes first.
Schedule 5: No restrictions, but if no refills are noted at the time of
issuance, a new prescription must be made for additional drug to
be dispensed.

14. Which vaccine components LEAST LIKELY gives an


allergic reaction
a. Egg
b. Gelatin
c. Neomycin
d. Thimerosal
PLEASE DOUBLE CHECK THANKS
a. egg - some influenza vaccines are prepared in eggs; some people
have history of egg allergy
b. gelatin - stabilizer in some vaccine like varicella and MMR, may
cause allergic in some
c. neomycin - antibiotic;use to ensure sterility in some vaccines ( ex.
MMR, trivalent IPV, and varicella vaccine), may cause adverse
reaction
d. thimerosal - trade name Merthiolate, one of most commonly used
preservatives, organomercurial, associated with neurotoxicity;
eliminated or reduced in vaccines for children under 6yrs old

17. Not myeloid in origin


a. Monocyte
b. Neutrophil
15. This drug-receptor interaction causes action of drug c. T-cell
to persist longer even after drug has dissociated with d. NOTA
receptor and until the drug-receptor complex is Page 621 (GG)
destroyed
a. Electrostatic
b. Covalent
c. Hydrophobic
d. Electromagnetic
Katzung (p. 6)
18. Which of the following is/are true

BK23 | TABLE OF CONTENTS


a. Thymus is a secondary lymphoid organ 24. Not true for phase 1 clinical trial
b. Spleen is the only organ connected to a a. Its goal is to determine the dose of a healthy
blood vessel individual
c. NK cells are not granular cells b. If the studied drug has significant toxicity,
d. All of the above volunteers .... diseases experiencing
individuals
c. Absorption, half-life and metabolism are
studied
d. Study of drug efficacy and dose ranging
D. (this is Phase 2) GG page 6

25. Phagocytosis:
a. Neutrophils
b. Eosinophils
c. Both
d. None
GG page 621

26. A patient complained with decreased urine output,


proteinuria, fever, rash, and peripheral edema. He
19. T-cell anergy due to the lack of had a history of NSAIDs intake a month ago.
a. CD28 Interstitial nephritis is considered. What type of
b. CD25 hypersensitivity
c. IL2 a. Type I
d. TNFa b. Type II
(page 626 GG) c. Type III
Anergy is defined by the inability of T cells to proliferate after engaging d. Type IV
a peptide MHC complex due to lack of IL2 production and p. 631 GG
signaling

20. Which of the following vaccines neutralizes toxin?


a. BCG
b. Tdap
c. Yellow fever
d. Pneumococcal vaccine
(Toxoids- GG page 660)

21. True of Metabolism


a. Start of elimination process
b. Phase 1 is the functionalization of the drug 27. Which of the following antibodies is first released by
c. Change of drug from lipophilic to hydrophilic the B cells upon activation?
d. All of the above a. IgE
b. IgA
c. IgG
d. IgM
p. 628 GG
22. Which is NOT an attribute of High Reliability
a. Preoccupation to failure
b. Sensitivity to mistakes
c. Culture of safety
d. None of the above
(it should be sensitivity to operations) Patient safety manual

23. Why human’s DISTRACTIBILITY is considered a 28. True about Absorption


strength? a. The drug has to be in aqueous solution to be
a. Humans easily get out of focus involved in Fick's passive diffusion
b. Humans team and carry out diverse b. Both facilitated transport and active
functions transport require a carrier protein
c. Humans deal with mistakes to correct it c. An ionized drug has increased lipophilicity
d. Humans are sensitive to minor changes and is easily transported
in the system d. All of the above
(distractibility helps us notice when something is unusual)
e. A and B
Patient safety page 114
From previous recalls hehe

BK23 | TABLE OF CONTENTS


29. Which of the following situations adhere fitness to
practice requirements?
a. Pediatrician doing a major surgery
b. Choosing the students to enroll in med
school
c. Following the management protocols in
treating the patient
d. B and C

30. Unexpected events that lead to death and serious


physical injury
a. Sentinel events
b. Adverse reactions
c. Errors
d. Mistakes
p. 96 WHO Patient Safety Curriculum Guide

32. Factors that influence bioavailability


a. Lipophilicity
b. Molecular size
c. First pass effect
31. Morphine and Tramadol are both effective d. AOTA
analgesics. But only a smaller dose is needed to
achieve an analgesic effect of Morphine compared 33. Statement that describes the illustration
to Tramadol. Which of the following is/are true? a. Drug A is a full agonist
a. Morphine is more potent than tramadol b. Drug B is an agonist with an antagonist
b. Tramadol is less efficacious than morphine c. Drug B is a partial agonist
c. Morphine is a full agonist while tramadol is d. AOTA
partial agonist e. A and B
d. AOTA
e. A and B 34. Which of the following drugs must be AVOIDED in
p. 34 GG pregnant women?
a. Hepatitis B
b. Pneumococcal vaccine
c. MMR
d. Rabies
37. Acidification and alkalinization of the urine will
enhance excretion of drugs. What mechanism?
a. Reduce reabsorption of drugs
b. Increased ionization of drugs in the
tubules
c. increase secretion of drugs
d. AOTA

38. Key cytokine produced by NK cells


a. INF-1
b. INF-2
c. IL-2
d. IL-12
(p. 625 GG)
The key cytokine produced by NK cells is IFN-ყ, also called Type 2 INF

BK23 | TABLE OF CONTENTS


39. You broke the tube filled with blood, the blood spilled
on your face and upper extremity. What is the first
line of defense of your body against the spilled
blood?
a. Serum IgA
b. Serum IgM
c. Skin
d. Secreted Pattern.. Receptor
43. Which of the following is a primary lymphoid organ?
40. TRUE of estimated plasma drug concentration: a. Bone marrow
a. It is proportional to drug clearance. b. Spleen
b. It is inversely proportional to the volume of c. MALT
distribution.
c. It is inversely proportional to the drug 44. Which is true with ion receptors?
bioavailability. a. Fast Cellular response…
d. All of the above b. Nicotinic receptors ....
e. A and B c. Voltage-gated channels....
41. Endogenous ligands that binds to a nuclear receptor, 45. True for B cell development
EXCEPT: a. Reside in spleen and lymphoid vessels
a. Vitamin D b. Immature B cells leave the thymus still
b. Estrogen immature
c. Serotonin c. Produce IgM and IgG
d. Aldosterone d. Both A and B

46. 25 year-old patient with septic shock was given a


broad spectrum antibiotic. Which drug property/ies
42. Which of the following statements about the immune is/are need/ed to compute for Loading dose:
system is correct a. Vd
a. Exogenous antigen of pathogens are b. Half-life
internally processed and presented by MHC c. Clearance
2 to CD8 cells d. AOTA
b. Both immunoglobulins and T cell receptors e. A and B
can recognize the antigen that in its native
form
c. B cells complete their maturation in bone
marrow
d. A small population of memory cells is left
after pathogen clearance by both B and T
cells and is responsible for secondary
immune response 47. Causes Gullian- Barre Syndrome
A. FALSE: MHC 2 to CD4 cells a. Influenza
B. FALSE: Only immunoglobulins recognize naive cells; TCRs b. Rotavirus
only recognize processed antigen fragments presented by
MHC
C. FALSE - B cells leave the bone marrow and complete their
maturation in the periphery.
D. TRUE

BK23 | TABLE OF CONTENTS


48. Which of the following would describe
PHARMACOKINETICS?

a. Relationship between plasma concentration


and response
b. Diffusion through membranes
c. Zero order kinetics offers constant
amount to be cleared
d. AOTA
e. A and B

BK23 | TABLE OF CONTENTS


Pharmacology 1_2 A. Beta blocker
B. Thiazide and ACEi
1. The pleasurable effect of Ethanol is due to its C. Nicardipine
interaction with the: D. CCB and Loop diuretic
GG figure 28-1
A. mu receptor but it basically says:
B. GABA-benzodiazepine receptor for stage 2 hypertension
C. NMDA-glutamate receptor - lifestyle modifications
- thiazide and ACEi, ARB, or CCB
D. All of the above
- ACE plus CCB
Katzung Chapter 32, table 32-1 page 577
-
7. Cefipime has good activity against the following
2. Which of the following leads represent the inferior
microorganism/s:
wall of the myocardium?
A. Pseudomonas aeroginosa
A. V5-V6
B. MRSA
B. V1-V3
C. ESBL (+) E.coli
C. I,II,III
D. All of the above
D. II,III, avF GG 13th ed; PAGE 1034
D. Lead II, III, aVF are located in the bottom of the heart, used to look
at the inferior portion of the heart.
8. The normal duration of PR interval is:
not A. cause V5-V6: lateral wall of the left ventricle A. 120-200 secs
B. 100-110 millisecs
3. This is considered the earliest ECG change in C. 0.12-0.20 secs
hyperkalaemia D. 0.10-0.22 secs
A. T wave inversion HARRISON'S 20th ed page 1676
B. Peaked T waves
C. Prolonged QT interval
D. Rabbit ears sign in leads V1-V3
Early changes of hyperkalemia include tall, peaked T waves with a
narrow base, best seen in precordial leads ; shortened QT
interval; and ST-segment depression. These changes are 9. A 50-year old male patient is being treated with
typically seen at a serum potassium level of 5.5-6.5 mEq/L.
Widened QRS complexes in hyperkalemia. ABVD chemotherapy for Hodgkin’s lymphoma. He
presents with for cycle 4 of planned 6 cycles with
4. EL is a 56-yer old male about to undergo new onset of cough. He also complains it started a
chemotherapy for rhabdomyosarcoma. His week ago and also feels he has a little trouble
chemotherapy includes Ifosphamide. Which of the catching his breath. Which is most likely the cause of
following measures should be included in the this toxicity?
chemotherapy orders of this patient? A. Doxorubicin
A. IV hydration and frequent urinalysis B. Bleomycin
B. Leucovorin and frequent urinalysis C. Vincristine
C. Allopurinol and frequent urinalysis D. Dacarbazine
D. Antibiotics and frequent urinalysis KATZUNG 14th ed; PAGE 962-TABLE 54-4
“severe urinary tract and CNS toxicity initially limited the use of
ifosphamide, but adequate hydration and coadministration of 10. The upper limit of Vitamin D intake is:
MENSA have reduced its bladder toxicity.”
GG page 1173: ifosphamide A. 400 IU/day
B. 800 IU/day
5. Which of the following is/are the effect/s of Thiazide C. 1000 IU/day
diuretics? D. 4000 IU/day
A. Increase PO4 in urine HARRISON'S 20th ed
B. Increase uric acid in urine on chronic use
C. Decrease Cl excretion
D. Increase K concentration in the serum
Effects of thiazide diuretics:
● enhances Na+ excretion in the urine leading to decreased
cardiac output
● inhibit renal Ca+ excretion leading to hypercalcemia
● decreases ECV
● excessive K+ excretion = hypokalemia
11. This ECG wave represents the final rapid phase of
6. A patient with BP reading of 170/110 mmHg should repolarization:
be started with this antihypertensive drug based on A. T wave
the ACC/AHA algorithm. B. QRS wave

BK23 | TABLE OF CONTENTS


C. P wave D. Modification of the target enzyme DHFR
D. ST segment ?*not sure (GG 13th ed, p 1179)
HARRISON'S 20th ed

16. What ARB is best given to a hypertensive and


diabetic individual with history of stoke?
A. Valsartan
B. Captopril
C. Losartan
D. Irbesartan
(GG , p 485)
12. Which chemotherapeutic agent is proven to be a risk
for developing Hemolytic Uremic Syndrome?
A. Mitomycin
B. Doxorubicin
C. Idarubicin 17. Which among the following drugs for dyslipidemia is
D. Vincristine not safe for children?
KATZUNG 14th ed; PAGE 962-TABLE 54-4 A. Atorvastatin
B. Cholestyramine
13. Which of the following compartment/structure would C. Simvastatin
an IV antibiotic easily penetrate? D. Gemfibrozil
A. Bacterial biofilm (GG Table, p 617)
B. Endocardial vegetation
C. Lung
D. Retina
(GG p 957-958)
A. and B. : biofilms have an exopolysaccharide that is negatively
charged which binds positively charged antibiotics and restrict
their access to the intended target.
D. Retina: there is poor penetration of drug from plasma to this
compartment

14. Caffeine is a hidden drug found in the following OTC


drugs:
A. Listerine
B. Vicks
C. Midol
D. Robitussin
(Katzung 14th ed, p 1129)

18. A 5-year old patient is about to undergo 3 cycles of


chemotherapy prior to surgery for Wilm’s Tumor
15. Resistance to methotrexate is a result of which of which of the following best describes the
the following mechanism? chemotherapy in this setting?
A. Increased activity of glutathione transferase A. Adjuvant chemotherapy
B. Increased cell thiol B. Neoadjuvant chemotherapy
C. Modification of DNA repair mechanism C. Palliative chemotherapy

BK23 | TABLE OF CONTENTS


D. Primary Induction chemotherapy
(Katzung 14th ed, p 950)

21. MM, 18 yo/M, with no comorbids, noted to have


fever and odynophagia for 2 days now. On PE,
tonsils are inflamed with scanty whitish exudates
associated with enlarged tonsillar lymph nodes.
19. The principle of anti-cancer drug selection for Which of the following is the appropriate antibiotic?
combination in which a drug is preferred if it can A. Penicillin V
produce complete remission in some fractions of B. Penicillin G
patients than those that only produce partial C. Gentamicin
remission. D. B and C
(page 1027, GG)
A. Effectiveness
B. Efficacy
C. Toxicity
D. Dose factor
(page 952 Katzung)

22. Which of the following antihypertensives is given to


diabetics and post MI patients because of the
advantages of slowing the development of diabetic
nephropathy and enhances cardiac remodelling?
A. Calcium channel blockers
B. ACE inhibitors
C. AT1 receptor antagonist
D. B1 selective adrenergic antagonist
(page 516, GG)

20. What would be the change in the sigmoid curve if


the microbes have developed resistance and 23. Which of the following is the correct dose of
eradication would not be possible? Penicillin V for the treatment of acute tonsillitis?
A. Sigmoid curve shifts to the left (decrease IC) A. 12 gms IV daily for 4 weeks
B. Sigmoid curve shifts to the right (increase IC) B. 24M units daily IV for 4 weeks
C. Sigmoid curve shortens (decrease Emax) C. 12 M units daily plus 1 gm Gentamicin IV for 2
D. Sigmoid curve lengthens (increase Emax) weeks
(page 960, GG) D. 500 mg BID PO for 10 days
(page 1027,GG)

BK23 | TABLE OF CONTENTS


27. A 5th generation Cephalosporin with activity against
MRSA
A. Cefipime
B. Ceftaroline
C. Ceftazidime
D. None of the above
Anti-MRSA cephalosporins have structural modifications allowing
binding to and inactivation of the altered PBPs expressed by
MRSA, MRSE, and penicillin-resistant S.pneumoniae.
24. John 18 yo with no comorbids went to the OPD due Ceftatroline and ceftobiprole are the currently used agents.
to dysuria for 10 days. Upon examination you noted GG page 1033

scanty turbid penile discharges associated with 28. A 57-year old woman was treated for Pneumonia
inguinal lymphadenopathy. Patient claimed he was where she was started with Piperacillin-Tazobactam.
seen by a doctor in the province and was given a 3 days later, SGPT was taken and noted to be
single dose of injectable medication only. What is elevated at 350 mg/dl. Which of the following should
the most likely antibiotic given? you do?
A. Cefipime A. Decrease Piperacillin dose
B. Cefixime B. Increase Piperacillin dose
C. Cefotaxime C. No dose changes
D. Ceftriaxone D. Change antibiotic
(page 1033, GG) (not sure)

29. Which of the following is true of general combination


cancer chemotherapy?
A. It is administered during several cycles of
treatment
B. It is less effective than monotherapy
C. It includes at least 2 drugs with similar dose-
limiting toxicities
D. It includes one drug that has no inherent
therapeutic activity
When several drugs of a given class are available and are equally
effective, A DRUG SHOULD BE SELECTED ON THE BASIS OF
TOXICITY THAT DOES NOT OVERLAP WITH THE TOXICITY
OF OTHER DRUGS IN COMBINATION
Katzung page 952
25. Which of the following dosing schedules maximizes
the antimicrobial effect of Aminoglycosides? 30. Physical dependence is manifested by the abuser’s:
A. AUC/MIC A. Tolerance from the drug
B. Cmax/MIC B. Addiction from the drug
C. T > MIC C. Withdrawal symptoms from the drug
D. PAE D. Craving for the drug
Physical dependence is characterized by Withdrawal or Abstinence
Syndrome (Doc’s ppt)

31. Which of the following information cannot be elicited


through the ECG?
26. Newborns are prone to Vitamin K deficiency
A. Cardiac hypertrophy
because they have:
B. Heart rate
A. Low fat stores
C. Myocardial ischemia and infarction
B. Immature intestines
D. Ejection fraction
C. Low vitamin K level passed on from the mother Harrison’s p. 1678 (not sure huhu)
D. Immature cells in the bone marrow
newborns are particularly susceptible because of low fat stores
Harrison’s 20th ed (page 2317)

BK23 | TABLE OF CONTENTS


35. Which of the following is an advantage or Ertapenem
32. Which of the following agents forms an inhibitory over Meropenem?
ternary complex with the enzyme thymidylate A. Less activity against Enterococcus and
synthetase? Acinetobacter thus lesser side effects
A. Cyclophosphamide B. Longer half life
B. Fluorouracil (5-FU) C. Good coverage against Pseudomonas
C. Mercaptopurine (6-MP) D. Less sensitive to renal dipeptidase
D. Methotrexate (MTX) GG p. 1035
Katzung p. 651

36. Which of the following chemotherapeutic drugs can


possibly cause secondary malignancies?
A. Antimetabolites
B. Vinca alkaloids
C. Alkylating agents
33. A 38- year old male arrived in the emergency room D. Anti-tumor antibiotics
severely agitated, sweating tachycardic and Katzung p. 954
hypertensive. He is a known heavy alcoholic drinker
for more than10 years now. His last sip of alcohol
was 6 hours ago, he is also known to be a “shabu”
user. Which of the following is correct regarding this
patient? 37. What susceptibility test for parasites is used by the
A. The patient is having “Delirium Tremens” health care workers to be able to individualize
B. Diazepam is contraindicated to this patient therapy?
C. The patient is acutely intoxicated with alcohol A. Disc diffusion test
D. The patient is acutely intoxicated with B. Broth dilution test
“shabu” C. Phenotypic assays
GG p. 441 D. None of the above
P. 961
Susceptibility testing for parasites, especially those that cause malaria
has been performed in the laboratory. The tests are similar to
broth tests for bacteria, fungi, and viruses

38. These statins are already in active form when


administered EXCEPT:
A. Atorvastatin
B. Fluvastatin
C. Simvastatin
D. Rosuvastatin

39. How soon should a patient with Stage 1


hypertension be re-evaluated after initiation of
antihypertensive drug based on the ACC/AHA
34. MIOSIS is typically seen in the use of: algorithm?
A. Amphetamine A. 2 weeks
B. Diphenhydramine B. 1month
C. Ecstasy C. 2 months
D. Morphine D. 3 months
GG p. 372 GG; Chpater 28, pg 509

BK23 | TABLE OF CONTENTS


45. The combination of these 2 lipid lowering drugs is
not recommended because the drug inhibits the
absorption of the other drug
A. Statins + Fibrates
B. Niacin + Bile acid sequestrants
40. What is a common major side effect of Ceftriaxone? C. Statin + Niacin
A. Seizure D. Bile acid sequestrants + Ezetimibe
GG; Chapter 33, pg 611
B. Kernicterus Combination of statins and fibrates is discouraged.
C. Biliary stones
D. Encephalopathy
P. 1034
Ceftriaxone's high biliary conc combined with its affinity for Ca can lead
to biliary pseudolithiasis

Cefepime - encepalopathy and nonconvulsive status epilepticus

41. This decongestant is present in numerous OTC cold


preparations and was taken out as an OTC drug 46. Mrs C, 70 yo, diabetic was admitted due to
because it has been used in illicit manufacture of productive cough, fever and chills. CXR showed
metamphetamine: inflammatory process on left lower lobe. She was
A. Phenylpropanolamine given Ceftriaxone for 72 hours but later changed to
B. Pseudoephedrine Piperacillin+ Tazobactam due to persistence of fever
C. Chlorphenamine maleate and 1 episode of hypotension. Which of the following
D. Phenylephrine hydrochloride bacteria is covered by Piperacillin but not with
Ceftriaxone?
42. A 45-year old female obese patient was seen at the A. Streptococcus pneumonia
ER for an acute onset of nape pain. There was no B. Anaerobes
associated loss of consciousness or vomiting. Her C. Pseudomonas aeroginosa
BP at the ER was 170/110 mmHg. Based on the D. All of the above
American Heart Association criteria, what is the GG; Chapter 57, Table 57-3, pg 611
classification of the patient’s BP?
Antipseudomonal drugs include 4th gen cephalosporins (Cefepime,
A. Prehypertension Cefolozane, and Cefazidime) and antipseudomonal penicillins
B. Stage 1 hypertension (piperacillin).
C. Stage 2 hypertension
Ceftriaxone (3rd gen cephalosporin)
D. Hypertensive crisis Escherichia coli, Klebsiella, Proteus, Haemophilus influenzae,
Moraxella catarrhalis, Citrobacter ,
e

43. The following are manifestations of folic deficiency Enterobacter ; Serratia; Neisseria gonorrhoeae; activity for S. aureus,
e

Streptococcus pneumoniae, and Streptococcus pyogenes


EXCEPT: comparable to frst-generation agents. Activity against
A. Megaloblastic anemia Bacteroides spp. inferior to that of cefoxitin and cefotetan.
B. Atrophic glossitis
C. Retinopathy
D. Depression

44. Which of the following compartments would a


hydrophobic molecule concentrate?
47. Clavulanic acid:
A. Blood
A. Piperacillin
B. Cytosol
B. Imipenem
C. Lipid bilayer
C. Amoxicillin
D. Pleural fluid
GG; Chpater 6, pg 85 D. Ceftazidime
GG; Chapter 57, pg 1036

Common drug: amoxiclav

BK23 | TABLE OF CONTENTS


DIURETICS (LTP; Love thy Pota)
1. LOOP
= inhibits Na/K/2Cl cotransporter in the ascending loop of Henle

2. THIAZIDES
= inhibits the Na/Cl cotransporter (NCC or SLC12A3) in the distal
convoluted tubules
48. A certain 25-year old female cancer patient develops
3. K SPARING
syndrome of inappropriate secretion of ADH = inhibits the ENaC (epithelial sodium channel/countertransporter) in
(SIADH) after 3 sessions of her anti-cancer regimen. the distal tubule membrane
Which agent could be the possible culprit of this
GOOGLE SEARCH
complication?
A. Etoposide
B. Paclitaxel
C. Vinblastine
D. Colchicine
GG; Chapter 66, pg 1187

49. 1 65-year old male, diabetic patient came to the


clinic for check-up due to headache. Upon taking the
BP, it was noted to be 180/100 mmHg. Based on the
BP reading, which predicts better of adverse
outcome?
A. Systolic BP
B. Diastolic BP
C. Mean arterial pressure
D. Mean BP on 2 separate occasions
GG; Chapter 28, pg 507

50. The site of action of the K sparing diuretic-


spironolactone:
A. Apical sodium membrane
B. Na/Cl cotransport in distal convoluted tubule
C. Na/K/2 Cl cotransport in the ascending loop of
Henle
D. Renal epithelial cells
GG; Chapter 28, pg 509, 512

BK23 | TABLE OF CONTENTS


Pharmacology 1st BME C. Drug B is more potent if the dose required is
greater than Drug A to produce the effect
1. Which does not involve drug-receptor interaction in D. All of the above
E. A and B only
following intervention Rationale:
A. Diphenhydramine > Allergy Potency - A measure of the amount of drug required to produce 50% of
B. Sucralfate > Peptic ulcer disease the maximal response.
C. Amiodarone > cardiac arrhythmia Efficacy - The maximal response that can be elicited by the drug
regardless of the dose.
D. Thyroxine > Hypothyroidism
Rationale:
Katzung 14th ed page 36
Amiodarone’s salutary effects and toxicities may also be mediated
through interactions with receptors...
G&G pages 31 5. Your patient’s ECG report came out and reads-sinus
bradycardia with acute anterior wall myocardial
2. TRUE of First-order kinetics: infarction. What does this ECG tracing look like?
A. Elimination constant does not exist A. ST depression at V1-V6
B. Occurs when the metabolic activity of the drug is B. ST elevation at II, III and AVF
saturated C. ST depression at I, II, III
C. A fraction of the drug is removed per unit D. ST elevation at V3-V4
time Rationale:
D. All of the above ST segment elevation in the anterior leads (V3 and V4) at the J point
and sometimes in the septal or lateral leads, depending on the
E. A and B only extent of the MI. This ST segment elevation is concave
Rationale:
downward and frequently overwhelms the T wave. This is called
“tombstoning” for obvious reasons; the shape is similar to that of
a tombstone.
Reciprocal ST segment depression in the inferior leads (II, III and aVF).
Net

6. John, 55 years old, a known diabetic for 5 years now


with poor sugar control went to your clinic for follow
up. Patient has been complaining of on and off
watery non bloody and non mucoid stools. No fever
but with occasional abdominal discomfort. He
claimed he has been taking amoxicillin for 4 weeks
now due to his non healing wound at the left foot.
G&G 13th ed page 20 Vital signs are stable with no signs of dehydration.
What is the next big thing to do?
3. Which of the following is not a part of the main A. Admit the patient and do full work up
domains of health care? B. Start another antibiotic such as Cloxacillin for
A. Patients family more potent gram positive coverage
B. Nurse aide C. Advise him to take metformin for better sugar
C. Quality control council control
D. Infection control protocol D. Advise him to stop amoxicillin and give
Rationale: probiotic
Rationale:
i think the intestinal normal flora got killed. Idk help.

7. The following are advantages of PTC drugs EXCEPT:


A. Patients can self-diagnose and self-manage
B. Benefits outweigh the risks
C. Adequately labeled
WHO patient safety summary D. Less costly
Rationale:
OTC drugs pptx

4. If Drug A and Drug B are both potent, then the


following statement/s is/are TRUE:
A. Both drugs should produce equivalent response
B. Drug A is more efficacious than Drug B if Drug A
requires a lesser dose to produce the effect 8. Which of the following is a transporter of biologic
amines?

BK23 | TABLE OF CONTENTS


A. Nicotine
B. Alcohol
C. Morphine
D. Ecstasy
Rationale: Katzung 14th ed page 577
Bind to Ionotropic receptors and Ion channels:
Nicotine, Alcohol
Bind to Transporters of Biogenic Amines:
Ecstasy
Activate G Protein-Coupled Receptors
Opioids (morphine, Heroin)

9. Which of the following statements regarding


hypersensitivity reactions to penicillin is true?
A. Anaphylaxis is most serious reaction and can
happen mostly in young antimicrobial-naïve
individuals
B. Common after IM administration of
penicillins
C. Only 1% of patients die from anaphylaxis
D. Nausea and vomiting are rare signs of
hypersensitivity
Rationale:
G&G 13th ed page 1030
Angioedema and Anaphylaxis are the most serious hypersensitivity
reactions produced by penicillins. It is induced by various
preparations.

Anaphylaxis most often has followed the injection of penicillin, although


it also has been observed after oral or intradermal administration

0.001% of patients die from anaphylaxis

The most dramatic reaction is sudden, severe hypotension and rapid


death. In other instances, bronchoconstriction with severe
asthma; abdominal pain, nausea, and vomiting

10. Which of the following cells is NOT associated with


chronic inflammation?
A. Eosinophil
B. Fibroblast
C. Lymphocyte
D. Monocyte
Rationale: G&G 13th ed page 631
Hypersensitivity GG 13th ed (Ch 36 pp 658-659)
Chronic
Chronic
Chronic

11. Which of the following is considered a live vaccine?


A. Adsorbed tetanus toxoid
B. Hepatitis B immunoglobulin
C. Oral typhoid vaccine
D. Polyvalent pneumococcal vaccine 12. TRUE of Delirium Tremens:
Rationale: A. Occurs as early as 6 hours of cessation of
A. Toxoid
B. Human hyperimmune globulins alcohol use
C. Live bacterial B. Can last up to 4 weeks
D. Bacterial polysaccharide C. Death can be caused by autonomic
instability such as arrhythmias and
hyperthermia
D. All of the above
Rationale:
A. Delirium tremens (48–96 h after last drink; rare in uncomplicated
withdrawal) (GG,Ch24 p436)
It is now commonly known to occur as early as 48 hours after abrupt
cessation of alcohol in those with chronic abuse. (NCBI)

BK23 | TABLE OF CONTENTS


B. It can last up to 5 days. (NCBI)

C. Delirium tremens can be associated with a number of complications


including respiratory depression, seizures, arrhythmias and
aspiration pneumonitis. Even with appropriate therapy, the
mortality varies from 5 to 15%. The only way to lower the
mortality is aggressive resuscitation and treat any concurrent
illness. Failure to treat or delay in diagnosis always leads to high
morbidity and mortality.

Source: https://www.ncbi.nlm.nih.gov/books/NBK482134/

13. What of the following statements about neutrophil is


INCORRECT?
A. It is the most abundant granulocyte
B. It is the first cell to arrive at the site of the injury
C. It is a motile and phagocytic cell
D. Its granules contain histamine
Rationale:
Source: https://www.who.int/news-room/commentaries/detail/criteria-
for-releasing-covid-19-patients-from-isolation

Source: Philippine Clinical Practice Guidelines on UTI 2015 Update:


Part 2

15. The completion of this phase allows the


pharmaceutical company to apply to FDA for
approval to market the drug:
GG 13th ed (Ch 34 p621) A. Phase I
B. Phase II
C. Phase III
D. Phase IV
Rationale:
The FDA-regulated clinical trials typically are conducted in four phases.
Phases I-III are designed to establish safety and efficacy, while
phase IV postmarketing trials delineate additional information
regarding new indications, risks, and optimal doses and
schedules. When initial phase III trials are complete, the sponsor
(usually a pharmaceutical company) applies to the FDA for
approval to market the drug; this application is called either an
NDA or a BLA.
14. Which of the following scenario practices Evidence-
Based Care? GG 13th ed (Ch1 p6)
A. Advising patient to buy UV lamps to prevent
COVID transmission 16. Which of the following statements is TRUE of
B. An asymptomatic patient is set free after a Basophils?
10-day quarantine in a COVID facility A. Basophils play a role in allergic response
C. Advising patient to drink cranberry juice to cure B. Basophils are mononuclear phagocytes
UTI C. Basophils are cells of the adaptive immune
D. All of the above system
Rationale: D. Basophils express surface receptors called TCR.
Rationale:
A. Basophils and mast cells have granules that contain histamine and
other pharmacologically active substances. In addition to their
protective function, these cells can become dysregulated during
the generation of allergic responses, in which they play an
important role. (GG,Ch34, p621)
B. Mononuclear phagocytes consist of monocytes and macrophages.
(GG,Ch34, p621)

BK23 | TABLE OF CONTENTS


C. The major effectors of innate immunity are complement, A. Polyvalent Pneumococcal vaccine
granulocytes, monocytes/macrophages, NK cells, mast cells,
and basophils. The major effectors of adaptive immunity are B
B. Meningococcal polysaccharide vaccine
and T lymphocytes. (GG,Ch35, p637) C. Meningococcal oligosaccharide diphtheria
D. The T lymphocytes, also known as T cells, express cell surface conjugate vaccine
pathogen receptors called TCRs. (GG, Ch 34, p622 D. Tetanus toxoid
Rationale:
17. Hepatotoxicity resulting from drug metabolism (not sure)
generally occurs from:
Subunit vaccines do not contain live pathogens; rather, subunit
A. Direct toxic effect of the drug to the hepatocyte
vaccines use a component of the microorganism as a vaccine
B. Phase 1 reaction antigen to mimic exposure to the organism itself. Subunit
C. Phase 2 reaction vaccines typically contain polysaccharides or proteins (surface
D. All of the above proteins or toxoids). Compared to live attenuated vaccines,
subunit vaccines induce a less-robust immune response.
E. A and B only
- Polysaccharide subunit vaccines are not very immunogenic.
Rationale:
(Eg. polyvalent pneumococcal vaccine, meningococcal
(not sure)
polysaccharide vaccine)
Xenobiotic-metabolizing enzymes have historically been categorized
- Conjugate subunit vaccines use a technology to bind
as
polysaccharide from the bacterial capsule to a carrier
• phase 1 reactions, which include oxidation, reduction, or hydrolytic
protein, often diphtheria or tetanus toxoid. This sort of
reactions; or
antigen combination can induce long-term protection in
• phase 2 reactions, in which enzymes catalyze the conjugation of the
infants and adults. These vaccines provide protection
substrate (the phase 1 product) with a second molecule.
against pathogens where plain polysaccharide vaccines fail
to work in infants and also provide more long-term protection
While xenobiotic-metabolizing enzymes facilitate the elimination of
in young children and adults. (Eg. Meningococcal
chemicals from the body, paradoxically these same enzymes
oligosaccharide diphtheria conjugate vaccine)
can also convert certain chemicals to highly reactive, toxic, and
- Toxoid vaccines, are effective because they elicit an immune
carcinogenic metabolites. Depending on the structure of the
response that results in the production of antibodies that can
chemical substrate, xenobiotic-metabolizing enzymes can
bind and neutralize these toxins, preventing cell damage in
produce electrophilic metabolites that react with nucleophilic
the patient (Eg. tetanus toxoid)
cellular macromolecules such as DNA, RNA, and protein. This
can cause cell death and organ toxicity. Most drugs and other
xenobiotics that cause hepatotoxicity damage mitochondria and
lead to hepatocyte death.

GG 13th ed (Ch 6 p86)

18. The following are true regarding the use of thiazide


diuretics in hypertension?
A. Maximum dose is 12.5mg per day
B. It does not cross the placenta and is therefore
safe in pregnancy
C. It can lead to changes in plasma lipids and
glucose tolerance
D. It is the antihypertensive of choice of choice for
patients with gout.

Rationale:
A. Clinical trials of antihypertensive therapy in the elderly demonstrated
the best outcomes for cardiovascular morbidity and mortality
when 25 mg of hydrochlorothiazide or chlorthalidone was the
maximum dose given. (GG, p510)

B. All thiazide-like drugs cross the placenta. While they have no direct
adverse effects on the fetus, administration of a thiazide during
pregnancy increases a risk of transient volume depletion that
may result in placental hypoperfusion. (GG, p512)

C. Thiazide diuretics have also been associated with changes in


plasma lipids and glucose tolerance that have led to some GG 13th ed (Ch36 p659-660)
concern. (GG, p512)
20. Neurotoxicity is a dose-limiting for which of the
D. Gout may be a consequence of the hyperuricemia induced by these
diuretics. The occurrence of either of these adverse effects is a following drugs?
reason for considering alternative approaches to therapy. (GG, A. Etoposide
p512) B. Methotrexate
C. Paclitaxel
19. Which of the following subunit vaccines would be D. Vincristine
more immunogenic and would provide long term Rationale:
protection? Vincristine

BK23 | TABLE OF CONTENTS


- Intravenous administration; extravasation causes irritation and 24. A drug which promotes the opposite intrinsic effects
ulceration
- Reduce dose in patients with impaired liver function
of the receptor:
- Least myelosuppressive Vinca alkaloid A. Allosteric antagonist
- Dose-limiting neurotoxicity B. Competitive antagonist
- Better tolerated by children than adults C. Functional antagonist
D. Inverse agonist
GG 13th ed (Ch66 p1199)
Rationale:
Allosteric Agonist- binding of an antagonist to a different site of the
21. Which of the following antimicrobials has poor CSF receptor reducing the number of receptors of the agonist
penetration capacity? Competitive antagonist- reversible binding . Binding of antagonist id
A. Piperacillin surmount
Functional antagonist- antagonist binding to a different receptor
B. Cefepime
initiating a completely opposite effect
C. Doripenem
D. Penicillin G
Rationale:

Piperacillin- Poor CSF penetration


Cefepime- Good CSF penetration
Doripenem
Penicillin G - CSF penetration with inflammation 25. Which of the following is CORRECT in the
G&G 13th ed pp 1036-1037
SYSTEMS APPROACH in patient safety and health
care system?
A. Examine organizational factors that underpin
dysfunctional health care
B. Blame the individual concerned with the issue
22. Physiologic receptor activated by insulin: C. HC worker is the victim during adverse effects
A. Ion-gated D. Limit reporting of adverse events and control the
B. GPCR dysfunction within yourselves
C. Transmebrane-linked Rationale:
D. Nuclear
Rationale:

Patient Safety PPT

26. In DSM- V which of the following is a


pharmacological criterion in the diagnosis of
substance use disorder?
A. Craving
B. Appearance of withdrawal symptoms
C. Hazardous use
G&G 13th ed page 40 D. Repeated attempts to quit
ALL OF THE ABOVE?
Rationale:
23. A 55-year-old man who is on chronic pain medication
was unconscious on the floor. An overdose was
suspected. Which of the following drugs was likely
taken?
A. Codeine
B. Diazepam
C. Phenobarbital
D. Lorazepam
Rationale:
Codeine- opioid
Diazepam and Lorazepam - more commonly used for sedation
Phenobarbital- long-acting barbiturate

G&G 13th ed page 668 Drug Abuse PPT

BK23 | TABLE OF CONTENTS


27. The preferred carbapenem for the treatment of 30. A 40-year-old breast cancer patient is receiving
meningitis because of low likelihood of cause Tamoxifen. What laboratory diagnostics will you
seizures: monitor?
A. Imipenem A. Activated thromboplastin time
B. Meropenem B. Fasting blood sugar
C. Doripenem C. Creatinine
D. Ceftolozane with tazobactam D. Triglyceride level
Rationale:

Rationale:

G&G 13th ed page 1035


28. You are attending a patient who is receiving
Flurouracil. Which of the following symptoms would
necessitate you to immediate discontinuation of the
medication?
A. Palmar-plantar erythrodysesthesia
B. Ataxia
C. Constipation
D. Insomnia
Rationale: GG, 13th ed 1240

31. Which of the following are potassium sparing


diuretics?
A. Chlothalidone
B. Indapamide
C. Furosemide
D. Triamterene
Rationale:
GG 13th ed p. 469

Katzung 13th ed page 929

29. Maintaining “continuity of care” in the health care


system is/are shown in the following situations:
A. Following the guidelines and protocols of the
hospital
B. Attending and learning from the morbidity and 32. Abraxane is often used to reduce hypersensitivity to
mortality meetings as well as briefings prior to which of the following drugs?
surgery A. Etoposide
C. Calling the ICU doctor from the ER and B. Paclitaxel
discussing the patients case before transport C. Vinblastine
D. Continuous reassessment of the staffs D. Vincristine
Rationale:
knowledge and practice of patient safety Katzung 14th ed p. 963
standards and improve if standard goals are not
met.
Rationale:
Not sure.
A. Practicing evidence-based care

33. A non-beta lactam beta lactamase inhibitor that


provides clinically useful inhibition against ESBL
type beta lactamases:
A. Avibactam
B. Tazobactam

BK23 | TABLE OF CONTENTS


C. Sulbactam
D. Clavulanic acid
GG 13th ed p. 1036

34. Which of the following monoclonal antibodies is used


in the treatment of asthma? 38. You are evaluating the medical history of a patient
A. Abciximab who will be receiving Asparaginase. Which of the
B. Omalizumab following should be noted in the history?
C. Secukinumab A. Diabetes mellitus
D. Palivizumab B. Pancreatitis
Rationale: C. Bronchial asthma
D. Ischemic heart disease
Rationale:
GG 13th ed p. 1195

GG 13th ed p. 742

35. This ECG tracing represents the depolarization of


the ventricles:
A. P wave
B. QRS waves
C. T wave
D. U wave
Rationale:
Bates’ 12th ed p. 352
Rationale:
p. 1200

36. TRUE of dose-response relationship: 39. TRUE of Steady-state concentration:


A. Drugs with high receptor affinity has low A. Fluctuations are proportional to the dosing
dissociation constant (KD) interval
B. Quantal response curve is bell-shaped B. Proportional to the drug clearance
C. Graded response measures whether a drug has C. Attained after approximately 3 half-lives
an effect or no effect to a population D. All of the above
D. All of the above E. A and B only
E. A and B only Rationale:
Rationale: GG 13th ed p. 25
GG 13th ed p. 34

37. Which of the following classes of cancer


chemotherapy agents bind to tubulin and cause
arrest of cells in the metaphase?
A. Alkylating agents
B. Antimetabolites
C. Antitumor antibiotics
D. Vinca alkaloids 40. What is the body’s first line of defense against
Rationale: pathogens?
Katzung 14th ed p. 961 A. Antibodies

BK23 | TABLE OF CONTENTS


B. Cytokines E. A and B only
C. Skin Rationale:
Not sure
D. Lymphatics Reference: GG ch 3 p 35-36
Rationale:
GG 13th ed p. 623

41. The following statements are true regarding ACE


inhibitors in hypertension: 44. Normal QRS duration
A. Preferred initial agent for patients with diabetic A. ms
glomerulopathy B. 100-110 ms
B. Cough is a common side effect C. 90-100 ms
C. Contraindicated in pregnancy D. Less than 0.44 sec
D. All of the above E. 120-200
Rationale: Rationale:
GG ch 28 p. 516-517

45. Rachitic rosary is found in this vitamin deficiency


A. Vitamin A
42. Which of the following is considered a secondary B. Vitamin B
lymphoid organ? C. Vitamin C
A. Bone marrow D. Vitamin D
B. Liver Rationale:
C. Spleen
D. Thymus net
Rationale:

46. A natural form of Vitamin K that is synthesized by


bacterial flora:
A. Vitamin K1
B. Vitamin K2
C. Vitamin K3
43. TRUE of ANTAGONISM: D. Vitamin K4
A. Competitive antagonists produce covalent Rationale:
net
binding to the receptor
B. A non-competitive antagonist shifts the agonist’s
dose-response curve to the right
C. Allosteric antagonism is surmountable
D. All of the above

BK23 | TABLE OF CONTENTS


51. Which of the following has the highest incidence of
skin rashes among the penicillins?
A. Penicillin V
B. Ampicillin
C. Amoxicillin
D. Oxacillin

47. TRUE of estimated plasma drug concentration:


A. It is expressed in concentration per unit volume
B. It is inversely proportional to the volume of
distribution
C. It is directly proportional to the drug
bioavailability
D. All of the above
Rationale:
Not sure

48. What condition/situation has been associated with


the development of Guillain Barre Syndrome?
A. Measles infection
B. Influenza infection
C. After administration of varicella vaccine
D. Use of thiomersal preservative in vaccine
Rationale:
52. ACE inhibitors and angiotensin receptor blockers are
excellent antihypertensive agents for patients with
heart failure. However, these drugs are not very
effective for the African American population. This is
mainly due to:
A. Low renin levels in the African American
49. The ideal time to collect blood sample in Therapeutic population
Drug Monitoring for dose adjustment: B. Noncompliance with medications
A. peak of blood level of the drug C. Increased hyperkalemia
B. At the lowest blood level of the drug D. All of the above
C. At the steady-state concentration of the drug Rationale:
D. At any time within the Area Under the Curve Compared with other classes of antihypertensive agents, there may be
a greater frequency of achieving blood pressure control with Ca
Rationale: 2+ channel blockers as monotherapy in elderly subjects and in
GG ch 2 p 29 African Americans, population groups in which the low renin
status is more prevalent. GG 13th ed (Ch 28, p516)

53. What is the cofactor form of Vitamin B6?


A. Flavine adenine dinucleotide
50. Which of the following is correct in the proper use of B. Pyridoxal phosphate
the prescription pad? C. Thiamine pyrophosphate
A. Prescription is imprinted with the name of the D. Methylcobalamine
physician and residential address Rationale:
B. The physician should print his name and Flavine adenine dinucleotide - redox active coenzyme
phone number when using an institutional Thiamine pyrophosphate- cofactor that is present in all living systems
prescription blank Methylcobalamine- Vitamin B12 active coenzyme
C. Prescription written in pencil is acceptable
D. Date of the prescription is unimportant part of 54. Which of the following ECG pattern most likely
the prescription signify hypocalcemia?
Rationale: A. Prolonged QRS
B. Shortened QRS
C. Prolonged QT interval

BK23 | TABLE OF CONTENTS


D. Shortened QT interval B. Non-selective beta blockers
Rationale: C. Non dihydropyridine calcium channel blockers
ECG Abnormalities Associated With Hypocalcemia
D. Sympatholytic agents
DOI:https://doi.org/10.1378/chest.119.2.668-a Rationale:
A major precaution regarding the use of the α 1 blockers for
hypertension is the so-called first-dose phenomenon, in which
symptomatic orthostatic hypotension occurs within 30–90 min (or
longer) of the initial dose of the drug or after a dosage increase.
(G&G 13th Ed pg. 514)

58. The following lifestyle modifications can help lower


blood pressure in patients with grade 1
55. Which of the following is a direct renin inhibitor? hypertension EXCEPT:
A. Phenoxybenzamine A. Smoking cessation
B. Reserpine B. Moderating alcohol consumption to 2 drinks per
C. Clonidine day for men and 1 drink per day for women
D. Aliskiren C. Restricting potassium intake to 1-2 grams
Rationale: per day
GG, 13th ed ch 28 p.
D. Increasing aerobic exercise
Rationale:

56. Which of the following statements about Yellow fever (GG 13th ed. page 508)
vaccine is correct?
A. It is approved for all ages
B. It is a live attenuated vaccine
C. It is administered upon arrival in the endemic
area
D. It is administered as multiple doses

59. Which of the following vaccines is considered live


bacterial vaccine?
A. Haemophilus B conjugate vaccine
B. Oral cholera vaccine
C. Plague vaccine
D. Rabies vaccine
Rationale: Rationale:

GG, 13th ed page 659

60. A drug of abuse not known to be addictive


A. LSD
B. Ecstasy
C. GHB
D. Diazepam
G&G 13th ed page 668 Not sure. Diazepam is a benzodiazepine

57. First dose phenomenon is characterized by


61. Angiotensin receptor blockers exert antihypertensive
symptomatic orthostatic hypotension which occurs
effects by blocking AT1 receptors. This can lead to
30-90 mins after initial dose of the drug. This is a
the following effects EXCEPT:
major precaution for hypertensive patients given
A. Relaxation of smooth muscles and vasodilation
which class of drugs?
B. Decrease plasma volume
A. Alpha 1 blockers

BK23 | TABLE OF CONTENTS


C. Decrease cellular hypertrophy
D. The inhibit ACE mediated degradation of
bradykinin
Rationale:

G&G 13th ed page 518 G&G 13th ed page 631

62. Which of the following is NOT a responsibility of the 64. Mary, 30 years old, is 28 weeks pregnant with no
FDA? known comorbids except for asthma and allergic
A. Ensure safety of human and veterinary drugs, rhinitis. She has few allergies including amoxicillin.
biological products, cosmetics and medical Her STI panel showed REACTIVE for syphilis test,
devices the obstetrician referred the patient to you for
B. Ensure that medicines and foods are affordable treatment. What would be you antimicrobial of
C. Ensure public is given science-based choice?
information about the use of foods and drugs A. Azithromycin
D. Ensure that foods are supplied to the public B. Penicillin G
Rationale: C. Penicillin V
GG, 13th ed page 5 D. Cefixime

Rationale:

fda.gov

G&G 13th ed page 1028


63. After ingestion of aspirin, the patient developed
swelling of the eyelids and mouth and was
complaining of difficulty breathing. What type of 65. This term is used to describe a pharmaceutical that
hypersensitivity reaction is the patient is usually structurally similar to a drug already on
experiencing? the market
A. Type I hypersensitivity reaction A. Follow-up drugs
B. Type II hypersensitivity reaction B. Me-too drugs
C. Type III hypersensitivity reaction C. Molecular modifications
D. Type IV hypersensitivity reaction D. All of the above
Rationale:
Rationale:

BK23 | TABLE OF CONTENTS


OTC PPT

66. Helps modulate G-Protein receptors


A. GRKs
B. RGS proteins
C. Arrestins G&G 13th ed page 513
D. All of the above
E. A and B only 69. Factor/s that modify the distribution of drugs:
Rationale: A. Lipophilicity
B. Tissue binding
C. Capillary permeability
D. All of the above
E. A and B only
Rationale:

G&G 13th ed page 42

67. Some beta blockers have intrinsic sympathomimetic


activity. These agents are not recommended for
treatment of hypertension since they can increase
night time mean heart rate. An example of this type
of beta blocker includes:
A. Pindolol
B. Bisoprolol
C. Propranolol G&G 13th ed page 18
D. Carvedilol 70. The following characteristics are important for a drug
Rationale:
molecule to chemically interact with the receptor
EXCEPT:
A. Atomic composition
B. Electrical charge
C. Shape
D. Size
E. None of the above
Rationale:
G&G 13th ed page 513 - Beta blockers

68. The following statements are true regarding


therapeutic uses of beta blockers:
A. Highly effective in the elderly and African
population
B. Preferred antihypertensive for patients with
ischemic heart disease and heart failure
C. They can cause retention of salt and water thus
co administration of thiazide diuretic is
recommended
D. Preferred antihypertensive for patients with high
risk for stroke
Rationale:

BK23 | TABLE OF CONTENTS


G&G 13th Ed. pg. 556

73. Which of the following can elicit a robust immune


response?
A. Typhoid Vi polysaccharide vaccine
B. Measles, mumps, rubella vaccine
C. Rabies vaccine
G&G 13th ed page 33 D. Plague vaccine
Rationale:
Live attenuated vaccines use a weakened form of a virus that contains
71. A 4thgeneration cephalosporin has no activity antigens that appropriately stimulate an immune response.
against this/these following causative agents: These attenuated viruses can elicit a robust immune response
A. Streptococcus pyogenes because they are similar to the natural pathogen. (G&G 13th Ed.
B. Neisseria gonorrhoeae pg. 658-659)
Typhoid Vi polysaccharide vaccine - bacterial polysaccharide;
C. Mycoplasma pneumoniae Measles, mumps, rubella vaccine - live attenuated; Rabies
D. All of the above vaccine - inactivated or subunit; Plague vaccine - inactivated
Rationale: bacterial (G&G 13th Ed. pg. 659)

74. What is NOT part of your considerations in the use of


live vaccines?
A. Indications and contraindications
B. Maintenance of the cold chain
C. Possibility of viral mutation
D. Tolerance to vaccine
Rationale:
Several drawbacks exist with these vaccines. Because these are live
viruses, they generally must be refrigerated to retain their
activity (maintenance of the cold chain). In remote areas of
the world where refrigeration is not available, obtaining and
storing this type of vaccine can be limiting. Because viruses can
mutate and change in the host (possibility of viral mutation),
it may be possible that viruses can become virulent again and
G&G 13th Ed. pg. 1034
cause disease, although the frequency of adverse reactions
using these vaccines is very low. Furthermore, attenuated
vaccines cannot be utilized in immune-compromised
individuals (e.g., patients with HIV or cancer). In addition, these
vaccines are usually not given during pregnancy (indications
and contraindications). (G&G 13th Ed. pg. 659)

75. An individual’s increase drug response can be due


G&G 13th Ed. pg. 1037
to:
A. Tolerance
B. Upregulation of receptors
72. 1 60-year-old patient went to the ER due to sudden C. Tachyphylaxis
onset of dizziness. Lab work up done. ECG showed D. Desensitization
absence of P wave, narrow QRS with irregularly Rationale:
irregular rhythm, with a HR of 115/min and no ST
segment elevation noted. What is your most likely
impression?
A. Sinus rhythm with nonspecific ST T wave
changes
B. 1st degree AV block
C. Atrial fibrillation
D. Ventricular fibrillation
Rationale:

BK23 | TABLE OF CONTENTS


76. Which of the following statements is NOT TRUE
about Phase 1 clinical trial in drug development?
A. Its goal is to determine the dose effect of drugs
in healthy volunteers
B. If the study drug is expected to have significant
78. Which of the following beta lactams is preferred for
toxicity, volunteer patients with the disease will
neonates due to its low potential in causing
participate in the study neonatal jaundice?
C. Absorption, half-life and metabolism of the
A. Ceftriaxone
experimental drug are studies
B. Cephalorine
D. It studies on the efficacy and dose ranging of
C. Cefoperaone
the drug
Rationale:
D. Cefotaxime
Rationale:
Study on the efficacy and dose ranging of the drug happens in Phase II
The high binding affinity of ceftriaxone for serum albumin may displace
clinical trial.
bilirubin, potentially causing jaundice in neonates; for this
reason, cefotaxime is the preferred agent in this patient
population. (G&G 13th Ed. pg. 1034)

79. Which of the following is an OTC drug?


A. Aspirin
B. Bisacodyl
C. Levoceterizine
D. Warfarin

G&G 13th Ed. pg. 6

77. All addictive drugs are believed to activate the:


A. Mesolimbic dopamine pathway
B. Mesolimbic serotonin pathway
C. Mesocortical dopamine pathway
D. Mesocortical serotonin pathway

Rationale:

80. A client is receiving Methotrexate for a series of


chemotherapy. Which of the following medications
should be readily available to avoid toxicity?
A. Mesna

BK23 | TABLE OF CONTENTS


B. Oseltamivir
C. Vigabatrin
D. Leucovorin
Rationale:
Leucovorin may be administered to counteract the potential toxicity of
MTX that escapes into the systemic circulation, although this
generally is not necessary. (G&G 13th Ed. pg. 1179)

81. The following statement/s is/are if Drug A and Drug


B are synergistic:
A. The combined response is greater than each
of the response of Drug A and Drug B
B. The EC50 of Drug A and Drug B will be lower
C. Drug A has no effect but enhances the effect of
Drug B
D. All of the above
E. A and B only
Rationale: (GG 13th ed. Page 658)

84. Distribution of blood flow is highest in the:


A. Kidneys
B. Heart
C. Brain
D. Liver
Rationale:

(step1.medbullets.com)

(Doc. Bongon’s PPT ) 85. Which of the following vaccines SHOULD BE


AVOIDED in pregnant women?
82. An individual can be given 4 different types of A. Hepatitis B vaccine
vaccines in a single day? B. MMR
A. Yes C. Pneumococcal vaccine
B. No D. Tetanus toxoid
C. Still under investigation Rationale:
Rationale:

(immunize.org)

83. Which of the following conditions does not involve


passive immunity?
A. Administration of anti-thymocyte globulin
B. Breast feeding an infant
C. Administration of botulism antitoxin
D. Administration of BCG

(GG 13th ed. page 667)

Rationale:

86. TRUE of ABSORPTION


A. The non-ionized formed of the drug is
absorbed

BK23 | TABLE OF CONTENTS


B. Highly lipophilic drugs can be poorly absorbed in
the GUT
C. Energy is always not necessary
D. All of the above
E. A and B only
Rationale:

(pharmacologyeducation.org)

87. Which of the following immunoglobulins has the


longest half-life and provides passive immunity to
the fetus?
A. IgA
B. IgE
C. IgG
D. IgM
Rationale:
(Patient Safety Guide, WHO p. 30)

90. Which of the following rarely develops withdrawal


symptoms?
A. Nicotine
B. Morphine
C. Marijuana
D. Lorazepam
Rationale:

(GG 13th ed. page 662)

88. “Flashbacks” is commonly encountered in:


A. Alcohol GHB
B. LSD
C. Cannabis
Rationale:

(GG 13th ed. Pages 440-441)

91. Parameters required to obtain the LOADING DOSE


of a drug:
A. Desired steady state plasma concentration
B. Bioavailability
C. Drug clearance
D. All of the above
E. A and B only
Rationale:
Loading dose = Vd x Desired Css / F (bioavailability)
( Based on pharmacokinetics ppt )

(Doc Bongon’s PPT) 92. A 40-year-old woman with breast cancer Stage III is
undergoing chemotherapy. She is on her 3rd cycle
of chemotherapy when she developed fever, chills,
89. The idea of “incident reporting” is an activity that abdominal pain and LBM with associated
fulfills which of the following principles of patient neutropenia. What vaccine could be given to the
safety: patient once neutropenia resolves?
A. Being an effective team player A. Oral cholera vaccine
B. Understanding and managing clinical risks B. Oral typhoid vaccine
C. Engaging patient and their careers C. Oral rotavirus vaccine
D. Improving medication safety D. A and B only
Rationale: E. None

BK23 | TABLE OF CONTENTS


Rationale: B. Cytarabin
Typhoid fever is an acute illness caused by the bacterium S. typhi,
which is transmitted by ingestion of contaminated water or food.
C. Etoposide
Typhoid vaccination is recommended for international travelers D. Vincristine
who will visit rural areas or villages that have inadequate Rationale: ( katzung , page 951)
sanitation. Symptoms include fever, headache, anorexia, and
abdominal discomfort; the disease can be fatal. Treatment is Cell cycle specific
challenging, and there has been an increase in the number of Antimetabolites - Cytarabine
drug-resistant strains of S. typhi over the last several decades. ( Topoisomerase II inhibitor - Etoposide
GG, page 668) Vinca alkaloids - Vincristine

93. Which of the following cell would the MHC class II 97. Tamoxifen is an older agent but has been used
cell present the exogenous peptide molecule? extensively to treat estrogen receptor positive (ER+)
A. CD 4 cell breast cancer. It is dosed per orem ad some QT
B. CD 8 cell prolongation could be associated with use of the
C. CD 28 cell agent. Which is true about this agent?
D. CD 86 cell A. It is metabolized into active metabolite
Rationale: B. It can possibly predispose patients to
MHC class II molecules consist of two noncovalently associated hypocalcemia
transmembrane glycoproteins, an α chain and a β chain. MHC
class II molecules are primarily expressed on the surface of
C. Tamoxifen inhibits CYP19A
professional APCs (DCs, macrophages, B cells) and present D. None of the above
peptides from exogenous antigens to CD4 T H cells. (GG page Rationale: (GG page 1238)
627) Metabolism of tamoxifen is complex and principally involves CYPs
3A4/5, and 2D6 in the formation of N-desmethyl tamoxifen and
CYP2D6 to form 4-hydroxytamoxifen, a more potent metabolite
94. This part of prescription is the instruction to the (Figure 68–1). Both metabolites can be further converted to 4-
pharmacist: hydroxy-N-desmethyltamoxifen (endoxifen), which retains high
A. Superscription affinity for the ER.
B. Inscription
C. Subscription 98. A client with a sarcoma is receiving Bleomycin. If you
D. Signa were the attending physician, which of the following
Rationale: diagnostic procedures will you prioritize in the
monitoring of its side effect?
superscription - includes the date the prescription order is written; the
A. Pulmonary function test
name, address, weight, and age of the patient; and the Rx
(Take). B. Cranial x-ray
inscription -contains the name and amount or strength of the drug to be C. Stress test
dispensed or the name and strength of each ingredient to be D. Electroencephalograph (EEG)
compounded. Rationale:
subscription- is the instruction to the pharmacist, usually consisting of a
short sentence, such as “dispense 30 tablets.” There is no known specific therapy for bleomycin lung injury except for
signa - or “Sig” is the instruction for the patient regarding how to take symptomatic management and standard pulmonary care.
the prescription Steroids are of variable benefit, with greatest effectiveness in the
earliest inflammatory stages of the lesion. ( GG page 1193)

99. Used to treat Nicotine craving:


A. Naloxone
B. Bupropion
C. Flumazenil
D. Mecamylamine
Rationale:
95. A 25-year-old man arrived at the ER tachycardic, The antidepressant bupropion is approved for nicotine cessation
hypertensive, profusely sweating, hyperthermic and therapy. It is most effective when combined with behavioral
with altered mental status. He was brought in from a therapies.
(katzung, page 584)
rave party where drugs were apparently distributed
to the crowd. Which of the following exposure is
least likely in this patient? 100. BIOAVAILABILITY from highest to lowest as to
A. The patient probably overdosed with MDMA route of administration:
B. The patient is severely intoxicated with alcohol A. Oral > Intravenous > Subcutaneous > Rectal
C. The patient snorted cocaine for the first time B. Intravenous > Transdermal > Rectal > Oral
D. The patient took “bath salts” C. Intravenous > Oral > Inhalation > Transdermal
Rationale: D. Oral > Inhalation > Transdermal > Rectal
In intermittent users of cocaine, the euphoric effect typically is not Rationale:
subject to sensitization. ( GG page 440)
intravenous - bioavailability is complete (F = 1.0) and distribution is
rapid
96. Which of the following chemotherapeutic agents is a
transdermal - absorption through the skin can be enhanced by
cell cycle non-specific agent? suspending the drug in an oily vehicle and rubbing the resulting
A. Busulfan preparation into the skin.

BK23 | TABLE OF CONTENTS


rectal - 50% of the drug that is absorbed from the rectum will bypass
the liver, thereby reducing hepatic first-pass metabolism.
However, rectal absorption can be irregular and incomplete
oral - bioavailability is potentially erratic and incomplete
( GG pp 16-18)

BK23 | TABLE OF CONTENTS


Pharmacology 2_1 A. Prevents degradation of atrial and brain
natriuretic peptides thus promoting
1. What medication is appropriate for the treatment of natriuresis
chronic idiopathic cough? B. Prolongs half-life of valsartan
A. Acetazolamide C. Increases myocardial contractility
B. Guaifenesin D. All of the above
Goodman & GIllman (13thed) Chapter 29 page 535
C. N-acetyl-cysteine
D. Pregabalin

Goodman & Gilman (13thed) Chapter 40 page 744

2. Positive inotropic agents are indicated in acutely


decompensated heart failure particularly if there are
signs of critically low cardiac output and poor
perfusion of vital organs. The beta adrenergic agent
of choice for the management of acute CHF is:
A. Dopamine
B. Dobutamine
C. Epinephrine
D. Norepinephrine
6. Lalisa M. is a 20-year-old call center agent who
came to your clinic for shortness of breath which
occurs 4 to 5x a week occurring at night and early
morning. This was associated with dry cough. She
first noted these symptoms when she was put on a
From Heart Failure PPT of the lecturer graveyard shift about 3 months ago. Spirometry
showed an obstructive defect with significant
3. Symptom control in asthma can be assessed by response to bronchodilator. Based on the GINA
determining: 2020, what would be the initial treatment of choice
A. Frequency of daytime symptoms for Lalisa?
B. Frequency of night-time symptoms A. As needed low dose ICS/formoterol
C. Use of ICS LABA B. Daily low dose ICS or as needed low dose
D. A and B ICS/formoterol
E. All of the above C. Low dose ICS/LABA plus as needed SABA
D. Medium dose ICS/LABA plus as needed
SABA
GINA 2020 p. 24

Step 4: Medium dose ICS/LABA plus as needed SABA


- symptoms most days, or waking with asthma once a week or
more, or low lung function

4. Pathologic cardiac remodeling is one of the key 7. Which of the following is not an indication for
processes that lead to heart failure. This includes: Omalizumab?
A. Myocyte hypertrophy due to cardiac overload A. Anaphylaxis prevention during immunotherapy
B. Increase capillary myocyte ratio B. COPD exacerbations
C. Normal gene expression of ion channels C. Poorly controlled severe asthma
D. All of the above D. Severe allergic rhinitis with asthma
Cardiac remodeling = hypertrophy, fibrosis, and cell death, Goodman & Gilman (13thed) Chapter 40 page 742
p 529 13th ed

5. The mechanism of action of neprilysin inhibition


includes

BK23 | TABLE OF CONTENTS


8. The following drugs can be considered for Stage B 11. A 45-year old male, Filipino came in for severe
heart failure patient, EXCEPT: dyspnea. He is hypertensive, diabetic, and was
A. ACEI or ARB previously admitted for acute myocardial infarction 1
B. Beta blocker month prior.
C. Cardiac glycosides
D. None of the above Upon examination, BP of 150/80 mmHg, HR 112bpm, RR
Page 531, GG 26cpm. Elevated JVP, rales were noted on both lung
fields, S3 gallop is appreciated on auscultation. He
9. A 45-year old male, Filipino came in for severe has grade 3 bipedal edema.
dyspnea. He is hypertensive, diabetic, and was
previously admitted for acute myocardial infarction 1 What is your choice of afterload unloader for this patient?
month prior. A. ACE inhibitor or ARB
B. ACE inhibitor plus ARB
Upon examination, BP of 150/80 mmHg, HR 112bpm, RR C. Hydralazine – Isosorbide dinitrate
26cpm. Elevated JVP, rales were noted on both lung combination
fields, S3 gallop is appreciated on auscultation. He D. All of the above
has grade 3 bipedal edema. Drugs used for Afterload Reduction includes Hydralazine-Isosorbide
dinitrate: ACEI and ARBS have no indication for routine
combination.
What stage of heart failure is the patient in?
A. Stage A GG page 536
B. Stage B
C. Stage C 12. Mary was discharged from hospital after 5 days of
D. Stage D hospitalization due to asthma exacerbation. She was
GG 13th ed, p. 531 given Prednisone for 1 week as one of her discharge
(Not sure, but it says stage D when patient has “refractory heart meds. When is the best time to take oral steroids?
failure”)
A. Once daily in the morning
10. A 45-year old male, Filipino came in for severe B. Twice daily after meals for sustained effects of
dyspnea. He is hypertensive, diabetic, and was the drug
previously admitted for acute myocardial infarction 1 C. Once daily at night to minimize night symptoms
month prior. D. Anytime of the day

Upon examination, BP of 150/80 mmHg, HR 112bpm, RR


26cpm. Elevated JVP, rales were noted on both lung
fields, S3 gallop is appreciated on auscultation. He
has grade 3 bipedal edema.

Based on the stage of heart failure of this patient, what


heart failure treatment should be your top priority?
A. Enalapril
B. Losartan
C. Metoprolol
D. Furosemide

13. Which medication is administered in cases of opioid


overdose?

BK23 | TABLE OF CONTENTS


A. Alimtrine Captopril - 1.7 hrs
Enalapril - 11 hrs
B. Naloxone Ramipril - 13-17 hrs
C. Flumazenil Trandolapril - 15-23 hrs
D. Theobromine
B. Naloxone - antagonizes opioid action GG 13th ed. Page 533 table 29-2
A.respiratory stimulant that enhances respiration; used in treating
COPD 18. Which of the following bronchodilator enhances IL-
C. used in treating benzodiazepine overdose in adults 10 release?
D. used as vasodilator in asthma; also a heart stimulant
A. Theophylline
14. Which of the following is NOT a pharmacologic B. Glycopyrronium
characteristic of Zileuton? C. Magnesium sulfate
A. It is associated with hepatic dysfunction D. Ipratropium
Interleukin 10 release. IL-10 has a broad spectrum of anti-inflammatory
B. It is associated with the development of effects, and there is evidence that its secretion is reduced in
Churg-Strauss syndrome asthma. IL-10 release is increased by theophylline, and this
C. It inhibits the synthesis of LTB4 effect may be mediated via inhibition of PDE activities, although
this has not been seen at the low doses that are effective in
D. It almost has a similar efficacy to antihistamines. asthma.

GG 13th ed. Page 733

19. Which of the following is a characteristic of


hydrocortisone?
A. Provides maximal beneficial effect
B. Rapid onset
C. Long duration of action
D. Lower systemic absorption
15. All of the following are beneficial in asthma Hydrocortisone is the steroid of choice because it has the most rapid
EXCEPT? onset (5–6 h after administration), compared with 8 h with
prednisolone.
A. Cetirizine
B. Diphenhydramine GG 13th ed. Page 739
C. Anti-leukotriene
Diphenhydramine is a first generation antihistamine. Although 20. What is the preferred route of delivery of drugs in
antihistamines may be useful in treating asthma. There are little patients with asthma and COPD?
evidence on the effectivity of the first gen drugs. However, 2nd
generation antihistamines such as cetirizine are thought to
A. Parenteral
provide beneficial effects relatively. B. Inhaled
C. Oral
GG 13th ed. Page 740
D. Topical
Inhalation is the preferred mode of delivery of many drugs with a direct
16. Indacaterol + glycopyrronium effect on airways, particularly for asthma and COP
A. LABA + ICS
B. LABA + LAMA GG 13th ed. Page 728
C. ICS + LABA
21. The following are true of heart failure with preserved
D. SAMA + LABA
LABAs include salmeterol, formoterol, and arformoterol. In addition,
ejection fraction (HFpEF) except:
there are also once-daily LABAs such as indacaterol, vilanterol, A. Typically associated with metabolic syndrome
and olodaterol. B. More common in men
LAMAs include Tiotropium bromide, ipratropium, Glycopyrronium C. Prevalence increases with age
bromide, umeclidinium bromide, and aclidinium bromide D. Left atrial size is often enlarged
GG 13th ed. Page 731-732 and 736

17. Which ACE inhibitor has the shortest half-life?


A. Enapril
B. Captopril
C. Ramipril
D. Trandolapril GG p.530

BK23 | TABLE OF CONTENTS


of dyspnea after cleaning the bathroom using
22. What is the indication for the use of dornase alfa? muriatic acid. He was then taken to the emergency
A. Asthma room. Vital signs at ER: BP – 100/80 mmHg, HR –
B. COPD 102 bpm, RR- 28 cpm, Temp – 37 C, O2 sat – 90%.
C. Cystic fibrosis Chest/lung finding: poor airway entry. Which of the
D. Severe rhinitis following should be given at the ER?
A. ICS + LABA 2 puffs now
B. LAMA + LABA 2 puffs now
C. ICS 1 puff now followed with SABA 1 puff now
D. SABA thru nebulization
Ratio: p. 745 G&G 13th Ed
Short-Acting β2 Agonists: Inhaled bronchodilators for symptom
relief and acute bronchodilation

27. These are used to reduce airway inflammation,


control symptoms, and reduce future risks of
exacerbations and decline in lung function:
A. Reliever (rescue) medication
B. Add-on therapies
C. Controller medications
D. Phosphodiesterase inhibitors
GG p.743 (GINA) recommend LABA and ICS, which are controller medications,
to reduce inflammation and reduce risk factors in the long run
23. Which of the following medications has no benefit in
the management of Bronchial Asthma? 28. Chronic neurohumoral activation produces
A. Cetirizine deleterious effects on the cardiovascular system that
B. Diphenhydramine ultimately leads to worsening of heart failure. These
C. Fluticasone effects include
D. Montelukast A. Vasoconstriction and increase in afterload
Ratio: Antihistamines are not recommended in the routine B. Decreased renal perfusion leading to sodium
management of asthma.
A. Cetirizine (p. 740 GG) newer antihistamines including cetirizine and
and water retention
azelastine, have some beneficial effects, but this may be C. Increased aldosterone secretion
unrelated to H1 receptor antagonist. D. All of the above
C. Fluticasone (p.737-740) is a corticosteroid. Because asthma is a All choices will lead to hypertension which worsens heart failure
chronic inflammatory disease, ICS are considered first line
therapy in all but patients with the mildest disease.
D. Montelukast (p. 740-741) is an antileukotriene. Anti-LT are indicated 29. Maximum bronchodilator effect brought about by
as an add-on therapy in patients who are not well controlled on SABA is achieved after:
ICS. A. 5 mins
B. 15 mins
24. Most effective bronchodilator:
C. 30 mins
A. B2 adrenergic agonist
D. 6 hours
B. Phosphodiesterase inhibitors
C. Methylxanthines
D. Muscarinic cholinergic agonists
Ratio: Bronchodilator of choice. p.730

25. Which of the following statements on anti- 30. The following can be used to document expiratory
leukotrienes is INCORRECT? airflow limitation in adults, EXCEPT:
A. Beneficial in allergic rhinitis A. Increase in FEV1 of >12% and >200ml from
B. Indicated as an add-on therapy in bronchial baseline, 10-15 minutes after 200-400mcg of
asthma salbutamol
C. Treatment of choice for mild cases of asthma B. Fall in FEV1 starting at >5% and >100ml from
D. Effective in preventing exercise- induced asthma baseline during an exercise challenge test
Ratio: p. 741 G&G 13th Ed C. Variation in FEV1 of >12% and >200ml in
Anti-LTs are considerably less effective than ICSs in the treatment of
mild asthma and cannot be considered the treatment of first between visits, outside of respiratory infections
choice D. Fall in FEV1 from baseline of ≥ 20% with
standard doses of methacholine or histamine
26. JJ 35 years old is a known asthmatic with poor
compliance to medications. He noted sudden onset

BK23 | TABLE OF CONTENTS


reducing all cause mortality in heart failure patients.
This was demonstrated by which clinical trial?
A. CONSENSUS
B. DIG Trial
C. PARADIGM HF
D. MERIT HF
(table 29-1 p 532 GG)

31. Beta adrenergic stimulation quickens the rate of


myocardial force development, otherwise known as
A. Inotropy
B. Lusitropy
C. Clinotropy
D. Dromotropy 36. (After 3 months of using low dose beclometasone
A. Inotropy - force of contraction /formoterol. Christian G still complains of having
B. Lusitropy - rate of myocardial relaxation
D. Dromotropy - electrical conduction daytime symptoms of coughing and chest tightness
(GG 13th ed; page 533) 2 to 3x a week. What should you do if you were his
doctor?
32. What is the predominant Phosphodiesterase (PDE) A. Ask him to show you how he uses hos inhaler
isoform that is targeted in the treatment of asthma and how often he uses his inhaler.
and COPD? B. Increase his dose immediately based on
A. PDE2 GINA 2020 asthma medication options.
B. PDE4 C. Tell him to just continue his usual dose as his
C. PDE6 symptoms are not that often.
D. PDE8 D. Refer him for phenotypic assessment.

33. Which of the following is/are effects of


(GINA 2020 p 36)
corticosteroids?
A. Increase the numbers of dendritic cells
B. Increase the numbers of endothelial cells
C. Upregulates B2 receptors
D. A and C
Upregulates B2 receptors (Figure 40-8, p738)
● Increases b2 receptors
● decrease dendritic cell
● Decrease leak in endothelial cells
37. Which of the following statements about theophylline
34. ACE inhibitors are among the most important is INCORRECT?
medications for heart failure. The benefits of using A. Therapeutic range is 5-15 mg/L
ACE inhibitors in heart failure include: (p 532) B. It is metabolized in the liver
A. Vasodilation and decrease in afterload C. Individualism of theophylline dosage is required.
B. Decrease aldosterone levels D. High protein-low carbohydrate diet increases
C. Sympatholytic effects theophylline clearance.
D. All of the above E. IV Aminophylline is superior to nebulized
beta 2 agonists for acute exacerbations for
35. A novel drug combination consisting of an ARB COPD.
(Valsartan) and neprilysin inhibitor (sacubitrul) has IV Aminophylline is superior to nebulized beta 2 agonists for acute
shown to be more effective than enapril alone in exacerbations for COPD. (P 734)

BK23 | TABLE OF CONTENTS


An asymptomatic stage exists that requires treatment to delay
transition to symptomatic heart failure (stage B)

38. In patients using low dose budesonide/formoterol as


both maintenance and reliever therapy, what is the
maximum dose of formoterol that they can use in a
day?
A. 48 mcg
B. 50 mcg
C. 72 mcg
D. 90 mcg
(p 746)

41. Lebron J, an 18-year old Engineering student at a


local university has been having chest tightness
associated with coughing every other day since the
start of his online classes. He did not note any
limitation of movement although he very seldom
39. Marco, 45 years old, was seen by his doctor and goes out to the house now because of the current
was given ICS + LABA. He was given such after situation. He has been using his as needed
repeated complaints of shortness of breath salbutamol metered dose inhaler 1 to 2x a day for
especially at night. He was previously on salbutamol the past 6 weeks. Last week, he noted an episode of
inhaler taken as needed. Which of the following waking up at night feeling very short of breath. This
is/are the advantage of ICS + LABA combination? was relieved after 2 puffs of his inhaler. What is the
A. The mechanisms of action of both drug level of asthma symptom control for his patient?
addresses the pathophysiology of asthma A. Well- controlled
B. Increases adherence to therapy B. Partly controlled
C. LABA shows to decrease asthma exacerbations C. Uncontrolled
D. A and B D. Cannot be assessed since he has limited
E. All of the above movement at home
3 out of 4 (UNCONTROLLED)
PLS DOUBLE CHECK SA GINA

40. A 58-year old man came in for consult regarding an 42. Gina A is a 36-year old mother who is a known
echocardiogram result. He is hypertensive, diabetic, asthmatic. She is using maintenance ICS/ LABA
and a smoker for 20 pack years. His echogram metered dose inhaler regularly. However, she claims
showed left ventricular hypertrophy with some to have night walking due to shortness of breath at
evidence of previous myocardial infarction, and the least twice a month which resolves after
ejection fraction was slightly low at 40%. However, nebulization. She did not think this was significant
he claims that he is asymptomatic and that he enough so she continued using her inhaler at the
regularly plays lawn tennis 3x a week. This can be usual dose. What is the level of asthma symptom
classified as what stage of heart failure? control for this patient?
A. Stage A A. Well- controlled
B. Stage B B. Partly controlled
C. Stage C C. Uncontrolled
D. None of the above D. Symptom is probably not asthma related

BK23 | TABLE OF CONTENTS


2 out of 4 (PARTLY CONTROLLED) B. Frequent pulmonary exacerbations
PLS DOUBLE CHECK SA GINA
C. Chronic bronchitis
D. Dyspnea when climbing stairs

48. In patients with asthma, the following statement is


true:
A. LABAs can be used alone in those with mild
asthma
43. The most common adverse effect of ACE inhibitors: B. LABAs should never be used alone because
A. Cough they do not treat the underlying chronic
B. Angioedema inflammation
C. Renal failure C. The main feature is the presence of a persistent
D. Hyperkalemia airway obstruction in Spirometry with
bronchodilator study.
D. Large volume space devices between a metered
dose inhaler and the patient can increase the
velocity of particles entering the upper airways
GG 13th Ed. Chapter 39 page 723 reducing the amount of drug that impinges in the
oropharynx
44. The physiologic hallmark of asthma is: Rationale for A and B:
A. Chronic airway inflammation GG 13th ed, Chapter 40, page 732: In patients with asthma, LABAs
B. Airway hyper responsiveness should never be used alone because they do not treat the
underlying chronic inflammation, and this may increase the risk
C. Narrowed airways of life-threatening and fatal asthma exacerbations; rather, LABAs
D. Plasma exudation should always be used in combination with an ICS in a fixed-
dose combination inhaler.
Rationale for C:

GG 13th Ed. Chapter 40 page 727

45. Which of the following is/are side effects of ICS?


A. PTB
B. Oropharyngeal candidiasis
C. Thyroid deficiency
D. All of the above
Rationale for D:
GG 13th ed, Chapter 40, page 730: Large-volume spacer devices
between the pMDI and the patient reduce the velocity of particles
entering the upper airways and the size of the particles by
allowing evaporation of liquid propellant. This reduces the
amount of drug that impinges on the oropharynx and increases
the proportion of drug inhaled into the lower airways.

49. The following ICS/LABA combination can be used as


GG 13th ed, p. 739 both maintenance and reliever in asthmatic patients:
46. This is a once-daily long-acting beta agonist (LABA): A. Beclomethasone/ Formoterol
A. Indacaterol B. Beclometasone/ Salmeterol
B. Glycopyrronium C. Budesonide/ Salmeterol
C. Formoterol D. A and C
D. Tiotropium Formoterol has a more rapid onset of action and is almost a full
GG pg 732: agonist, salmeterol is a partial agonist with slower onset of action
pg. 732 GG

50. Causes of diuretic resistance in heart failure include:


47. Which of the following is not an indication to start A. Excess dietary Na intake
Roflumilast (a Phosphodiesterase inhibitor) in B. Concomitant NSAID use
COPD? C. Decreased renal perfusion
D. All of the above
A. FEV1 <50%

BK23 | TABLE OF CONTENTS


GG 13th Ed Page 536

BK23 | TABLE OF CONTENTS


Pharmacology 2_2
1. What will be your criteria for discharging
the patient with pneumonia?
A. Systolic BP of < 90 mmHg
B. Patient is nauseated 4. Which of the following anti TB drugs has
C. Pulse rate of 110 bpm the highest oral bioavailability rate?
D. Blood oxygen saturation of 92% A. Ethambutol
CAP Guidelines, p. 20
B. Isoniazid
C. Pyrazinamide
D. Rifampin
taken from GG

5. Mr. Aris Tutol is your patient whom you


treated for PTB with regimen 1 (2HRZE/4HR). He
followed up at your clinic 4 months after he started
his treatment. He stated that he was unable to take
his TB medications for 40 days already as he was
transferred to another city for work. He has just been
transferred back. He asked what to do about his TB
2. True for sputum follow-up examinations treatment. What should you do?
in patients with new or retreatment, bacteriologically- A. Continue treatment and just prolong to
confirmed PTB: compensate for 40 missed doses
A. Xpert MTB/RIF assay on the second B. Label him as lost to followup and begin a
month of treatment new treatment regimen
B. Sputum microscopy at the end of C. Request for sputum microscopy
intensive phase (2nd month), end of 5th D. Request for Xpert MTB/RIF assay
month, and end of treatment (6th month) Ask for SM first before deciding.
C. If sputum microscopy is negative at the From Dra. Elumba slide
end of the intensive phase, there is no need to
get sputum microscopy at the end of the 5th and
6th months
D. There is no need for sputum follow-up
examinations if patients show clinical
improvement

6. Umi Ga is a 40-year-old construction


worker who came in with complaints of hemoptysis
and coughing for 1 month. His Xpert MTB/RIF result
is: MTB detected, RIF resistance detected. He has
no previous history of TB treatment and claims to
have no contact to persons being treated for
tuberculosis. What should you do next?
A. Repeat the Xpert MTB/RIF assay and
treat according to the second result
3. What is the regimen of choice for patients
B. Treat immediately with drug-sensitive TB
with drug-sensitive TB of the bones?
regimen as he has no risk for drug-resistant TB
A. Regimen 1 - 2 HRZE, 4 HR
C. Refer for treatment with drug-resistance
B. Regimen 2 - 2 HRZE, 10 HR
TB regimen
C. Regimen 3 – standard short all oral
D. Do sputum microscopy of 2 samples
regimen
D. Levofloxacin for 10 months

BK23 | TABLE OF CONTENTS


9. The following statement/s is/are true
7. The following patients should be regarding sputum microscopy:
screened for active pulmonary TB, except: A. The result is considered negative if only
A. An HIV patient who presents with 3 days one of the 2 specimen is positive
fever and cough B. A positive result is when at least one
B. ≥15-year-old patient who presents with ≥2 sputum smear is positive for AFB (+n, 1+, 2+,
weeks cough 3+)
C. A call center agent who presented with C. It is no longer considered as the primary
apical opacification on chest xray but does not diagnostic test for active TB screening
report any symptoms D. B and C
D. There is no exception
Reference : Doc Elumba lecture on TB, TB manual

10. Bacteriologically-confirmed extensively


drug-resistant TB (BC XDR-TB) is defined as:
A. Positive for MTB using rapid diagnostic
test with resistance to rifampicin
B. Positive for MTB complex with resistance
to at least both isoniazid and rifampicin
C. Positive for MTB with resistance to
any fluoroquinolone and at least one second-
line injectable drug in addition to isoniazid
and rifampicin
D. Resistance to more than one first-line
anti-TB drug other than both isoniazid and
rifampicin

8. Metoclopramide takes it effect by which


of the following mechanism of action?
A. Inhibits NK1 receptor
B. Stimulates 5-HT3 receptor
C. Inhibits D2 receptor
D. Stimulates 5-HT4 receptor 11. A 38-year old gastroparetic teacher is
(GG, PAGE 923) also diagnosed with GERD and you advised
Bethanecol for therapy. You will achieve its
therapeutic effects by this which mode of action?
A. Inhibits M3 receptors in the GI tract
B. Stimulates M3 receptors in the GI tract

BK23 | TABLE OF CONTENTS


C. Inhibits acetylcholine esterase in the GI
tract
D. Stimulates acetylcholine esterase in the
GI tract
bethanecol exerts its effects via directl stimulating the muscarinic
receptors (M1-M5) of the PNS; Chapter 9 page 150 GG

12. A 45 year old clerk sought consult for


fever and cough of 1 week duration. No antibiotics
were taken prior to consult. She is hypertensive and
diabetic. She is on losartan 50mg once a day and on
Metformin/Vildagliptin 500mg/50mg one tablet 2x a
day. Her usual BP is 130/80-120/70 mmHg with
medication. Her last FBS was 110mg/dl. She is
awake, responsive but weak, tachypneic and febrile
at 39C with a respiratory rate of 30/minute. BP is 14. JJ is a 45 yo male patient with chronic
100/60mmHg. CXR shows pulmonary infiltrates at kidney disease. He was recently diagnosed with
both lower lung fields with blunting of the right PTB and was given a separate dose regimen of
costophrenic angle. You are considering pneumonia. pyrazinamide. What is the ideal dosing frequency of
What is the risk stratification of this patient? pyrazinamide for this patient with low estimated
A. Low risk CAP without co-morbid illness glomerular filtration rate (egfr)?
B. Low risk CAP with stable co-morbid A. Weekly
illness B. Every other day
C. Moderate risk CAP C. Every 3 weeks
D. High risk CAP D. Monthly
“Pyrazinamide clearance is reduced in renal failure; therefore, the
a or b?
dosing frequency is reduced to three times a week at low
glomerular filtration rates.” GG, chap 60, page 1071

15. Aminoglycosides are most potent to:


A. M. kansasii
B. M. avium
C. M. tuberculosis
D. M. abscessus

16. Combination therapy with phenothiazines


of which of the following drugs that provides
13. The mechanism of non-inflammatory type synergistic antiemetic action and appears to
of diarrhea is: attenuate the adverse effects of both agents?
A. Direct invasion of bacteria into the A. Cannabinoids
intestine B. Corticosteroids
B. Destruction of villus cells by viruses C. Macrolides
C. Production of cytotoxins D. Phenothiazines
D. B and C

BK23 | TABLE OF CONTENTS


19. A patient with bacteriologically-confirmed
TB at the beginning of treatment and who was
smear- or culture-negative in the last month of
treatment and on at least one previous occasion in
the continuation phase:
A. Treatment completed
B. Cured
C. Lost to follow up
D. Treatment failed

17. According to the CPG guidelines in the


treatment of acute infectious diarrhea, which of the
following statements is WRONG?
A. Routine stool exam is indicated in
parasitism
B. Stool culture can be collected as soon as
diarrheal symptoms occur
20. Which of the following is NOT an
C. Procalcitonin test distinguishes
indicator for early deescalation of the patient’s
bacterial and nonbacterial pathogen for
antibiotic treated for CAP?
diarrhea
A. Afebrile for 48 hours
D. Rapid diagnostic tests may be used
B. RR of 20 cpm
during suspected outbreaks of cholera and
C. Etiologic agent is Methicillin resistant S.
shigella, but confirmation with stool culture is still
aureus
recommended
D. Serum creatinine of 1 mg/dL
There is insufficient evidence to support the use of biomarkers (CRP,
calprotectin, ESR, procalcitonin, total serum WBC) in
distinguishing the cause of acute infectious diarrhea. (CPG
pocket edition page 4)

18. The primary diagnostic test for pulmonary


and extra pulmonary TB in adults and children
(according to the National Tuberculosis Program –
Manual of Procedures 2020) is:
A. Direct sputum smear microscopy of 3
specimen
B. Rapid diagnostic test such as Xpert
MTB/RIF assay
C. Direct sputum smear microscopy of 2
specimen
21. Palonosetron has potent antiemetic
D. Chest x-ray posteroanterior view
effects by blockade of what receptors?
A. 5-HT2 receptors
B. 5-HT3 receptors
C. 5-HT4 receptors
D. 5-HT5 receptors
ratio: GG p.936

BK23 | TABLE OF CONTENTS


22. J. Kim, a 21-year-old female sought
consult for pulmonary clearance for work. She has a
right upper lobe reticular lesion associated with
coughing for 1 month. She claims to have been
started on TB medications a year ago when she had
the same chest xray finding but discontinued the 24. A patient treated for pneumonia with
medications after she was cleared for work in 2 antibiotics became afebrile on the second day of
weeks. Her sputum Xpert MTB/RIF assay detected treatment but she complains that cough has not yet
MTB with no rifampicin resistance. How should you resolved. How will you address her complain?
classify the patient? A. Inform her that she is steadily
A. Pulmonary tuberculosis, retreatment, improving and her cough will subside within
bacteriologically-confirmed 2-4 weeks
B. Pulmonary tuberculosis, new case, B. You order for a sputum gram stain and
bacteriologically-confirmed repeat her CBC
C. Pulmonary tuberculosis, retreatment, C. You shift the antibiotic
clinically-diagnosed D. You advise her to be admitted
D. Pulmonary tuberculosis, new case,
clinically-diagnosed
ratio: Drug-resistant PPT

25. When should the patient feel back to


normal after a moderate risk pneumonia infection?
A. After 1 week of completing the antibiotics
B. 6 weeks
23. What antibiotic regimen will you start to a C. 3 months
patient with moderate risk CAP? D. 6 months
A. Clarithromycin 500mg BID PO
B. Cefuroxime 500mg BID PO +
Azithromycin 500mg OD PO
C. Ceftriaxone 2gms OD IV +
Azithromycin 500mg OD PO
D. Meropenem 1gm IV + Levofloxacin
500mg IV

26. What prokinetic drug that has a potent


anti-nausea and antiemetic action without affecting
the small intestine and colonic motility?
A. Dompiredone
B. Ondansetron

BK23 | TABLE OF CONTENTS


C. Meclizine 30. Which of the following is TRUE of
D. Omeprazole racecadotril?
(GG 13th edition, page 924) A. An inhibitor of enkephalinase that crosses
It has modest prokinetic activity. Although it does not readily cross the
BBB to cause extrapyramidal side effects, domperidone exerts
the blood brain barrier
effects in the parts of the CNS that lack this barrier, such as B. Anti-secretory and anti-motility
those regulating emesis, temperature, and prolactin release. C. Can be given up to 14 days
Domperidone does not appear to have a significant effects on
lower GI motility. D. Can cause constipation after
prolonged intake
27. Ms. Jee Yu is a primary school teacher A= does not cross
B = only anti-secretory (?)
who sought consult for 3 months intermittent cough C= up to 7 days maximum
with associated weight loss. Vital signs at the clinic: D = less constipation than loperamide but not sure if no constipation,
BP 110/70, HR 75, Temp: 36.8, RR 20. Weight: so D.

45kg. On follow-up, Xpert MTB/RIF result detected


31. Which of the following is TRUE of
MTB but not rifampicin resistance. Chest x-ray
rifampin?
showed bilateral upper lobe infiltrates. What
A. Safe to use in 3rd trimester of pregnancy
medications should be started?
B. Can be used as monotherapy against
A. Isoniazid, Rifampicin, Pyrazinamide,
staphyloccocal endocarditis
Ethambutol (HRZE) 3 tablets once a day for 2
C. Can be used as prophylaxis for
months and shift to HR 3 tablets once a day
meningococcal disease
for 4 months
D. B and C
B. Isoniazid, Rifampicin, Pyrazinamide, Rifampin pregnancy category C, not recommended for use in late
Ethambutol (HRZE) 4 tablets once a day for 3 pregnancy
months and shift to HR 3 tablets once a day for
another 3 months
C. Isoniazid, Rifampicin (HR) 3 tablets once
a day for 6 months
D. Isoniazid, Rifampicin, Pyrazinamide,
Ethambutol (HRZE) 4 tablets once a day for 2
months and shift to HR 2 tablets once a day for 32. Which of the following rifamycins has the
4 months longest half-life?
Since the patient is not rifampin resistant, you go for 2HRZE/4HR
regimen. 3 tablets are used because the patient weighs 45kg. A. Rifabutin
B. Rifampin
See tables 6 and 7 page 35 of 2020 MOP TB control
C. Rifapentine
D. None of the above
GG Table 60-3
28. John went back to your clinic for ff up. He Rifabutin = 32-67 hrs
was on his 3rd month of treatment with fixed dose Rifampin = 2-5 hrs
Rifapentine = 14-18 hrs
combination drug for PTB. He is complaining of
33. Which of the following beta lactam
on/off arthralgia on multiple joints relieved with
antibiotics was shown to be useful for MDR TB
celecoxib. Lab results showed uric acid level of 10
treatment with a sputum conversion rate of more
mg/dl. Which of the following anti TB drugs could
than 60%?
have caused his problem?
A. Coamoxiclav
A. Pyrazinamide
B. Faropenem
B. Rifampin
C. Carbapenem + cephalosporin
C. Isoniazid
D. Ertapenem
D. All of the above
P.39 table 12 of 2020 MOP TB control

29. Jaundice due to hepatitis in patients


taking a fixed-dose combination drug therapy for TB
can be due to:
A. Pyrazinamide
B. Rifampicin
C. Isoniazid
D. All of the above
P.39 table 12 of 2020 MOP TB control GG p 1078

BK23 | TABLE OF CONTENTS


34. The following statement is true regarding
specimen collection for TB testing:
A. Sputum volume of at least 5ml is needed
for Xpert MTB/RIF assay
B. Sputum induction is done to make
sputum thinner and easier to collect for those
with productive cough
C. For sputum smear microscopy, 1 ml
sputum is acceptable GG page 935
D. For those with productive cough, you
can instruct patient to expectorate one 37. Maria 30 y.o was seen at the ER due to
sputum sample on the spot, with proper watery stools and abdominal discomfort noted few
precautions hours prior to consult. She claimed she was taking
Loperamide as advised by her neighbour. What is
the maximum dose of loperamide per day?
A. 2 mg
B. 4 mg
C. 16 mg
D. 24 mg
National TB Control Program Manual of Procedures 6th ed p 20

35. A patient who has both pulmonary and


extrapulmonary tuberculosis is classified as:
A. Pulmonary tuberculosis
B. Extrapulmonary tuberculosis
C. Drug-resistant tuberculosis
D. Extensive tuberculosis
GG page 931

38. A 3 month old infant, breastfeeding, was


brought to your clinic because of one day history of
diarrhoea for 3 episodes. The child is alert and
playful and there were no changes in milk intake.
You advised the mother to give her child oral
rehydration solution. How much fluid of ORS should
the mother give to her child?
A. 100ml per loose stool
National TB Control Program Manual of Procedures 6th ed p 18 B. 200ml in 4 hours
C. 500ml in 4 hours
D. 700ml in 4 hour
36. A 60-year old patient with cardiac failure
is diagnosed with ovarian cancer. She begins using
Cisplatin but becomes nauseated and suffers from
severe vomiting. Which of the following medications
would be most effective to counteract the emesis in
this patient without exacerbating her cardiac
CPG Diarrhea page 56
problem? 39. Which of the following fluoroquinolones is
A. Domperidone considered central for the treatment of MDR TB?
B. Dolasetron A. Ciprofloxacin
C. Prochlorperazine B. Gatifloxacin
D. Palonosetron C. Levofloxacin
D. Moxifloxacin
GG 13th Ed. page 1076 (2nd column)

40. Acute infectious diarrhea is suspected if a


patient presents with the passage of 3 or more

BK23 | TABLE OF CONTENTS


loose, watery or bloody stools within 24 hours that 44. In patients taking a fixed-dose
may be accompanied by combination drug therapy for TB, the orange color in
A. Nausea urine is caused by:
B. Vomiting A. Ethambutol
C. Abdominal pain B. Pyrazinamide
D. All are correct C. Rifampicin
CPG Diarrhea D. Isoniazid
R- red orange urine
I-impaired nerves(peripheral neuropathy)
P-purine (hyperuricemia,gout)
E-eye damage (optic neuritis)

45. Based on the organism likely causing the


pneumonia, how long will you give your antibiotics if
sputum culture growth will show Pseudomonas
41. What is/are the clinical parameters that aeruginosa and negative blood culture growth?
would indicate presence of dehydration in children A. 5-7 days
with acute infectious diarrhea? B. 10-14 days
A. Normal thirst C. 14-21 days
B. Presence of tears D. 28 days
C. Abnormal respiratory pattern
D. Edema
CPG Diarrhea

42. Optic neuritis is considered a remarkable


side effect of this anti TB drug:
A. Isoniazid
B. Rifampin
C. Pyrazinamide
D. Ethambutol
R- red orange urine
I-impaired nerves(peripheral neuropathy)
P-purine (hyperuricemia,gout)
E-eye damage (optic neuritis)

43. A patient is receiving treatment with


Lorazepam prior to chemotherapy to help reduce her 46. What is the LEAST LIKELY etiologic
anticipatory nausea and vomiting. Which of the organism in moderate risk CAP?
following should generally be avoided in this patient? A. Legionella pneumophila
A. Ethanol B. Moraxella catarrhalis
B. Ondansetron C. Pseudomonas aeruginosa
C. Polyethylene glycol D. Streptococcus pneumoniae
D. Castor oil
Lorazepam is a CNS depressant. Alcohol is also a CNS depressant.

BK23 | TABLE OF CONTENTS


49. A 25 year old medical student sought
consult due to fever and cough of 5 days duration.
No antibiotics were taken. She is allergic to
crustaceans NSAIDS and Pencillin. BP is
47. The risks for drug-resistance tuberculosis 100/60mmHg, HR 105 bpm, RR 25 cpm, temp
(DR-TB) include the following, except: 38.3oC. Crackles heard over her Right lower lung
A. Previous history of TB treatment field. What antibiotic will you prescribe?
B. Close contacts of a non-converter of A. Amoxicillin 1gm TID PO
drug-sensitive TB regimen B. Azithromycin 500mg OD PO
C. Extensively symptomatic patients C. Co-Amoxiclav 1 gm BID PO
D. Close contacts of a known DR-TB case D. Cefuroxime 500mg BID PO
Azithromycin is a macrolide; other drugs are related to penicillin; (not
sure though)

48. The clinical triad of isioniazid overdose


includes:
A. Decrease consciousness but easily
arousable
B. Refractory seizure treated with
benzodiazepines
C. Seizures treated with high dose 50. How long will you administer
phenytoin Azithromycin used for treatment of low-risk
D. Metabolic acidosis treatable with sodium pneumonia?
bicarbonate A. 5-7 days
E. C and D B. 10-14 days
Goodman and Gillman 13th Ed page 1073-1074 C. 14-21 days
D. 28 days

CAP Guidelines 2016 Update

BK23 | TABLE OF CONTENTS


Pharmacology 2nd BME
3. Which of the following drugs blocks the binding of
1. A 75-year old man with chronic obstructive pulmonary IgE to high affinity IgE receptors?
disease is diagnosed with suspected influenza A. Montelukast
based on his complaints of flu-like symptoms that B. Cetirizine
began 24 hours ago. Which of the following agents C. Mepolizumab
D. Omalizumab
is most appropriate to initiate for the treatment of
influenza? Rationale:
A. Ribavirin
B. Oseltamivir
C. Zanamivir
D. Amantadine
Rationale:
4. Serious adverse side effect of this antiviral agent is
associated with alteration of dopamine
neurotransmission
A. Ribavirin
B. Oseltamivir
C. Zanamivir
D. Amantadine
Rationale: not sure but it’s the only one i found with CNS adverse
effects

Zanamivir - not generally recommended for tx of patients with


underlying airway disease
Amantadine - active only for influenza A

2. FV, a 37-year-old teacher, came to your clinic for


followup. He is a known asthmatic maintaining daily
low-dose ICS-formoterol for the past 2 years. In the
past 3-4 months, he started having cough and chest
tightness at least 2x a week relieved by using his
ICS-formoterol as reliever. He also had an episode
of suddenly waking up early in the morning feeling
short of breath associated with wheezing a few
days ago which was also relieved by his inhaler.
Currently, at the clinic, he has no symptoms. What
will you do with his maintenance medication? 5. Metoclopramide is notorious to which of the following
A. Continue his low-dose maintenance ICS- adverse drug effects?
formoterol but let him rest at home for 1 A. Cardiac arrythmia
week B. Parkinsonian symptoms
B. Stop his maintenance ICS-formoterol and C. Diarrhea
give oral steroid D. Atopic reactions
C. Increase to medium-dose maintenance Rationale:
ICS-formoterol and workup for infection
D. Prescribe as needed salbutamol +
ipatropium nebulization on top of his
maintenance ICS-formoterol
Rationale:

BK23 | TABLE OF CONTENTS


6. Which of the following best describes prednisone in 10. Juan, 65 yo was on his 2nd month of TB treatment
the treatment of asthma? using combination drug. He has early dementia with
A. 1 week of short course therapy may not need minimal supervision from family. He was brought to
tapering of dose the ER due to sudden onset of dizziness and
B. Maximal benefit is usually achieved at 60-80 mg per slurred speech. His son noticed he's been taking his
day medication 3x a day. What is the next best thing to
C. Indicated in severe asthma exacerbation do?
D. All of the above A. Give high dose of sodium bicarbonate right
away
Rationale: page 739 GG 13th Ed B. Give IV pyridoxine and benzodiazepine
C. Get CT of brain to rule out other causes
D. Intubate the patient
Rationale:

7. An effective antiviral agent should be


A. Virucidal
B. Virustatic
C. Affects latent viruses
D. Is active outside the host cell
Rationale: Katzung p. 863 14th ed

8. The following feature increases the probability of


asthma:
A. Chronic sputum production
B. Isolated cough
C. Chest pain
D. Worse at night or early in the morning
Rationale:

9. Treatment of persistent diarrhea include the following


EXCEPT:
A. Multivitamins and minerals supplementation
B. Routine antibiotics
C. Nutritious diet
D. Give frequent small meals

BK23 | TABLE OF CONTENTS


11. Which of the following statements is CORRECT
regarding the seasonal prophylaxis with
Amantadine/Rimantadine?
A. Either of which can be used as post exposure
prophylaxis in contacts of untreated patients
B. Should be given after 2 days of symptom or rash
onset
C. Given 200mg/day for 7-10 days
D. Prophylaxis will give you 70-90% protection
against influenza B illness
Rationale: GG pg 1113

12. A patient is receiving chemotherapy treatment with


oral Aprepitant. Which of the following should the
patient refrain from taking in?
A. Ciprofloxacin
B. Ondansetron
C. Hyoscine
D. Paracetamol
Rationale: not sure, GG chapter 50

13. Homemade ORS solution can be made by adding:


A. 10grams/L common sugar and 2 grams/L table
salt
B. 8 grams/L common sugar and 8 grams/L table
salt
C. 13 grams/L common sugar and 18 grams/L table
salt
D. 18 grams/L common sugar and 3 grams/L
table salt

BK23 | TABLE OF CONTENTS


Rationale: Anti-diarrheal slides
16. Which of the following antiretroviral drugs has no
activity against HIV2?
A. Lamivudine
B. Nevirapine
C. Tenofovir
D. Zidovudine
Rationale: NNRTIs are HIV-1 specific and have no activity against
HIV-2.

14. Which of the following PMDI doses needs a spacer


to minimize side effects?
A. 100 micrograms per day 17. The following are risk factors for drug-resistance
B. 400 micrograms per day tuberculosis, except:
C. 800 micrograms per day A. More than 6 months history of cough
D. 2000 micrograms per day associated with hemoptysis
Rationale: GG pg 739 B. Previous history of TB treatment
C. Close contacts of non-converter of DS-TB
regimen
D. Close contact of a known DR-TB case
Rationale:

18. A 20-year old woman with leukemia was undergoing


chemotherapy with the intravenous antineoplastic
drugs. During the treatment, she developed a
systemic infection from an opportunistic pathogen.
There was no erythema or edema at the catheter
insertion site. A white vaginal discharge was
observed. After appropriate specimens were
15. Which drug is appropriate for oral use in the obtained for culture, empiric antibiotic therapy was
treatment of vaginal candidiasis? started with gentamicin, nafcillin and ticarcillin
A. Clotrimazole intravenously. This regimen was continued for 3
B. Griseofulvin days, during which the patient’s condition did not
C. Fluconazole improve significantly. Her throat was sore and white
D. Nystatin plaques appeared in her pharynx. On day 4, none
Rationale: Antifungal slides of the cultures had shown any bacterial growth, but
both the blood and urine cultures grew out Candida
albicans. The best course of action is to?
A. Continue antibiotics
B. Stop current antibiotics and start
amphotericin B
C. Start IV ketoconazole
D. Continue antibiotics and start flucytosine

BK23 | TABLE OF CONTENTS


Rationale:

23. Interactions between this drug and cell membrane


components can result in the formation of pores
lined by hydrophilic groups present in the drug
molecule. Identify the drug
A. Caspofungin
B. Nystatin
19. Which of the following statements is TRUE of inhaled
C. Griseofulvin
corticosteroids?
D. Terbinafine
A. Asthma patients on ICS has increased risk of Rationale: According to Katzung’s, the answer is Amphotericin B.
lung infections like TB However, nystatin is a polyene macrolide like Amphotericin
B. Oropharyngeal candidiasis occurs in 5% of B.
patients
C. Bruising is a local side effect
D. All of the above
Rationale:

20. How long antiretroviral agents should be given?


A. 1 week
B. 1 month
C. 1 year
D. 1 lifetime
Rationale: Sa ARV, may forever!

21. The following metabolic abnormalities are common


with diuretic use EXCEPT:
A. Hypernatremia
B. Hypokalemia
C. Metabolic alkalosis
D. Hypomagnesemia
Rationale: Hyponatremia is a very common side effect of diuretic use.
Patients on diuretics need to be monitored for possible
hyponatremia, hypokalemia, and hypomagnesemia.

22. An immunocompromised patient suffering from HIV


is diagnosed with sub-acute invasive pulmonary
aspergillosis caused by Aspergillus fumigatus.
Which drug would be the best choice for
monotherapy?
A. Voriconazole
B. Amphotericin B
C. Terbinafine
D. Fluconazole
Rationale:

24. Which of the following statements about ARV drugs


is correct?
A. Emtricitabine the most toxic NRTI agent

BK23 | TABLE OF CONTENTS


B. Rilpivirine is less susceptible to high-level 26. Which is a characteristic of formoterol?
drug resistance A. Rapid onset
C. Drug interaction is not a consideration when B. Partial beta agonist
using Etravirine C. Both
D. Nevirapine is associated with the most CNS side D. Neither
effects Rationale: GG 13th Edition

Rationale:
Emtricitabine is one of the least-toxic antiretroviral drugs and has few
significant adverse effects (Cihlar and Ray, 2010).
Etravirine should not be administered with tipranavir/ritonavir,
fosamprenavir/ritonavir, or atazanavir/ritonavir in the absence of
better data to guide dosing. Etravirine should not be combined
with other NNRTIs. Unlike other NNRTIs, etravirine does not
appear to alter the clearance of methadone.
Main side effects of nevirapine is mostly dermatologic.

27. A 50 year old man who is hypertensive & diabetic


was admitted for 5 day history of fever, productive
cough & tachypnea. He is awake, responsive but
weak. BP 90/60 mmHg, HR 125 bpm, RR 30 cpm,
Temp 38.5C. CXR revealed lobar pneumonia on
the right lung. He still has good urine output. He
had been seen at the ER OPD 2x for the past 2
months for cellulitis on his leg. He was unable to
recall the antibiotics prescribed. What is the
category of his pneumonia?
A. CAP Low Risk with stable comorbidities
B. CAP Low Risk without comorbidities
25. Mechanism of action of dapsone: C. CAP Moderate Risk
A. Competitive inhibitor of thymidylate synthase D. CAP High Risk
B. Competitive inhibitor of dihydropteroate Rationale: not sure between moderate or high risk
synthase
C. Competitive inhibitor of dihydrofolate synthase
D. Competitive inhibitor of dihydrofolate reductase
Rationale:

28. Diuretics are important for the treatment of heart


failure. Which of the following statement is TRUE
regarding the use of diuretics in heart failure?
A. It improves mortality
B. It should be used in all patients with LV
dysfunction
C. Sustained diuresis can decrease cardiac
output

BK23 | TABLE OF CONTENTS


D. Results in an increase in left ventricular filling
pressure
Rationale: GG 13th ed pg 535; not sure (please recheck)
B. should not be used in all patients, appropriate patients only (stage B
AHA GUIDELINE)
D. should be decreased in filling pressure
A. prognostic efficacy of diuretics cant be assessed since diuretics
should only be given to patients with congestion (ethically
impermissible)
29. When is the use of Maraviroc indicated?
A. CXCR4 tropic virus
B. CCR5 tropic virus
C. Dual tropism
D. Predominantly CXCR4 tropic virus
Rationale: GG 13th ed pg 1151

31. Which of the following antibiotics can be used as a


step-down oral drug from IV Ceftriaxone?
A. Cefuroxime
30. A 55 year old man came in for dyspnea and B. Cefpodoxime
respiratory distress. Physical examination showed a C. Cefixime
BP of 160/90 mmHg, HR 110/min, RR 28/min. D. A and B
There was elevated JVP, rales on both lung fields, E. B and C
S3 gallop on cardiac auscultation. Which of the Rationale: CAP GUIDELINES p. 11
following drugs is NOT the best choice in this heart
failure patient?
A. Enalapril
B. Furosemide
C. Metoprolol
D. Telmisartan
Rationale: GG 13th ed pg 534 (use beta blockers in clinically
stable patients, here patient is in respiratory distress) 32. The Beta agonist of choice for the treatment of heart
failure with reduced ejection fraction
A. Dobutamine
B. Dopamine
C. Norepinephrine
D. Epinephrine
Rationale: GG, p. 540

33. A 55-year old female presents to the hospital with


shortness of breath, fever and malaise. She has a
history of breast cancer, which was diagnosed 3
months ago and has been treated with
chemotherapy. Her chest X-ray shows pneumonia
and respiratory cultures are positive for Aspergillus
fumigatus. Which of the following is the most
appropriate choice for treatment?

BK23 | TABLE OF CONTENTS


A. Voriconazole
B. Fluconazole
C. Flucytosine
D. Ketoconazole
Rationale:

Lippincott

34. Which of the following statements correctly pair the


antifungal drugs with their likely mechanisms of
action?
A. Flucytosine-ergosterol synthesis
B. Echinocandins-inhibition of squalene epoxidase
C. Azoles-blocks synthesis of ergosterol to
lanosterol (lanosterol → ergosterol)
D. Griseofulvin-prevents mitosis

Rationale:

36. Which of the following statements describes the


mechanism of action of Acyclovir?
A. Acyclovir diphosphate competes with deoxyGTP
as a substrate for viral DNA polymerase
B. Acyclovir triphosphate competes with
deoxyGTP as a substrate for viral DNA
polymerase
C. Acyclovir triphosphate competes with deoxyGDP
as a substrate for viral DNA polymerase
D. Acyclovir triphosphate competes with deoxyGTP
as a substrate for viral RNA polymerase
Rationale:

35. A 38-year-old man is being treated with


voriconazole or invasive aspergillosis. His dose
should be adjusted if it is determined that he has 37. Which of the following is an ICS LABA combination?
this disease: A. Fluticasone + albuterol
A. Renal failure (cyclodextrin component of IV B. Budesonide + formoterol
voriconazile is nephrotoxic) C. Ipratropium + salbutamol
B. Hepatic cirrhosis D. Glycopyyronium + mometasone
C. Rheumatoid arthritis Rationale:
D. Type 2 diabetes mellitus
Rationale: GG, p. 1095

BK23 | TABLE OF CONTENTS


C. Clofazimine
D. Bedaquiline
Rationale:

42. A 35 year old diabetic had a history of influenza 2


weeks ago. She sought consult because of the
38. A fisherman uses a transdermal scopolamine patch recurrence of fever and cough. A repeat CXR
to assist with the nausea associated with being on showed patchy infiltrates on both lung fields. Based
the boat. What is the most likely side effect he will on the most likely etiologic agent causing the
experience? illness, what antibiotic should be included in the
A. Acute dystonic reaction treatment of the pneumonia?
B. Euphoria A. Levofloxacin
C. Sedation B. Linezolid
D. Tremor C. Meropenem
Rationale: D. Piperacillin-Tazobactam
Rationale:
The patient is showing signs of moderate risk CAP

39. A 33-year-old, HIV positive homosexual man living


near the Mississippi River, was admitted to the
hospital with a 3-week history of cough, productive
yellow sputum, progressive dyspnea of exertion,
and 14-pound weight loss. Pertinent lab results on
admission were: CD4+ lymphocyte counts 320
cells/mm, lactate dehydrogenase 5000 IU/L (normal
< 600). A bloodstain showed neutrophils containing
small, uninucleated, and encapsulated 43. The most likely drug in Regimen 1 for PTB that
microorganisms. Which of the following statements causes psychosis and convulsion:
correctly pairs the most likely disease of the patient A. Isoniazid
with the appropriate treatment? B. Rifampicin
A. Progressive disseminated histoplasmosis- C. Pyrazinamide
ampho B D. Ethambutol
B. Progressive disseminated coccidioidomycosis- Rationale:
fluconazole
C. Pulmonary blastomycosis-fluconazole
D. Meningeal cryptococcocosis-ampho B

40. Which of the following agents has no antiemetic


properties?
A. Corticosteroids
B. Phenothiazines
C. Scopolamine
D. Erythromycin
Rationale: All have antiemetic properties except Erythromycin. 44. Which of the following statements on Enfuvirtide is
NOT CORRECT?
41. RR 25 yo went back to your clinic for follow up. She A. It is not active against HIV 2
was on treatment for Mycobacterium infection. You B. It is administered subcutaneously
noticed reddish black discoloration of her skin. C. It is eliminated by the kidneys
Which of the following drugs could have brought D. It is associated with injection site reaction
that side effect? Rationale:
A. Rifampin Acc. to GG page 1152 under Enfuvirtide, “The major route of
B. Pyrazinamide elimination for enfuvirtide has not been determined.”

BK23 | TABLE OF CONTENTS


45. The following are causes of diuretic resistance in 48. High levels of Acyclovir obtained in the target viruses
heart failure EXCEPT: such as herpes simplex virus is a result of which
A. Noncompliance with medications properties?
B. Increased sodium intake A. Binding to the major immediate early region 2
C. Concomitant NSAID use (IE2) of CMV mRNA
D. Increased renal perfusion and glomerular B. Direct inhibition of viral DNA polymerase and
filtration rate RNA polymerase
Rationale: C. Host cell enzyme conversion to triphosphate
compounds
D. Monophosphorylation by the thymidine kinase
Rationale:

46. A 31-year-old man presented to his physician with


an annular lesion with raised borders in his left 49. Which of the following antiretroviral agents is used in
hand. Microscopic examination of skin scrapings the treatment of hepatitis B infection?
revealed branching hyphae. The diagnosis of tinea A. Abacavir
manus was made and the patient has prescribed B. Didanosine
appropriate oral treatment. Which of the following C. Tenofovir
skin structures was most likely the site of action of D. Zidovudine
the prescribed drug?
A. Stratum basale
B. Stratum corneum
C. Stratum spinosum
D. Stratum germinativum
Rationale: Chapter 61 of goodman and gilman, antifungal agent
MOA is on the stratum corneum, recheck

47. A 65-year-old patient presents with a chief complaint


of discolored toenails that have recently begun to
cause mild pain and discomfort. Histological
analysis of an under-nail swab is positive for
Trichophyton rubrum. What oral medication would
be the best choice to treat this infection?
A. Amphotericin B
B. Terbinafine
C. Caspofungin
D. Nystatin
Rationale:

50. Which of the following is NOT a criterion for


admission in a child with acute infectious diarrhea?
A. Lethargic
B. Suspected typhoid ileitis
C. Distended abdomen
D. Recently immunized with Rotavirus
vaccine
Rationale:

BK23 | TABLE OF CONTENTS


arthralgias, anorexia, nausea and vomiting, dysuria, malaise,
and fever.

(GG, 13th ed p.1072)

53. A 50 year old man who is hypertensive & diabetic


was admitted for 5 day history of fever, productive
cough and tachypnea. He is awake, responsive but
weak. BP 90/60 mmHg, HR 125 bpm, RR 30 cpm,
Temp 38.5C. CXR revealed lobar pneumonia on
the right lung. He still has good urine output. He
had been seen at the ER OPD 2x for the past 2
months for cellulitis on his leg. He was unable to
recall the antibiotics prescribed. Based on the
category of his pneumonia, what antibiotic will you
give to treat his pneumonia?
A. Ceftriaxone + Azithromycin
51. A 57-year-old woman who has been on chronic B. Ertapenem + Azithromycin
therapy with corticosteroids for arteritis presents C. Cefepime + Azithromycin
with a high fever, severe headache and neck D. Ampicillin Sulbactam
Rationale: I think this is low-risk CAP but the choices are moderate-
stiffness. She also admits to feeling confused and risk and high-risk CAP
irritable, and having felt nauseous over the past 24
hours. A lumbar puncture is used to obtain a
54. JH is a 21 year-old-woman who comes to your
sample of CSF, which reveals the presence of
Ob/Gyn clinic with a chief complaint of vaginal
Cryptococcus neoformans in culture. The patient is
soreness and a burning discomfort during urination.
admitted to hospital and you start an induction
A pelvic exam reveals signs of inflammation and
antifungal protocol with i.v. amphotericin B plus
discharge. A quick microscopic exam of the
flucytosine for 2 weeks. Her condition slowly
discharge indicates the presence of a yeast
improves and you decide to switch her to an oral
infection. You review the list of JH's current
maintenance therapy which she will likely need to
medications, which include an
continue for up to a year after discharge from the
estrogen/progesterone oral contraceptive, and a
hospital. Which agent would be the best choice for
daily multivitamin. You then administer an oral dose
maintenance therapy?
of 150 mg fluconazole to treat her infection. For
A. Fluconazole
what reason did you review her current list of
B. Itraconazole
medications?
C. Griseofulvin
A. Fluconazole's bioavailability is low, and
D. Micafungin
Rationale: may be affected by other drugs
B. Fluconazole can induce CYP-450 and
Fluconazole 400 mg daily, has been recommended as continuation increase the hepatic clearance of other
therapy in patients without AIDS with cryptococcal meningitis drugs
who have responded to an initial course of C-AMB or L-AMB and
for patients with pulmonary cryptococcosis. C. Fluconazole is CYP-450 inhibitor, and
Itraconazole is the drug of choice for patients with indolent, can decrease the metabolism of other
nonmeningeal infections due to B. dermatitidis, H. capsulatum, drugs
P. brasiliensis, and Coccidioides immitis. D. Fluconazole enhances the renal
Griseofulvin is fungistatic in vitro for various species of the
dermatophytes Microsporum, Epidermophyton, and clearance of several drugs by
Trichophyton. The drug has no effect on other fungi or on inhibiting p-glycoprotein
bacteria. Rationale: not sure
Micafungin is approved for the treatment of invasive candidiasis (Fritz Bioavailability is high
et al., 2008) and esophageal candidiasis and for prophylaxis in Inhibitor of CYP3A4 and CYP2C9
hematopoietic stem cell transplant recipients. GG, 13th ed 1094-1095

G&G 13th ed page 1089 Table 61-1, page 1093, 1095, 1097, 1098
55. What antiretroviral agent is associated with “Fatal
52. The anti-TB drug that causes arthralgia due to Hypersensitivity Syndrome”?
hyperuricemia: A. Abacavir
A. Isoniazid B. Ritonavir
B. Rifampicin C. Lopinavir
C. Pyrazinamide D. Fosemprenavir
Rationale:
D. Ethambutol The most important adverse effect of abacavir is a unique and
Rationale: potentially fatal hypersensitivity syndrome characterized by
In nearly all patients, pyrazinamide inhibits excretion of urate, fever, abdominal pain, and other GI complaints; a mild
resulting in hyperuricemia, which may cause acute episodes of maculopapular rash; and malaise or fatigue. (G&G 13th ed page
gout. Other untoward effects observed with pyrazinamide include 1143)

BK23 | TABLE OF CONTENTS


A. It blocks Acetycholine receptor in the
56. The following is a true statement regarding asthma in periphery
adults: B. It blocks Dopamine receptors in the
A. Positive bronchodilator reversibility chemoreceptor trigger zone
is defined as increase FEV1 of >12% C. It blocks Glucocorticoid receptors in the
or >200ml from baseline 10-15 vomiting center
minutes after 200-400mcg of D. It blocks 5-HT2 receptor in the
salbutamol chemoreceptor trigger zone
B. A positive exercise challenge test is Rationale:
Metoclopramide and other substituted benzamides are derivatives of
defined as fall in FEV1 of >10% and
para-aminobenzoic acid and are structurally related to
>200ml from baseline procainamide. The mechanisms of action of metoclopramide are
C. A confirmed variable expiratory airflow complex and involve 5HT4 receptor agonism, vagal and central
limitation is enough for the diagnosis of 5HT3 antagonism, and possible sensitization of muscarinic
receptors on smooth muscle, in addition to dopamine receptor
asthma even in the absence of antagonism. Administration of metoclopramide results in
symptoms coordinated contractions that enhance transit. Its effects are
D. Daily diurnal peak expiratory flow confined largely to the upper digestive tract, where it increases
variability is twice daily PEF (day’s lower esophageal sphincter tone and stimulates antral and small
intestinal contractions. Metoclopramide has no clinically
highest minus day’s lowest/mean of significant effects on large-bowel motility.
day’s highest and lowest) averaged over (GG 13th ed p.923)
1 month (over 1 week)
Rationale:
Global Strategy for Asthma Management and Prevention (2020 58. Which of the following characteristics differentiates
update), page 23 budesonide from fluticasone?
A. Used as treatment for asthma and
COPD
B. The latter has greater 1st pass
metabolism
C. The former can be nebulized
D. Can be given as once daily
Rationale: (not sure)
A. Combination inhalers that contain a LABA and a
corticosteroid (e.g., fluticasone/salmeterol,
budesonide/formoterol) are now widely used in the treatment
of asthma and COPD. (GG, 13th ed p.732)
B. Budesonide and fluticasone propionate have a greater first-
pass metabolism than BDP and are therefore less likely to
produce systemic effects at high inhaled doses. (GG, 13th
ed p.738)
C. (GG, 13th ed p.745)
D. Administration once daily of some steroids (e.g.,
budesonide, mometasone, and ciclesonide in mild asthma and
fluticasone furoate in all patients) is effective when doses of 400
μg or less are needed. (GG, 13th ed p.739)

59. The mode of action of this antiviral drug is


associated with its interference to viral messenger
RNA:
A. Amantadine
B. Zidovudine
57. An 18-years old Lupus nephritis adolescent female C. Ribavirin
has nausea and vomiting after the first monthly D. Lamivudine
dose Cyclophosphamide IV pulse therapy. She was Rationale:
given Metoclopramide IV stat dose and as needed A. Amantadine and rimantadine inhibit an early step in viral
replication, probably viral uncoating; for some strains, they
basis. How does Metoclopramide works?
also have an effect on a late step in viral assembly, probably
mediated through altering hemagglutinin processing. The

BK23 | TABLE OF CONTENTS


primary locus of action is the influenza A virus M2 protein, an
integral membrane protein that functions as an ion channel.
By interfering with this function of the M2 protein, the drugs
inhibit the acid-mediated dissociation of the
ribonucleoprotein complex early in replication and potentiate
acidic pH–induced conformational changes in hemagglutinin
during its intracellular transport later in replication. (GG, 13th
ed p.1113)
B. Intracellular zidovudine is phosphorylated to zidovudine 5′-
triphosphate. Zidovudine-triphosphate terminates the
elongation of proviral DNA because it is incorporated by
reverse transcriptase into nascent DNA but lacks a 3′-
hydroxyl group. (GG 13th ed p.1140)
C. The exact mechanism(s) of action of ribavirin or its
phosphorylated derivatives in vivo are unknown, but several
immunomodulatory and antiviral effects have been observed
in vitro. (GG 13th ed p. 1129)
D. Cellular enzymes convert lamivudine to the triphosphate,
which competitively inhibits HBV DNA polymerase and 62. Which of the following is NOT a candidate for
causes chain termination. (GG 13th ed p.1125) cycloserine therapy?
A. Patient with history of alcoholism
60. Astrid, on her 25th week of gestation, wanted to go B. MTB infection
to the US for a business trip but she heard in the C. Patient with known E. coli infection
news regarding the flu outbreak. She went to your D. Patient diagnosed with seizure
clinic asking for advice regarding the best way to disorder
Rationale: (GG p.1078)
prevent influenza during her trip. What would be
your best advice to the patient?
A. Start Oseltamivir right away since it is
effective for Influenza A and B
B. Take Amantadine since it has lower GI
side effects
C. Educate her on how to protect herself
against airborne infections and
consider vaccination 63. When using as needed low dose budesonide-
D. Consider Zanamivir since it is easier to formoterol, what is the maximum recommended
administer total of formoterol in one day?
Rationale: (not sure)
Five drugs are currently approved for the treatment and prevention of
A. 50 mcg
influenza virus infection: the adamantine antivirals, amantadine B. 72 mcg
and rimantadine; oseltamivir; zanamivir; and peramivir. C. 90 mcg
Resistance to these drugs has arisen as a consequence of their D. 100 mcg
overuse, including in veterinary applications. Development of Rationale:
resistance and the spread of resistant viruses are major
challenges in the chemotherapy and chemoprophylaxis of
influenza and are likely to drive future recommendations for use
of these drugs in global populations. The CDC annually issues
recommendations for influenza vaccinations and comments on
effective medications.

(GG, 13th ed p.1112)

61. Which of the following theophylline side effects is


due to A1 receptor antagonism?
A. Behavioral disturbance
B. Seizure GINA PPT
C. Gastric discomfort
D. Headache 64. Myelosuppression side effects of Ganciclovir is more
Rationale: (GG p.735)
pronounced if the patient is concurrently taking this
drug
A. Paracetamol
B. Azathioprine
C. Prednisone
D. Cyclosporin
Rationale:

Zidovudine and probably other cytotoxic agents increase the risk of


myelosuppression, as do nephrotoxic agents that impair
ganciclovir excretion

BK23 | TABLE OF CONTENTS


Concomitant cyclosporine and probably other nephrotoxic agents
enhance the risk of nephrotoxicity

(GG p.1109 and 1111)

65. Which of the following antiretroviral agents should be


AVOIDED in anemic HIV individuals?
A. Atazanavir
B. Enfuvirtide
C. Delaverdine
D. Zidovudine 68. Drugs used to treat Hepatitis B include the following:
Rationale:
A. Polymerase inhibitors
B. Protease inhibitors
C. Integrase inhibitors
D. Nucleotide/nucleoside analogs
Rationale: GG 13th ed p. 1120

(GG p.1140)

66. Mary, 40 yo was admitted at the ICU for respiratory


distress secondary to MDR TB. Which of the
following antibiotics is shown to have activity
against MDR TB infection?
A. Ertapenem
B. Meropenem
C. Amoxicillin with clavulanic acid
D. A and B
E. All of the above
Rationale: GG 13th ed p. 1078

69. Which of the following is the most dangerous


symptom of isoniazid overdose?
A. Ataxia
B. Seizure
C. Metabolic acidosis
D. Slurred speech
Rationale: GG 13th ed p. 1074

70. Which of the following antibiotics is NOT indicated


for aspiration pneumonia?
A. Ampicillin sulbactam
B. Clindamycin
C. Clarithromycin
67. What is the use of a spacer chamber?
D. Moxifloxacin
A. This reduces the amount of drug that Rationale: CAP CPG 2016 p. 5
impinges on the oropharynx and
increases the proportion of drug
inhaled into the lower airways
B. It increases the velocity of the particles
from the pressurized metered dose
inhaler causing more particles of the
drug to go into the lower airways
C. It decreases the unpleasant taste of the
particles coming out of the inhaler
D. It is used to make holding the inhaler
device easier
Rationale:
GG 13th ed p. 730

BK23 | TABLE OF CONTENTS


74. Digoxin is a drug with known positive inotropic effect
71. This intravenous fluid is used for rapid rehydration in thus its use in heart failure. The mechanism of
a patient with severe dehydration digoxin that leads to increased myocardial
A. Albumin contractility is:
B. Plain lactated ringer's solution A. Increased sodium influx during
C. D5 0.9% NaCl depolarization
D. D5water B. Increased influx of calcium from the
Reference: Diarrhea CPG p. 51
ECF
C. Decreased efflux of potassium
D. Increased calcium release from the
sarcoplasmic reticulum
Reference: GG Ch29 p. 538
72. Your patient followed up at the clinic and complained
of orange-colored urine. He has been on Regimen
1 for PTB for just 3 days. What will you do?
A. Get urinalysis and a complete blood
count
B. Reassure the patient
C. Discontinue the medication and check
75. A 35 year old diabetic had a history of influenza 2
serum creatinine
weeks ago. She sought consult because of the
D. Shift to isoniazid/rifampicin and continue
recurrence of fever and cough. A repeat CXR
for 9 months instead of 6 months
reference : TB CPG p. 39 showed patchy infiltrates on both lung fields. What
organism likely has caused her pneumonia in this
setting?
A. Anaerobes
B. Streptococcus pneumoniae
C. Staphylococcus aureus
D. Legionella pneumophila
Reference: not sure; found this sa net

73. What ARV is eliminated by hepatic metabolism?


A. Nevirapine
B. Efavirenz
C. Delavirdine
D. All of the above 76. Which of the following statements correctly pairs the
Reference: Ch 64: GG 13th ed p 1144-1146 antifungal drug with its typical adverse effect?
A. Amphotericin B-hypertension
B. Terbinafine-visual and auditory
hallucinations
C. Flucytosine-myelosuppression
D. Voriconazole-acute kidney injury

BK23 | TABLE OF CONTENTS


Rationale: PPT lecture C. MRSA
Amphotericin B - fever and chills, renal impairment, hypotension,
D. A and B
thrombophlebitis
Rationale:
Terbinafine- GI disturbances, headache, rash
Flucytosine- neutropenia, thrombocytopenia, bone-marrow depression
Voriconazole- visual and auditory hallucinations

77. Amount of drug swallowed using conventional PMDI:


A. 10%
B. 50%
C. 80%
D. 100%
Rationale: (GG, p.729)
Of the total drug delivered, only 10-20% enters the lower airways with
a conventional pMDO.

78. Which of the laboratory tests should be ordered to


assess the side effect of antileukotrienes?
A. Creatinine
B. Protime
C. ECG
D. SGPT
Rationale: (GG, p.741) (GG, p.1068)
Zileuton, zafirlukast and montelukast are all associated with rare cases
of hepatic dysfunction, thus liver-associated enzymes should be
monitored. 81. When do you streamline antibiotic regimen for CAP?
A. Afebrile for more than 24 hours
79. CE, a 20-year-old medical technologist came in due B. RR is 30 cpm
to symptoms of nonproductive cough associated C. O2 saturation at room air is 90%
with shortness of breath usually occurring during D. BP is 100/70mmHg
her night shifts at work. She has been having these Rationale:
symptoms at least 1-2x a week for 2 months
already and has been attributing it to the cold and
dry air inside the laboratory. She has never been
diagnosed with asthma. You decided to start her on
asthma medication right away after a review of her
medical and family history, and physical
examination since she said that she will not have
time to have spirometry done for the next 2 weeks.
What regimen will you start her on?
A. As needed short-acting β2-agonist
B. As needed low dose inhaled
corticosteroid-formoterol
C. Daily medium-dose ICS-formoterol
D. Daily 10mg Montelukast

(CPG CAP)
82. A 50 year old man who is hypertensive and diabetic
was admitted for 5 day history of fever, productive
Rationale:
cough and tachypnea. He is awake, responsive but
weak. BP 90/60 mmHg, HR 125 bpm, RR 30 cpm,
Temp 38.5C. CXR revealed lobar pneumonia on
the right lung. He still has good urine output. He
had been seen at the ER OPD 2x for the past 2
months for cellulitis on his leg. He was unable to
recall the antibiotics prescribed. Based on the
GINA PDF category of his pneumonia, how long will you treat
this patient?
80. Which of the following is/are inhibited by rifampicin? A. 3-5 days
A. Mycobacterium sp. B. 5-7 days
B. S. aureus C. 7-14 days

BK23 | TABLE OF CONTENTS


D. 14-21 days
Rationale: Not sure if patient is experiencing S. Pneumoniae
symptoms (lobar pneumonia)

(CPG DIARRHEA )

84. Which of the following antibiotics is NOT indicated


for ESBL (extended spectrum beta-lactamase)
producing bacteria?
A. Ertapenem
B. Aztreonam
C. Imipenem
D. Meropenem
Rationale:

(GG, p.1035)

85. Jay, 35 yo is a known asthmatic on as needed


salbutamol inhaler. He noted 3 days of non-
productive cough and coryza. Night prior to
admission he noted shortness of breath temporarily
relieved by nebulization. Persistence of symptoms
sought consult at the ER. Vital signs: BP 120/90,
HR 105 bpm, RR 28 cpm, Temp 36 C, Sat 90%. On
auscultation, poor airway entry. Which of the
following should be given to the patient?
A. IV Hydrocortisone
B. Salbutamol + ipratropium nebulization
C. Oral prednisone
83. An infant with mild to moderate dehydration has a
D. Salmeterol + fluticasone inhaler
fluid deficit of:
E. A and B
A. <5%
B. 5-10%
C. >10%
D. 15%
Rationale: 86. A diabetic patient with gastroparesis was given with
Bethanecol to relieve the feeling as if her stomach
is full after eating very little. What is the drug
classification of Bethanechol?
A. A1-adrenoceptor antagonist
B. M3-receptor agonist
C. D2-receptor antagonist
D. 5-HT4 receptor antagonist
Rationale:

BK23 | TABLE OF CONTENTS


Cholinomimetic agonists such as bethanechol stimulate muscarinic persistently febrile neutropenic patients with suspected fungal
M3 receptors on muscle cells and at myenteric plexus infections (Walsh et al., 2004). (G&G 13th Ed. pg. 1097)
synapses. Bethanechol was used in the past for the treatment
of GERD and gastroparesis. Of note, caspofungin is licensed for use in invasive aspergillosis
(Katzung 14th Ed. pg. 1097) only as salvage therapy in patients who have failed to
respond to amphotericin B, and not as primary therapy.
(Katzung 14 Ed. pg. 860)
87. Which of the following differences of Amantadine
and Rimantadine is/are correct?
A. Amantadine is more potent than 90. A patient undergoing chemotherapy with Cisplatin
Rimantadine has severe nausea. Which of the following would be
B. Amantadine has greater bioavailability the drug to use in this patient?
than Rimantadine A. Cyclizine
C. Exacerbations of previous seizures B. Naloxone
occur frequently with Rimantadine use C. Ondansetron
than Amantadine use D. Scopolamine
Rationale:
D. CNS side effects are more frequent Marked nausea and vomiting occur in almost all patients and usually
with Amantadine use can be controlled with 5HT 3 receptor antagonists,
Rationale: neurokinin (NK1 ) receptor antagonists (e.g., aprepitant), and
The most common side effects related to amantadine and rimantadine high-dose corticosteroids. (G&G 13th Ed. pg. 1176)
are minor dose-related CNS and GI effects: nervousness, light-
headedness, difficulty concentrating, insomnia, loss of appetite, H1 receptor antagonist - Cyclizine
and nausea. CNS side effects (5%–33%) occur in patients Opioid antagonist - Naloxone
treated with amantadine at doses of 200 mg/d but are 5HT 3 receptor antagonist - Ondansetron
significantly less frequent with rimantadine. (G&G 13th Ed. Antimuscarinic agent - Scopolamine
pg. 1114)
91. This is the most essential composition in ORS
88. What is the regimen of choice for TB arthritis? solution
A. 2 months of isoniazid/rifampicin/ A. Sodium
pyrazinamide/ethambutol and 4 months B. Potassium
of isoniazid/rifampicin C. Glucose
B. 4 months of isoniazid/rifampicin/ D. Lactate
pyrazinamide/ethambutol and 10 Rationale:
months of isoniazid/rifampicin
C. 12 months of isoniazid/rifampicin/
pyrazinamide/ethambutol
D. 2 months of isoniazid/ rifampicin/
pyrazinamide/ ethambutol and 10
months of isoniazid/rifampicin Rehydrate.org
Rationale:

92. In the Global Initiative for Asthma (GINA 2020), the


preferred reliever is:
A. As needed short-acting β2-agonist such
as salbutamol
B. As needed low dose inhaled
corticosteroid-formoterol
89. Which drug can be used for invasive Aspergillosis in combination
patients who cannot tolerate other antifungal C. As needed 2mg salbutamol capsule
medications? D. As needed budesonide nebulization
A. Caspofungin Rationale:
B. Fluconazole
C. Nystatin
D. Flucytosine
Rationale:
Caspofungin is approved for the treatment of invasive candidiasis and
as salvage therapy for patients with invasive aspergillosis
who fail or are intolerant of approved drugs, such as
amphotericin B formulations or voriconazole. Caspofungin is also
approved for both esophageal and invasive candidiasis (Mora-
Duarte et al., 2002; Villanueva et al., 2001) and for treatment of

BK23 | TABLE OF CONTENTS


95. What antibiotic can be prescribed for a patient with
CAP Low Risk who is allergic to penicillin?
A. Amoxicillin
B. Azithromycin
C. Clarithromycin
D. A and B
E. B and C
Rationale:

GINA 2020, p. 21

93. The reduced osmolarity ORS contains osmolarity of:


A. 170 mOsm/l
B. 245 mOsm/l
C. 275 mOsm/l
D. 311 mOsm/l
Rationale:

CAP Guidelines 2016

96. True statement regarding tuberculosis diagnosis and


treatment:
A. New – refers to a patient who has never
had treatment for TB or who has taken
anti-TB drugs for less than 2 months
B. Previously treated for TB
(Retreatment) – refers to a patient
who had received one month or more
of antiTB drugs in the past
Diarrhea PPT p. 26
C. Direct sputum smear microscopy is still
94. Which of the following principles in the treatment of the primary diagnostic test for
diarrhea in children is true? pulmonary and extrapulmonary
A. Offer the child food 3x a day to avoid tuberculosis
vomiting D. Tuberculosis primarily involving the
B. Child with diarrhea who are not lymphnodes in the presence of minimal
breastfed should be given special milk upper lobe infiltrates is still considered
formula temporarily as extrapulmonary tuberculosis
C. You may give as much fluids as they
want for older children
Rationale:
D. Zinc may be given for 7 days

Rationale:

DOH TB PDF

97. Which of the following anti-fungal drugs should be


avoided in patients with evidence of ventricular
dysfunction?
A. Micafungin
B. Itraconazole
C. Terbinafine
D. Posaconazole
CPG Acute Infectious Diarrhea Rationale:

BK23 | TABLE OF CONTENTS


GG 13th ed page 1102

98. The test of choice for sputum follow-up examination


for patients on DS-TB regimen:
A. Direct sputum smear microscopy
B. Xpert MTB/Rif test
C. Tuberculin skin test
D. Sputum TB culture
Rationale:

DOH TB PDF, PAGE 34

99. Which of the following is TRUE of ethambutol?


A. Pretreatment visual acuity testing is
recommended
B. Hyperuricemia is seen in 90% of
patients
C. Decrease in visual acuity is always
bilateral
D. All of the above
Rationale:

GG 13th ed CHAP 60 PAGE 1074

100. Which of the following NRTI’s is associated with the


development of Lipodystrophy?
A. Abacair
B. Didanosine
C. Stavudine
D. Zalcitabine
Rationale:

GG CHAPTER 64

BK23 | TABLE OF CONTENTS


Pharmacology 3_1 4. Stef, 35 yo was seen at the OPD due to fever and
dysuria. She had previous episodes of UTI in the
past months thus urine culture and sensitivity test
1. A 31-year-old actor brought to the emergency room. was ordered which showed E. coli sensitive to
He allegedly ingested 30 tablets of 500 mg
Ciprofloxacin. She was given Ciprofloxacin 500mg
paracetamol half an hour ago. What intervention
BID for 7days. What is the mechanism of action of
should be done next? quinolones in Stef’s case?
A. Intubate the patient and attach to
A. Inhibits bacterial DNA gyrase
mechanical ventilator if breathing is
B. Inhibits bacterial topoisomerase IV
compromised
C. Inhibits bacterial dihydrofolate reductase
B. Gastric lavage
C. Administer the antidote N-Acetylcysteine D. Inhibits dihydrofolate reductase
Rationale: Pinca (Trans lifted from GG)
D. Give multiple-dose activated charcoal
Rationale: Supportive measures are the basics (SBCDs) of poisoning
treatment. First, the airway should be cleared of vomitus or any other
obstruction and an oral airway or endotracheal tube inserted if needed.
For many patients, simple positioning in the lateral, left-side-down
position is sufficient to move the flaccid tongue out of the airway.
A - airway
B - breathing
C - circulation
D - dextrose
From: Katzungs, 14th Edition, Chapter 58: Management of the
Poisoned Patient, p.1037

2. During induction a patient was given succinylcholine Since E. Coli is Gram negative, the quinolone’s primary target is the
as muscle relaxant. Which of the following is an “DNA Gyrase”.
appropriate reversal agent for succinylcholine?
A. Neostigmine 5. Which of the following conditions would be a
B. Atropine contraindication to start Benznidazole for Chaga’s
C. Acetylcholine Disease?
D. None A. 50 year old man
Clinical Anesthesia by Paul Barash, etc., 7th ed, Ch 20, pg 533 B. HIV patient
C. Decompensated Heart disease
D. Patient presenting with a purplish skin
rash with lymphadenopathy
Rationale: not sure but urticaria dermatitis is the only one stated in the
book to discontinue the drug (GG p.993)

3. Which among the quinolone have enhanced potency


against Pseudomonas spp?
A. Ciprofloxacin
B. Gatifloxacin 6. Which of the following is considered a side effect of
C. Moxifloxacin amphotericin B?
D. Norfloxacin A. Cardiotoxicity
Rationale: Katzung, 14th ed, p 838 B. Neurotoxicity
C. Nephrotoxicity
D. Ototoxicity
Rationale: Known for nephrotoxicity? Used as lipid formulation to
decrease toxicity

BK23 | TABLE OF CONTENTS


10. What is the contributing factor for the prolonged
antilesmanial effect of Sodium Stibogluconate?
A. It is rapidly absorbed after oral administration
and distributed to tissues
B. It is slowly excreted by the kidneys
C. It is sequestered in the macrophages
D. It undergoes slow hepatic metabolism
Rationale:
(Not sure, B or C?) GG 13th ed, page 995

The drug is given intravenously or intramuscularly; it is not active


orally. The agent is absorbed rapidly and distributed in an apparent
volume of about 0.22 L/kg. Elimination occurs in two phases, the first
with a t 1/2of about 2 h, the second with a much longer half-time (33–
7. Local anesthetics act on which channel 76 h). The prolonged terminal elimination phase may reflect conversion
A. Acetylcholine receptor channels of the Sb 5+ to the more toxic Sb 3+ that is concentrated in and only
slowly released from tissues. The drug is eliminated in the urine.
B. Na+ Channels
C. Calcium channels The pentavalent antimonials act as prodrugs that are reduced to the
D. GABA receptor channels more toxic Sb 3+ species that kill amastigotes within the
Rationale: Clinical Anesthesia by Barash, Cullen 7th edition pg 561 phagolysosomes of macrophages.

11. Herbal medication known to treat urinary stones:


A. Ginger
B. Sambong
C. Saw palmetto
D. Gingko biloba
8. The most common significant finding of
nephrotoxicity after aminoglycoside use?
A. Mild proteinuria
B. Appearance of granular casts in urinalysis
C. Mild elevation in creatinine
D. Presence of bilirubinuria
Rationale: Goodman and Gilman 13th Ed p.1044
Although severe acute tubular failure

9. An undesired toxic effect to which an individual


manifests severe toxicity in low doses or markedly
resistant to high doses of the agent: 12. The following are characteristics of non-depolarizing
A. Tolerance neuromuscular blocker EXCEPT
B. Idiosyncratic reaction A. Rapid in onset
C. Idiopathic reaction B. Highly ionized
D. Allergic reaction C. Poor lipid solubility at physiologic pH
Idiosyncrasy - is an abnormal reactivity to a chemical that is peculiar to D. Provide reversible antagonism at the NMJ
a given individual; the idiosyncratic response may be extreme
sensitivity to low doses or extreme insensitivity to high doses of drugs
with ACh
G&G page 58

anesthesia ppt, slide 43

BK23 | TABLE OF CONTENTS


13. Atracurium undergoes metabolism through
A. Plasma cholinesterase
B. Pseudocholinesterase
C. CYP450
D. Hoffman degradation
Rationale:
Atracurium: metabolized spontaneously in plasma by Hofmann
degradation and causes histamine release;

Mivacurium: short acting and metabolized by plasma cholinesterase.


The esters are hydrolyzed in the blood by pseudocholinesterase.

17. Which of the following would be considered side


14. TRUE of Toxicokinetics: effect/s of Sulfonamides?
A. The slower the absorption rate, the higher the A. Anemia
toxicity B. Crystalluria
B. The organ where most of the toxic agent is C. Rash
distributed always has the highest toxic effect D. All of the above
C. The more rapid the excretion of the drug, Rationale: Page 013
the less toxicity is produced A. Anemia: rare acute hemolytic anemia
B. Crystalluria: low occurrence, seen in dehydrated HIV patients
D. All of the above
C. Rash: occurs most often after first week of therapy

18. Goals of General Anesthesia do not include


A. Amnesia
B. Muscle relaxation
C. Hypnosis
D. Nerve blockade

15. Quinolones and drugs that decreases its


bioavailability should be given :
A. Before meals altogether
B. After meals 12 hours apart
C. 2 hours apart 19. A patient undergoing ambulatory Upper
D. Mixed with food Gastrointestinal Endoscopy was given
Rationale: Midazolam 5mg. The procedure lasted only
All quinolones form complexes with divalent and trivalent cations (e.g., 5minutes. Which medication can be given so
calcium, iron, aluminum). When coadministered orally with quinolones,
patient can be discharged home without residual
these cations can chelate the quinolone and reduce systemic
bioavailability. Thus, a separation of at least 2 h between oral
anesthetic effects?
administration of quinolones and these cations is recommended (p. A. Naloxone
1018) B. Nalbuphine
C. Flumazenil
16. The CORRECT descending order of D. Diazepam
effectiveness of drugs by route: Rationale:
A. Parenteral – Intraperitoneal – Intramuscular – Flumazenil is used to reverse the effects of Benzodiazepine
Subcutaneous
B. Oral – Parenteral – Inhalational – 20. Which of the following lab findings is consistent
Subcutaneous with chronic use or TMP SMX toxicity?
C. Parenteral – Intramuscular – Inhalational – A. WBC count of 15000 in CBC
Oral B. Elevated serum potassium
D. Parenteral – Subcutaneous – C. Decreased creatinine
Intramuscular – Oral D. Protime with INR of 1.0
Rationale:
Trimethoprim has the same structure as Potassium sparing diuretics

BK23 | TABLE OF CONTENTS


21. Inappropriate adaptation of the body in response
to a toxic agent may result to which of the
following effect?
A. Cancer
B. Impaired cellular maintenance
C. Dysregulation of gene expression
D. All of the above

24. TRUE of Herbal Medicines:


22. At the post anesthesia care unit (PACU), a A. Generally indicated for chronic medical
patient who underwent exploratory laparotomy conditions
under general anesthesia complains of B. Contain excipients and other chemically-
generalized muscle pains. Which muscle defined active substances
relaxant was most likely given? C. Plant-based preparations allowed to be
A. Rocuronium marketed for cure, treatment or diagnosis of
B. Vecuronium diseases
C. Atracurium D. All of the above
D. Succinylcholine
Rationale: Page 179; Goodman and Gillman 13th Ed

25. The mechanism behind the administration of


multiple-dose activated charcoal in acute
poisoning treatment is:
A. Decontamination of toxic agent
Page 483; Katzung 14th ed
B. Neutralization of the toxic substance
C. Conversion of the toxic agent to a non-toxic
metabolite
D. Interruption of the enterohepatic
recirculation of the toxic agent
Rationale: G&G p.63

23. Property/ies of a drug to be effectively removed 26. He introduced labor analgesia using
by dialysis: chlorophorm on Queen Victoria.
A. Low volume of distribution A. John Snow
B. Not protein bound B. Need Stark
C. Small molecular weight C. Leon Greyjoy
D. All of the above D. Tyrion Lannister
Rationale: ppt Doc Chan

BK23 | TABLE OF CONTENTS


30. A 25 year old indigent patient who is 12 weeks
pregnant showed you her laboratory result at the
ER. CBC showed hgb 14 g/dl, hct 42%, wbc
7000 x 103/ul, platelet 240 x 103/ul. Urinalysis
showed color light yellow, ph 6, specific gravity
1.020, protein trace, glucose negative, ketones
negative, blood negative, WBC 15/hpf, epithelial
cells rare, and bacteria moderate. She has no
other complains. What will you do next?
A. You admit the patient immediately
B. You must get urine culture before staring
antibiotic
27. The CORRECT Drug-Herb interaction effect C. You start oral antibiotic
involving Ginseng: D. You should get blood culture
A. Enhance sedation when given with trazodone
B. It enhances the antiplatelet effect of
aspirin
C. Increased INR with warfarin
D. All of the above
Rationale:
-Ginseng has antiplatelet activity that when combined with aspirin may
enhance the antiplatelet effects
-Ginseng interferes with warfarin, it counteract its effect and results to
decrease in INR 31. A pregnant woman sought consult in your clinic
with complaints of urinary frequency and
28. What is the drug of choice for antimonial dysuria. You are considering cystitis and plan to
resistant leishmaniasis? give trimethoprim-sulfamethoxazole. When is
A. Amphotericin B this antibiotic contraindicated?
B. Antimony A. First trimester
C. Miltefosine B. Second trimester
D. Ketoconazole C. Third trimester
D. All throughout pregnancy

29. Which of the following is also known as laughing


gas 32. What drug causes brownish discoloration of the
A. Nitrogen urine?
B. Nitrous oxide A. Nitazoxanide
C. Helium B. Metronidazole
D. Xenon C. Nifurtimox
D. Rifampicin

BK23 | TABLE OF CONTENTS


36. One of the following is NOT an adverse effect of
TMP SMX:
A. Steven Johnson syndrome
B. Henoch Schonlein purpura
C. Folate deficiency
D. Sweets Syndrome

33. Which of the following statements best describes


the pharmacokinetics of trimethoprim and
sulfamethoxazole?
A. The drugs does not enter the blood brain
barrier
B. Trimethoprim has longer half life 37. Supplement with beneficial role in the treatment
C. Single strength TMP SMX has 80mg of COVID19 infection
A. CoQ
sulfamethoxazole
B. Melatonin
D. Peak concentrations of trimethoprim is
C. Glucosamine
achieved longer that sulfamethoxazole
Rationale: GG page 1014 D. Garlic
The half lives of TMP-SMX are 11 and 10 hours respectively Mentioned by Doc Bongon

34. Which of the following drugs penetrates the


CNS?
A. Amphotericin
B. Eflornithine
C. Metronidazole
D. All of the above
GG Page 990
Eflornithine:
The drug does not bind to plasma proteins but is well
distributed and penetrates into the CSF where estimated
concentrations of at least 50 um must be reached for parasite
clearance. 38. Which of the following conditions would you
GG page 992 screen for asymptomatic bacteriuria?
Metronidazole penetrates well into body tissues and fluids, A. 30 year old diabetic man
including vaginal secretions, seminal fluid, saliva, breast milk, and B. 65 year old diabetic woman
CSF.
GG page 1088
C. 35 year old post renal transplant man
Amphotericin B D. 50 year old man with enlarge prostate for
Regardless of formulation, very little amphotericin B transrectal biopsy
penetrates into CSF, vitreous humor, or normal amniotic fluid.

35. What is the long term side effect of


Nitrofurantoin?
A. Acute Pneumonitis
B. Gastrointestinal distress
C. Hypersensitivity reaction
D. Pulmonary fibrosis
Rationale:
GG page 1018
Interstitial pulmonary fibrosis can occur in patients (especially the
elderly) taking the drug chronically.

BK23 | TABLE OF CONTENTS


39. The CHOLINERGIC toxidrome include:
A. Diarrhea, mydriasis, fever, tachycardia
B. Diarrhea, miosis, lacrimation, bradycardia
C. Diarrhea, miosis, blindness, dryness
D. Delirium, mydriasis, blindness, dryness

43. Antidote for Isoniazid toxicity:


40. While the patient was taking metronidazole for 2 A. Deferroxamine
weeks to treat his liver abscess, he complained B. Pyridoxine
of numbness and tingling sensation of his C. Flumazenil
extremities. What will will be your plan for this D. Thiamine
patient? Rationale: GG 13th ed pg 63
A. Administer antihistamine
B. Administer vitamin B complex
C. Discontinue the drug
D. Gradually increase the dose of the drug
E. Lower the dose of the drug

41. Which of the following statements is TRUE to


Methenamine?
A. It is a urinary tract antiseptic agent
B. It generates formaldehyde
C. It works best in an acidic medium (i.e.urine)
D. All of the above 44. “Before whom in all time, Surgery was agony” is
written on who’s epitaph?
A. William T.G. Morton
B. John Snow
C. Frances Burney
D. Joseph Priestly
Rationale: UCSM Pharma Anes Lec PPT

45. What is the most common side effect of


Metronidazole?
42. Which of the following conditions will you
A. Bone marrow suppression
withhold the administration of sulfonamide?
A. 3 year old with Pneumonia B. Metallic taste
B. 30 year old with Pneumocystis Pneumonia C. Rash
C. 13 year old with Shigellosis D. Vertigo
Rationale: Goodman and Gilman 13th edition page 992
D. 23 year old pregnant in her 3rd trimester Common side effects are headache, nausea, dry mouth, and a metallic
with UTI taste.

BK23 | TABLE OF CONTENTS


46. The MAINSTAY of therapy of all poisoning 49. A 75yo male undergoing hip arthroplasty under
cases: epidural anesthesia suddenly develops
A. Emergency stabilization hypotension and new onset arrhythmias are
B. Enhancing elimination of the absorbed poison seen on the ECG. Which local anesthetic was
C. Antidote administration most likely used in this patient?
D. Supportive treatment and Observation A. Lidocaine
Rationale: Goodman and Gillman’s 13th ed., pg 63 B. Bupivacaine
C. Ropivacaine
D. Prilocaine

47. MIOSIS is typically seen in:


A. Morphine toxicity
50. Which of the following aminoglycoside is
B. Methamphetamine toxicity
considered the LEAST nephrotoxic?
C. Atropine toxicity
A. Neomycin
D. Diphenhydramine toxicity
B. Gentamicin
C. Streptomycin
D. Tobramycin
Rationale: Goodman and Gilman 13th edition page 1044

48. What is the drug of choice for late stage East


African trypanosomiasis?
A. Melarsoprol
B. Miltefosine
C. Metronidazole
D. Nitazoxanide
Rationale: Goodman & Gilman’s - The Pharmacological Basis of
Therapeutics 13th ed pg 991

BK23 | TABLE OF CONTENTS


Pharmacology 3_2 4. Which of the following is NOT a WHO
recommendation for the control of malaria?
A. Use of insecticide treated nets
1. The anti-mitotic effect of colchicine takes place in
B. Prompt malaria treatment for suspected
this phase of cell cycle
individuals
A. G0
B. G1 C. Indoor residual spraying
C. S D. Prompt access to diagnostics in individuals
D. M with signs and symptoms
Rationale: GG, 13th edition, p. 702 Rationale: Internet

5. Arthur 55y.o, OFW, working as a welder in Nigeria is


admitted at the ICU and treated for malaria. After a
few days of treatment, patient noted to have sudden
onset of hypotension as low 60/40 mmHg with
nausea and vomiting. His capillary blood glucose is
2. Which of the following is NOT a principal target persistently low at 60-80 mg/dl. The infectious
organ in artemisinin toxicity studies? disease specialist thinks this could be an adverse
A. Liver effect of a medication. Which of the following
B. Bone marrow medication/s could have caused this?
C. Brainstem A. Chloroquine
D. Kidney B. Mefloquine
C. Quinine
D. Primaquine
Rationale: From Goodman & Gilman’s 13th ed, Chapter 53, page 979

3. Which of the following is/are true regarding chronic


fluoride poisoning?
6. Arthur 55 yo obese, non-hypertensive and diabetic
A. Dental fluorisis can be prevented by using
with good sugar control was admitted and diagnosed
flouride after tooth eruption
with uncomplicated malaria. Which of the following
B. Osteosclerosis is characterized by increase
statements is TRUE of obese/large adults with
in bone density secondary to increase
uncomplicated P. falciparum malaria?
osteoclastic activity (correct would be osteoblastic)
A. Dosage is calculated according to age
C. Mottling is one of the first signs of
B. Dose is based on ideal body weight
fluoride excess
D. None of the above is true C. Same dosage is followed for lighter patients
GG 13th ed page 901 D. Maximum dose available and possible should
be given
Rationale: WHO Malaria Guidelines p. 266

BK23 | TABLE OF CONTENTS


7. Which of the following drug/s is NOT compatible with 10. Baby Mico was born preterm with 35 weeks Ballard
artesunate + amodiaquine due to its hematologic score. She was purely breastfeed by her mother
and hepatic toxicity when used altogether? Carla. Mother Carla was lectured by her neighbour
A. Rifampicin Alec who is a medical student that human milk only
B. Zidovudine contains Vitamin D concentration of 25IU/L or less
C. Clindamycin and thus breast milk alone cannot provide the
D. Lopinavir based ARV adequate Vitamin D needed for her baby. How much
Rationale: Malaria guidelines 3rd ed. Vitamin D supplementation must Alec advised to
Carla for her baby?
A. 100 IU per day
B. 200 IU per day
C. 300 IU per day
D. 400 IU per day
GG, page 888

8. Which of the following bisphosphonates is


CORRECTLY PAIRED?
A. First generation- Pamidronate
B. Second generation- Tiludronate
C. Third generation- Zoledronate
D. Fourth generation- Clodronate
GG 13th ed, page 898 11. What is the role of thiazide diuretics in the treatment
of osteoporosis?
A. Increases bone resorption
B. Enhances absorption of Ca in the tubules
C. Decreases urinary Ca excretion
D. Increases new bone formation
loop = hypocalcemia; increase Ca excretion;
thiazide = hypercalcemia; decrease Ca excretion

9. Recommended dose of artesunate in a 15 kg child


GG 13th Ed, Ch25, Page 455
with severe malaria
A. 2.4mg/kg/dose
B. 3mg/kg/dose
C. 2.4 mg/kg/min
D. 5mg/kg/day
Malaria Guidelines page 72

GG 13 Ed, Ch 25, Page 896

BK23 | TABLE OF CONTENTS


12. Which of the following is NOT contraindicated in 15. A salicylate used in the treatment of Inflammatory
malaria treatment for pregnants? Bowel Disease:
A. Hydroxychloroquine A. 5 aminosalicylic acid
B. Coartem B. Choline-Mg salicylate
C. Atovaquone + proguanil C. Diflusinal
D. Mefloquine D. Methyl salicylate
GG 13th ed, p. 981 Mesalamine (5-aminosalicylic acid) is a salicylate that is used for its
local effects in the treatment of inflammatory bowel disease. Oral
formulations that deliver drug to the lower intestine are efficacious in
the treatment of inflammatory bowel disease (in particular, ulcerative
colitis). These preparations rely on pH-sensitive coatings and other
delayed-release mechanisms such as linkage to another moiety to
create a poorly absorbed parent compound that must be cleaved by
bacteria in the colon to form the active drug. Mesalamine is available
as a rectal enema for treatment of mild-to-moderate ulcerative colitis,
proctitis, and proctosigmoiditis and as a rectal suppository for the
treatment of active ulcerative proctitis. Mesalamine derivatives in
clinical use include balasalazide, sulfasalazine, and olsalazine.
13. This is a non-purine xanthine oxidase inhibitor Goodman and Gilman’s The Pharmacological Basis of Therapeutics,
approved for use among hyperuricemic patients with 13th Edition P. 694
gout attacks but not recommended for treatment of
asymptomatic hyperuricemia 16. This antimalarial drug may exacerbate psoriasis
A. Rasburicase A. Chloroquine
B. Allopurinol B. Hyroxychloroquine
C. Feboxustat C. Atovaquone/Proguanil
D. Colchicine D. Doxycycline
Rationale:
Febuxostat is a nonpurine inhibitor of XO. Unlike oxypurinol, the active
metabolite of allopurinol, which inhibits the reduced form of XO,
febuxostat forms a stable complex with both the reduced and oxidized Source: Goodman and Gilman’s The Pharmacological Basis of
enzymes and inhibits catalytic function in both states. Therapeutics, 13th Edition P. 977

Febuxostat is approved for hyperuric patients with gout attacks but is


not recommended for treatment of asymptomatic hyperuricemia. It is
17. What antimalarial agent causes “Black water fever”?
available in 40- and 80-mg oral tablets. A. Primaquine
B. Artemisinin
Source: Goodman and Gilman’s The Pharmacological Basis of C. Chloroquine
Therapeutics, 13th Edition P. 704
D. Quinine
Severe hemolysis can result from hypersensitivity to these cinchona
14. This is the pathognomic sign of gouty arthritis alkaloids. Hemoglobinuria and asthma from quinine may occur more
A. Heberden’s nodes rarely. “Blackwater fever”—the triad of massive hemolysis,
B. Tophi hemoglobinemia, and hemoglobinuria leading to anuria, renal failure,
and in some instances death—is a rare hypersensitivity reaction to
C. Renal uric acid stones quinine therapy that can occur during treatment of malaria. Goodman
D. Podagra and Gilman’s The Pharmacological Basis of Therapeutics, 13th Edition
P. 979

18. Which of the following statements is NOT a


characteristic of Pyrimethamine?
A. It has a very long half life
B. It can cause megaloblastic anemia
Source: Doc Camomot’s Lecture C. Its therapeutic response is enhanced by
administration of folic acid
D. It distributes well into tissues
A. Pyrimethamine is slowly eliminated from plasma, with a t1/2 of 85–
100 h.
B. Excessive doses can produce a megaloblastic anemia, resembling
that of folate deficiency, which responds readily to drug withdrawal or
treatment with folinic acid.
D.The compound is significantly distributed in the tissues and is about
90% bound to plasma proteins.
Source: Goodman and Gilman’s The Pharmacological Basis of
Therapeutics, 13th Edition P. 975

BK23 | TABLE OF CONTENTS


19. The following are adverse effects of calcium
administration EXCEPT:
A. Peripheral vasodilation
B. Hypotension
C. Cutaneous burning sensation
D. Kidney stones
Injections of calcium chloride are accompanied by peripheral
vasodilation and a cutaneous burning sensation. The usual intravenous
preparation is a 10% solution (equivalent to 1.36 mEq Ca2+/mL). The
rate of injection should be slow (not more than 1 mL/min) to prevent
cardiac arrhythmias from a high concentration of Ca2+. The injection
may induce a moderate fall in blood pressure owing to vasodilation.

Source: Goodman and Gilman’s The Pharmacological Basis of


Therapeutics, 13th Edition P. 896
23. After an adolescent sustained a fracture, how long
does it take for fracture to heal (complete remodeling
20. This bisphosphonate is more superior than
cycle)?
pamidronate in rapid normalisation of serum calcium
A. 6 weeks
and longer duration of action
B. 6 months
A. Alendronate
C. 8 months
B. Zolendronate
D. 12 months
C. Etidronate
D. Residronate
Intravenous bisphosphonates (pamidronate, zoledronate) have proven
very effective in the management of hypercalcemia. These agents
potently inhibit osteoclastic bone resorption. Pamidronate is given as
an intravenous infusion of 60–90 mg over 4–24 h. With pamidronate,
resolution of hypercalcemia occurs over several days, and the effect
usually persists for several weeks. Zoledronate has largely superseded
pamidronate because of its more rapid normalization of serum Ca2+ 24. Rosanna got married to a Nigerian boyfriend after
and longer duration of action. being discharged because of malaria where she was
treated with Mefloquine. When is Rosanna safe to
Source: Goodman and Gilman’s The Pharmacological Basis of get pregnant considering her mefloquine drug use?
Therapeutics, 13th Edition P. 896
A. 3 weeks after
B. 1 month after
21. Pregnancy should be avoided for 3 months after use
C. 2 months after
of this drug because of its prolonged t 1/2
D. 3 months after
A. Co-Artem
Page 979, Goodman and Gilman’s 13th ed
B. Mefloquine
C. Primaquine
D. Doxycycline
p. 979, Goodman and Gilman’s 13th ed

25. This drug is known to react with the anti-platelet


effects of Aspirin
A. Ibuprofen
22. Concomitant treatment with this anti-malarial drug B. Celecoxib
and Rifampicin for tuberculosis increases the C. Piroxicam
recrudescence rate of malaria infection fivefold D. Acetaminophen
higher
A. Quinine
B. Primaquine
C. Sulfadoxine
D. Tetracycline
GG p.979

BK23 | TABLE OF CONTENTS


26. The reason for increase acute gouty attacks during 29. Kris Sy sought consult to Dr. Rivera to ask advice on
initial phase of treatment of gout is: prophylaxis against malaria because she plans to go
A. Rapid and significant decrease in ambient to Palawan for a show 2 weeks from now. Kris Sy is
rate concentration following treatment pregnant and in her first trimester. What would be
B. Non-compliance of the ULT the most appropriate regimen that Dr. Rivera should
C. It depends on the severity of the gouty attack advise to her?
prior to ULT A. Atovaquone/Proguanil 1 tablet orally once a
D. Due to partial response to ULT day starting 1-2 days before travel until 7
days after leaving the area
B. Primaquine 1 tablet orally once a day
starting starting 1-2 days before travel until 7
days after leaving the area
C. Doxycycline 1 tablet orally once a week
starting 1-2 days before travel until 4 weeks
after leaving the area
27. The following are indications to start urate lowering D. Hydroxychloroquine 1 tablet orally once
drugs on patients with gout EXCEPT a week starting 1-2 weeks before travel
A. More than 2 gout attacks per year until 4 weeks after leaving the area
Only hydroxychloroquine is safe in pregnant women.
B. Chronic kidney disease stage 3 (Drug Facts for Your Personal Formulary: Regimens for Malaria
C. Uric acid level of 10mg/dL Treatment) GG 13th ed p. 983-985
D. Family history of gout
Gout guidelines 30. A 45 year old woman underwent parathyroidectomy.
The next day, patient was noted to have numbness
and tingling sensations in the perioral area,
carpopedal spasm and laryngospasm. What is the
best drug that can be given to this patient?
A. Calcium chloride to be given
intravenously
B. Calcium carbonate to be given orally
C. Calcium chloride to be injected directly to the
tissues
D. Calcium carbonate to be given as infusion
GG 13th ed p. 896

28. Which part of the kidneys will phosphate contribute 31. A 50 year old alcoholic patient diagnosed with gout
to the renal buffering system? one year ago came to the clinic with swollen left
A. Proximal convoluted tubule wrist, after a drinking spree last night. Severity was
B. Descending part of the loop of Henle moderate. He was maintained on Allopurinol but was
C. Ascending part of the loop of Henle non-compliant. What treatment regimen would you
D. Distal convoluted tubule give to the patient at this point?
GG 13th edition pg 889 A. Loading dose of 1.2mg colchicine
followed by 0.6mg colchicine one hour after
B. Colchicine 0.6 mg three times a day
C. Intramuscular triamcinolone acetonide
D. Intramuscular Ketorolac
GG 13th ed p. 703

BK23 | TABLE OF CONTENTS


34. Which of the following antimalarial prolongs the QT
interval?
A. Piperaquine
B. Chloroquine
C. Both
D. None of the above
P 40 Malaria Guidelines 3rd ed

32. Plasmodium knowlesi infection is often


misdiagnosed with what plasmodium specie seen in
light microscopy?
A. P. falciparum
B. P. vivax 35. Which of the following is a CORRECT statement/s
C. P. ovale regarding artemisinins?
D. P. malariae A. Can be given in oral and rectal routes only
GG 13th ed p. 970 B. Can’t be used for prophylaxis
C. Highly effective first line treatment even when
used alone
D. High bioavailability
P 973 GG

33. What is the consequence of severe hypercalcemia


due to vitamin D toxicity in children?
A. Congenital supravalvular aortic stenosis
B. Impaired growth spurt
C. Osteomalacia
D. Osteonecrosis 36. The safest and best drug for chemoprophylaxis of
long term travellers:
A. Doxycycline
B. Primaquine
C. Chloroquine
D. Mefloquine

37. A 25 year old reporter who visited Rizal, Palawan


developed malaria. At the E.R. he is drowsy, very
weak & highly febrile. Laboratory tests showed
delicate ring forms, banana shaped gametocytes,
pigment containing trophozoites and schizonts. The
parasite density is 100,000/ul, hgb 7g/dl, hct 26 %,
platelet 70,000/mcl, cbs 70mg/dl, creatinine 2mg/dl.
What is your consideration?
A. Complicated Plasmodium ovale malarial
infection
B. Complicated Mixed malarial infection
C. Severe Plasmodium falciparum malaria
D. Severe Plasmodium knowlesi malaria
E. Severe Plasmodium vivax malaria
https://pubmed.ncbi.nlm.nih.gov/22939181/
*Letter B daw kay naa daw trophozoites? not sure :(

BK23 | TABLE OF CONTENTS


38. Indomethacin is FDA approved for closure of PDA in 41. What stage of malarial development is
premature infants. Successful closure can be Pyrimethamine-Sulfadoxine most effective?
expected in how many percent of neonates: A. Gametocytes
A. 20% B. Asexual blood stage
B. 50% C. Latent liver stages
C. 70% D. Primary liver stages
D. 100% Rationale: Goodman and Gilman 13th page 970

GG page 697

42. Which of the following is NOT an effect of the


39. This is the best drug to lower serum urate in Parathyroid hormone?
overproducers and urate stone formers A. It enhances calcium reabsorption
A. Febuxostat B. It inhibits tubular reabsorption of phosphate
B. Probenecid C. It increases bone turnover
C. Colchicine D. It stimulates conversion of Vitamin D into its
D. Allopurinol active form
Rationale: Goodman and Gilman 12th page 998 Rationale: Goodman and Gilman Figure 48-3
Among the choices, only Probenecid is a uricosuric agent.

43. What malarial specie causes glomerulonephritis


syndrome?
A. P. falciparum
B. P. vivax
40. Hypercalcemia from malignancy is best treated with C. P. ovale
this drug when treatment of the underlying D. P. malariae
malignancy is not possible Rationale: Goodman and Gilman 13th page 970
A. Calcitonin
B. Corticosteroids
C. Bisphosphonates
D. Plicamycin
Rationale: Goodman and Gilman 12th page 1232

BK23 | TABLE OF CONTENTS


44. Which of the following malaria patients can receive 46. Which of the following is NOT an early manifestation
chloroquine treatment? of Colchicine toxicity?
A. Malaria infected patients with myasthenia A. Nausea
gravis B. Diarrhea
B. Malaria infected patients with creatinine C. Bone marrow suppression
clearance of 40 D. Abdominal pain
C. Malaria infected patients with upper GI Goodman and Gilman’s, 13th ed. Pg. 703
bleeding and with platelet count of
80,000/mm3
D. Malaria infected patients with psoriatic
lesions but compliant to maintenance
medications
Rationale: Goodman and Gilman 13th page 976

47. Piperaquine has the longest half-life among ACT


partner drugs. This characteristic prevents
reinfection even after the drug has been
discontinued. What is plasma t ½ of piperaquine?
A. 2 weeks
B. 4 weeks
C. 5 weeks
D. 8 weeks
45. The clinical manifestations of malaria coincides with:
A. Sporozoites infecting individual hepatocytes
B. Asexual replication in the liver cell forming
schizonts
C. Merozoites infecting red blood cells
D. Merozoites developing into gametocytes
Goodman and Gilman 13th page 971

48. The uricosuric action of probenecid is blunted by the


co-administration of this drug:
A. Theophylline
B. Aspirin
C. Furosemide
D. Ampicillin
Goodman and Gilman’s 13th ed. pg 695

BK23 | TABLE OF CONTENTS


49. A 40 year old construction worker went to the
Emergency Room for acute, severe pain (pain score
7/10) on his left wrist, left second
metacarpophalangeal and right ankle joints started
12 hours ago. He was diagnosed with gouty arthritis
5 years ago and has been on xanthine oxidase
inhibitor (XOI). How should this patient be best
managed?
A. Start Colchicine within 24 hours of acute
gout attack onset and continue XOI
medication
B. Start mono therapy with systemic
corticosteroids since the severity
assessment is severe
C. Get serum uric acid and X-ray of the
affected joints
D. Refer to specialist

50. A 28 year old journalist was in your clinic to inquire


on chemoprophylaxis for malaria because their crew
is set to explore Palawan in 2 weeks’ time. She is
single, non-pregnant, no known allergies or known
medical problems. Which anti-malarial agent will you
LEAST likely prescribe?
A. Atovaquone-Proguanil
B. Chloroquine
C. Doxycycline
D. Mefloquine

GG 13th ed chap 53 pg 976

BK23 | TABLE OF CONTENTS


Pharmacology BME 3 4. Which of the following is a correct antibiotic regimen
for C-UTI?
1. What antiprotozoal agent would give off a greenish
A. Co amoxiclav 625 mg twice daily for 7
tint to the urine?
days
B. Levofloxacin 750mg once a day for 7 days
A. Metronidazole
C. Ertapenem 1 gram once a day
B. Nitazoxanide
D. Amikacin 15mg once a day
C. Paromomycin Rationale: CPG UTI
D. Rifampicin
Rationale: page 994 Goodman and Gilman 13th Ed

2. What anti-malarial agent can you give to a pregnant


woman in her first trimester who is infected with
malaria?

A. Artesunate + Doxycycline
B. Artesunate + Clindamycin
C. Quinine + Clindamycin
D. Quinine + Tetracycline 5. Which of the following is contraindicated for
Rationale: Malaria guidelines chapter 5, pg. 49 nitrofurantoin use?

A. Uncomplicated cystitis
B. Creatinine of 0.9mg/dL (Normal pa ni)
C. Both
D. Neither
Rationale: GG 13th ed. p. 1018 and 1020

3. What is the therapeutic range of free Quinine?

A. 0.1 - 1.0 mg/L


B. 0.2 - 2.0 mg/L
C. 0.3 - 3.0 mg/L
D. 0.4 - 4.0 mg/L

BK23 | TABLE OF CONTENTS


6. This is the best agent producing amnesia 8. Mrs. Chiu, 45 yo, a known diabetic with good sugar
control. She has a history of renal calculi on
A. Desflurane ultrasound 1 year ago. She visited your clinic for
B. Propofol following up and claims she felt better. She was
C. Midazolam treated for UTI 2 weeks ago and was able to
D. Ketamine complete 1 week course of antibiotic. Repeat urine
Rationale: GG, p. 402 or Katzung, p. 393 culture showed the same organism pending
sensitivity test. What is the next best thing to do?

A. Refer to urologist
B. Advise cystoscopy
C. Advise CT stonogram (an imaging to give
detailed findings of the kidney and
urinary bladder)
D. Start empiric treatment

7. This drug is given to refractory cases of Paget’s 9. Goals of General Anesthesia do not include:
disease
A. Amnesia
A. Etidronate B. Muscle relaxation
B. Calcitonin C. Hypnosis
C. Mithramycin D. Nerve blockade
D. Cinacalet
Rationale: GG 13th ed, pp. 898-900

10. Which of the following is the least likely adverse


effect of sulfonamides?

A. Hemolytic anemia
B. Drug fever
C. Petechial rashes
D. Liver failure
*Calcitonin is also used for the treatment of Paget’s disease and is Rationale: Brunton, et. al - Goodman & Gilman’s 13th ed, pg 1013-1014
effective for up to 6 h in the initial treatment of hypercalcemia, however
patients become refractory after a few days which is likely due to
receptor downregulation.

•Also, use of calcitonin does not substitute for aggressive fluid


resuscitation, and the bisphosphonates are the preferred agents.

BK23 | TABLE OF CONTENTS


11. What enzyme converts arachidonic acid to PGG2 15. What antimalarial agent is effective for malarial
and PGH2? relapse due to latent tissue forms?

A. hospholipase C A. Artesunate
B. Glutathione transferase B. Chloroquine
C. PGH2 synthase C. Primaquine
D. Lipooxygenase D. Quinine
Rationale: p. 687 Goodman and Gilman 13th Edition Rationale:
Primaquine, an eight-amino quinoline that is effective against primary and
latent liver stages as well as gametocytes. Primaquine is used most
commonly to eradicate the intrahepatic hypnozoites of P. vivax and P. ovale that
are responsible for relapsing infections. (GG 13th Ed. pg. 970)

16. Which of the following quinolones is part of the


alternative regimen for MDR TB and atypical MAC
infection?
12. Which antimalarial agent is contraindicated in
pregnant women? A. Moxifloxacin
B. Ciprofloxacin
C. Levofloxacin
A. Clindamycin
D. Gatifloxacin
B. Hydroxychloroquine Rationale:
C. Tetracycline Currently, fluoroquinolones, especially moxifloxacin, continue to be central for
D. Quinine the treatment of pulmonary MDR-TB, and their presence or absence correlates
Rationale: Malaria guidelines chapter 5, pg. 51 with clinical outcomes. (GG 13th Ed. pg. 1076)

17. TRUE of a Hormetic substance:

A. Low dose is not beneficial


B. Presents a sigmoid shape dose-response
curve
C. High dose leads to toxicity
D. All of the above
Rationale:
Some drugs cause low-dose stimulation and high-dose inhibition. Such U-
shaped relationships are said to display hormesis. Several drug-receptor
systems can display this property (e.g., prostaglandins, endothelin, and
purinergic and serotonergic agonists), which may be at the root of some drug
13. Which of the following is a FALSE statement toxicities. (GG 13th Ed. pg. 34)
regarding Ibandronate?
18. What is a side effect associated with the use of
A. Given as 3 mg IV every 3 months Iodoquinone?
B. Reduces vertebral and non-vertebral
fractures A. Anemia
C. Increases BMD of the lumbar spine B. Hypersensitivity
D. Marketed as Boniva C. Hair loss
D. Vision loss
Rationale:
14. This is the antidote for malignant hyperthermia
When used at appropriate doses (never to exceed 2 g/d) for short periods of time
(not greater than 20 days in adults), adverse effects are unusual. However, using
A. Paracetamol these drugs at high doses for long periods carries significant risk. The most
important toxic reaction, ascribed primarily to clioquinol, is subacute myelo-
B. Dantrolene optic. (GG 13th Ed. pg. 990)
C. Cold bath
D. Neostigmine 19. The ANTICHOLINERGIC toxidrome include:
Rationale:
While dantrolene also is used to manage the syndrome of malignant A. Mydriasis, fever, tachycardia, profuse
hyperthermia induced by general anesthetics, the neuroleptic-induced form of sweating
hyperthermia probably is not associated with a defect in Ca2+ metabolism in
skeletal muscle. (GG 13th Ed. pg. 291)
B. Delirium, mydriasis, blindness, dryness
C. Diarrhea, miosis, lacrimation, bradycardia
D. Flushed skin, miosis, blindness, dryness

BK23 | TABLE OF CONTENTS


23. What is the drug of choice for hypnozoites?

A. Artesunate
B. Chloroquine
C. Primaquine
D. Quinidine

20. Gouty arthritis is caused by deposition of:

A. Calcium pyrophosphate crystals


B. Monosodium urate crystals
C. Amorphous urate crystals
D. Cholesterol crystals
Rationale: Gout results from the precipitation of urate crystals in the tissues and
the subsequent inflammatory response… Urate tends to crystallize as Primaquine acts against primary and latent hepatic stages of Plasmodium spp.
monosodium urate in colder or more acidic conditions. (GG 13th ed. p. 702) and prevents relapses in P. vivax and P. ovale infections. This drug and other 8-
aminoquinolines also display gametocytocidal activity against P. falciparum and
other Plasmodium species. (GG 13th ed. P. 974, 980)
21. Risk factors for Allopurinol Hypersensitivity
syndrome (AHS) include the following:
24. At the PACU, a patient who underwent exploratory
laparotomy under general anesthesia complains of
A. Concurrent thiazide diuretic use
generalized muscle pains. Which muscle relaxant
B. Poorly controlled hyperuricemia was most likely given?
C. Previous uric acid stones
D. Previous history of allopurinol allergy
Hypersensitivity reactions have been reported in patients with compromised
A. Rocuronium
renal function, especially those who are receiving a combination of allopurinol B. Vecuronium
and a thiazide diuretic. (GG 13th ed. p. 704) C. Atracurium
D. Succinylcholine
22. A 50 year old woman diagnosed with breast cancer
sought consult for hot flushes. She is non-
hypertensive and non-diabetic. She is taking
Tamoxifen (an estrogen receptor antagonist) for her
breast cancer. What medication can you administer?

A. Estrogen
B. Zoledronate
C. Teriparatide
D. Denosumab

25. Mrs A, 30 yo, on third trimester of pregnancy with no


comorbidities. U/A showed WBC 2-5/hpf, RBC 0-
1/hpf, bacteria-many, epithelial cells- none, crystals-
none. Two urine cultures showed bacterial isolates
>100,000 CFU/ml. No symptoms on further
sciencedirect.com investigation. Which of the following antibiotic is
appropriate?

A. Cefuroxime
B. TMP-SMX
C. Ciprofloxacin
D. Amoxicillin

BK23 | TABLE OF CONTENTS


Source: GG 13th ed page 900

29. This is the most serious side effect of Allopurinol

Google A. Toxic epidermal necrolysis


B. Pruritus
26. What is the antimalarial drug of choice for malaria C. Hepatotoxicity
acquired in the Philippines? D. Dermatitis

A. Chloroquine
B. Clindamycin
C. Artemether Lumefantrine
D. Primaquine Source: GG 13th ed page 704
Rationale:
Artemisinins are highly effective for the first-line treatment of malaria when
combined with other antimalarials. (GG 13th ed. p. 973, 982) 30. Which of the following drugs would be effective for
the primary liver stage of the malarial parasite?

A. Artemether Lumefantrine
B. Atovaquone Proguanil
C. Hydroxychloroquine
D. Quinine

Source: GG 13th ed page 974

31. Mr A, 45 yo, diabetic with diabetic kidney disease,


was diagnosed with CAP (S. pneumoniae isolate).
Which of the following is a safer quinolone for the
27. Ben, 25 y.o, sought consult at the OPD due to penile patient?
discharge. You're planning to start Ceftriaxone and
Azithromycin, but patient claimed he is allergic to the A. Ciprofloxacin
latter. Which of the following would you start?
B. Levofloxacin
C. Moxifloxacin
A. Moxifloxacin D. Norfloxacin
B. Ciprofloxacin
C. Gatifloxacin
D. Ofloxacin Source: GG 13th ed page 1017

32. This is a side effect of Pamidronate when infused


Source: GG 13th ed page 1017
rapidly

28. This is considered the safe fluoride content in water A. Flushing


that decreases incidence of dental caries: B. Renal toxicity
C. Tinnitus
D. Hypotension
A. 0.7 ppm
B. 1.0 ppm
C. 4 ppm
D. 12 ppm

BK23 | TABLE OF CONTENTS


37. Which of the following is the cornerstone in the
management of gout?
Source: GG 13th ed page 898
A. Patient education
B. Low purine diet
33. Recurrent UTI is diagnosed thru:
C. Urate lowering drugs
D. Non-steroidal anti-inflammatory drugs
A. Urinalysis Colchicine, NSAIDS, or glucocorticoids are preferred first line therapy.
B. Urine culture and sensitivity testing Source: ACR Guideline for gout 2020 page 10

C. Both
D. Neither 38. Which of the following conditions is NOT associated
Recurrent UTI is diagnosed when a healthy non-pregnant woman with no known with the use of Pentamidine?
urinary tract abnormalities has 3 or more episodes of acute uncomplicated
cystitis documented by urine culture during a 12-month period OR 2 or more
episodes in a 6- month period. Source: CPG for UTI 2015 page 17 A. Hypoglycemia
B. Nephrotoxicity
34. An immunologically mediated undesired toxic effect C. Risk of acquiring other infections
brought about by previous sensitization of an D. Sterile abscess
side effects of pentamidine include hypoglycemia (paradoxically,
individual to the drug.
hyperglycemia as well), and it is also nephrotoxic. If administered in IM route, it
is associated with sterile abscess at injection sites which can become infected
A. Tolerance secondarily.
Not sure what’s the answer.
B. Allergic reaction Source: GG 13th ed pp. 994-995
C. Idiosyncratic reaction
D. Idiopathic reaction 39. Ben, 25 yo, HIV positive, was started on twice daily
An allergy is an adverse reaction, mediated by the immune system, that results
from previous sensitization to a particular chemical or to one that is structurally TMP SMX by his doctor in the province. Which
similar. Source: GG 13th ed page 57 following lab test will you order to assess for TMP
SMX toxicity?
35. Used for inhalation induction because it is non-
pungent A. CBC
B. Potassium
A. Desflurane C. Both
B. Isoflurane D. Neither
C. Enflurane Toxicity of TMP-SMX includes hematologic reactions. Hyperkalemia can also be
observed as TMP has similar structure to K+ sparing diuretics. Source: GG 13th
D. Sevoflurane ed page 1015
Sevoflurane has properties that make it an ideal induction agent: pleasant
smell, rapid onset, and lack of irritation to the airway. Thus, it has largely
replaced halothane (not available in the U.S.) as the preferred agent for 40. In overdose stage of general anaesthesia, what
anesthetic induction in adult and pediatric patients. Source: GG 13th ed page 394
organ is important to evaluate and monitor?

36. Alice, 25 yo with no comorbids. On her annual PE, A. Cardiovascular


her urinalysis showed microscopic hematuria of 5-10 B. Renal
per hpf. Upon review all her previous urinalysis were
C. Hematopoetic
normal and no urinary changes noted. Does this
D. Liver
patient need KUB ultrasound to investigate for Without circulatory and respiratory support, death would rapidly ensue.
urologic abnormalities?
41. Which of the following would be candidate for the
A. Yes treatment of Chagas disease using NECT?
B. No
Imaging of the urinary tract is warranted whenever anatomic or structural
abnormalities are suspected as contributing to a UTI. Such cases would include A. 75 year old man
(a) pyelonephritis that is not responding to usual treatment, (b) severe B. Advanced heart failure / cardiomyopathy
pyelonephritis in certain high risk groups (e.g. DM), and (c) recurrent UTI in a C. Megaesophagus
man. The patient has none of the factors mentioned. Source: CPG for UTI 2015
page 46
D. HIV patient
The current recommendations are the patients less than 50 years of age with
either acute or recent chronic-phase disease, without advanced cardiomyopathy,
should be treated. Therapy is strongly encouraged for patients who will receive
immunosuppressive therapy or who are HIV positive. (p. 993 GG)

BK23 | TABLE OF CONTENTS


42. What will you advise women who will be on 45. This sedative is very painful upon injection and can
Miltefosine therapy? cause the most incidence of hypotension of the
induction agents
A. Birth control during treatment
B. Birth control during treatment until 2 weeks A. Desfluorane
after treatment B. Propofol
C. Birth control during treatment until 5 C. Midazolam
months after treatment D. Ketamine
D. Birth control during treatment until 1 year PROPOFOL
▪ Short effect-site equilibration time (fast onset)
after treatment
GG p.993 ▪ Wears of within 10 to 15 minutes (rapid metabolism)
▪ Painful upon injection
▪ Can cause most hypotension of all induction agents
through
vasodilation and myocardial infarction
▪ MOA: decrease the rate of dissociation of GABA from the
recepto

46. Herbal medication claimed to be effective against


43. Which of the following best describes TMP SMX?
hypertension:

A. Most MRSA strains are susceptible A. Saw palmetto


B. All strains of E coli are resistant B. Garlic
C. Susceptibility to Proteus and Pseudomonas C. Ginseng
remains high D. All of the above
D. Most S pneumoniae strains are resistant
GG p. 1015

44. Herbal medication/s that increased the risk for


bleeding when combined with warfarin:

A. Garlic
B. Ginseng
C. Gingko biloba
D. All of the above

47. What is the approximate daily dose of Vitamin D that


is considered excessive?

A. More than 50 units / day


B. More than 500 units / day
C. More than 5,000 units / day
D. More than 50,000 units / day
Hypervitaminosis D The acute or long-term administration of excessive amounts
of vitamin D or enhanced responsiveness to normal amounts of the vitamin leads
to derangements in calcium metabolism. In adults, hypervitaminosis D results
from overtreatment of hypoparathyroidism and from faddist use of excessive
doses. The amount of vitamin D necessary to cause hypervitaminosis varies
widely. As a rough approximation, continued daily ingestion of 50,000 units or
more may result in poisoning. The initial signs and symptoms of vitamin D toxicity
are those associated with hypercalcemia.

BK23 | TABLE OF CONTENTS


48. Which of the following statements is INCORRECT? 53. Which of the following malaria chemoprophylaxis
should be taken on a daily basis while in a malaria
A. Chloroquine is the drug of choice for endemic region?
infections due to P. malariae
B. Chloroquine is extensively sequestered in A. Chloroquine
tissues B. Doxycycline
C. Chloroquine enhances the efficacy of C. Hydroxychloroquine
Yellow Fever Vaccine D. Mefloquine
D. chloroquine can cause irreversible G&G 13E p.977

retinopathy
It attenuates the efficacy of the YELLOW FEVER VACCINE when administered
at the same time.

49. A 30 year old presented with a history of tonic clonic


seizure and complains of tingling sensation of the
hands and lips, (+) Trousseu and (+) Chvostek sign.
What is your consideration?

A. Hypocalcemia
B. Hypercalcemia
C. Hypophosphatemia
D. Vitamin D deficiency
Trousseau sign is more specific than Chvostek sign for latent tetany, which can
be caused by hypocalcemia, hypomagnesemia and metabolic alkalosis. 54. What is the approved drug for the treatment of
cryptosporidiosis in immunocompromised adults?
50. What is the half-life of Melarsoprol?
A. Azithromycin
A. 12 hours B. Miltefosine
B. 24 hours C. Nitazoxanide
C. 36 hours D. Sulfadiazine
D. 43 hours G&G 13E p.988

51. Which of the following statements is a


pharmacologic characteristic of Melarsoprol?

A. It can cause fatal encephalopathy


B. It is often coadministered with Prednisolone
C. It is used to treat late stage East African 55. What is the drug of choice for the treatment of
sleeping sickness Trichomoniasis?
D. All of the above
page 991, Goodman and Gilman’s 13th edition A. Folinic acid
B. Metronidazole
- Despite the fact that it causes an often-fatal encephalopathy in 2%–10%
of the patients treated with it, melarsoprol is the only drug for the C. Nitazoxanide
treatment of late (CNS) stages of East African trypanosomiasis caused
by T. brucei rhodesiense.
D. Paromomycin
- Concurrent administration of prednisolone is frequently employed G&G 13E p.988
throughout the treatment course to reduce the prevalence of
encephalopathy.

52. Most common side effect of ketorolac after IM 56. He introduced labor analgesia using chlorophorm on
administration: Queen Victoria

A. Pain at injection site A. John Snow


B. Somnolence B. Ned Stark
C. Pruritus
C. Frances Burney
D. Headache
Side effects of systemic ketorolac include somnolence (6%), dizziness (7%), D. Tyrion Lannister
headache (17%), GI pain (13%), dyspepsia (12%), nausea (12%), and pain at Doc Chan lecture slides
the site of injection (2%). G&G p. 698

BK23 | TABLE OF CONTENTS


57. Fosfomycin is active against the following
microorganism:

A. Proteus
B. Pseudomonas
C. Acinetobacter
D. All of the above
G&G 13E p.1020
60. Which of the following characteristics is common to
all quinolones?

A. Long half life


58. Procedure/s to minimize the absorption of poison: B. Good oral bioavailability
C. Potent agent for Pneumocystis
A. Gastric lavage D. Recommended agents for UTI
G&G 13E p.1017
B. Urine manipulation
C. Multiple-dose activated charcoal
administration
D. All of the above
G&G 13E p.62

61. BB, 35 yo, non-pregnant with uncontrolled asthma,


sought consult at your clinic due to fever and chills.
Upon PE, breath sounds are clear. KPS (+).
Urinalysis showed: wbc 15-20/hpf, rbc 5-10/hpf,
epith cells- few, bacteria – few. What is the
diagnosis?

A. Acute uncomplicated cystitis


B. Acute pyelonephritis
C. Complicated UTI
D. Asymptomatic bacteriuria

59. Which of the following agents is proven effective as


continuous prophylaxis?

A. Cefaclor
B. TMP SMX
C. Both
D. Neither 62. Herbal medication/s indicated for respiratory
Philippine Clinical Practice Guidelines on UTI 2015 Update: Part 2
illnesses:

A. Ephedra
B. Lagundi
C. Echinacea
D. All of the above

BK23 | TABLE OF CONTENTS


67. What is the duration of treatment when administering
artemether Lumefantrine?

A. 48 hours
B. 3 days
C. 7 days
D. 2 weeks
A 3-day treatment schedule with a total of 6 oral doses is
recommended for both adult and pediatric patients based on weight. -
GG p.978

68. What malarial species is associated with


63. Local anesthetics act on which channel glomerulonephritis?

A. Acetylcholine receptor channels A. Plasmodium falciparum


B. Na+ Channels B. Plasmodium knowlesi
C. Calcium channels C. Plasmodium malariae
D. GABA receptor channels D. Plasmodium ovale
Rationale: An uncommon but potentially fatal complication of
P.malariae is a glomerulonephritis syndrome that does not respond to
antimalarial treatment. - GG p. 970

69. Supplement that is derived from crustaceans:

64. Biotransformation of a compound to its harmful A. CoQ


product occurs at this stage in the development of B. Melatonin
toxicity: C. Glucosamine
D. Comfrey
A. Delivery Rationale: Glucosamine is commercially derived from crabs and other
B. Alteration of biological environment crustaceans. - Katzung p. 1142

C. Cellular dysfunction
70. Which of the following conditions need a special
D. Inappropriate repair and adaptation
precaution to quinolone use?
65. Which of the following statements would be
A. History of stroke
associated with the use of NECT for Chagas
B. Diabetic neuropathy
Disease?
C. MDR-TB (multidrug resistant TB)
D. History of bone fracture
A. It should be given 30minutes before a meal
Rationale: In animal models, fluoroquinolones may damage growing
B. It could be administered anytime during the cartilage and cause arthropathy. Thus, these drugs have not been
course of infection recommended as first-line agents for patients under 18 years of age. -
C. It is associated with bone marrow Katzung p. 839
suppression
D. it should be administered for at least 14 days 71. Which of the following statements is NOT associated
NECT toxicity (Chagas disease): urticarial dermatitis, bone marrow with Chloroquine?
suppression, GI symptoms and peripheral neuropathy. - Doc
Tambago’s ppt A. It has a low therapeutic index
B. It should not be used with mefloquine
66. TRUE of Herbal Medications: C. It can exacerbate psoriasis
D. It is an inducer of CYP2D6
A. Able to cure a wide variety of diseases
B. Contain excipients and other chemically-
defined active substances
C. Classified as Food rather than a Drug
D. All of the above
Rationale:
Herbal medicines: categorized as “Food” by FDA, commonly used for
chronic conditions rather than acute illnesses, and allowed to be
marketed without proof of safety and efficacy. - Doc Bongon’s ppt

BK23 | TABLE OF CONTENTS


74. How will you instruct a patient who wants to take
doxycycline to prevent malaria?

A. Doxycycline 100mg daily taken one day


before travel and for 4 weeks after
leaving the area
B. Doxycycline 100mg daily taken 1 week
before travel and for 1 week after leaving the
area
GG p.976
C. Doxycycline 100mg weekly taken 1-2 weeks
before travel and for 4 weeks after leaving
72. Antidote for iron toxicity
the area
D. Doxycycline 100mg weekly taken 1-2 weeks
A. Pyridoxine
before travel and for 1 week after leaving the
B. Flumazenil area
C. Deferoxamine
D. Thiamine

75. Which of the following conditions would resolve


without treatment?

A. 2 small cutaneous leishmania lesions on the


arm
73. What parasitic infection/s can be treated with B. Single 6cm cutaneous leishmania lesion
Clindamycin? on the cheek
C. Mucocutaneous leishmaniasis
A. Babesiosis D. Visceral leishmaniasis
B. Malaria
C. Toxoplasmosis
D. All of the above

76. Can minimize GI absorption in Iron overdose:

A. Ascorbic acid
B. Egg white
C. Sodium bicarbonate
D. Activated charcoal

BK23 | TABLE OF CONTENTS


80. Urine isolate that warrants screening for urologic
abnormalities:
77. Which of the following is NOT a characteristic of an
unanesthetized patient? A. E. coli
B. Proteus mirabillis
A. Protective reflexes are preserved C. Candida tropicalis
B. Presence of spontaneous respiration D. Klebsiella pneumoniae
C. Intact homoestasis UTI CPG Part 2 2015 p.17
D. Presence of apnea
Stages of Anesthesia Based on Guedel's Classification
https://www.ncbi.nlm.nih.gov/books/NBK557596/

81. Which of the following is/are characteristics of TMP


SMX?

A. Half-life is around 4 hrs


78. Good clinical evaluation of a poisoned suicidal B. Volume of distribution of SMX is nine times
patient should involve: that of TMP
C. Enters the CSF
A. Information about the identity of the toxic D. All of the above
GG p1014
agent
B. Drug history of the patient
C. Laboratory
D. All of the above

79. Principal action of Ketamine is at which receptor site

A. GABA
B. NMDA
C. Mu
D. Neuromuscular junction

BK23 | TABLE OF CONTENTS


82. Pneumocystis jiroveci prophylaxis among HIV
patients is given:

A. Once daily
B. Twice daily
C. Thrice daily
D. Twice a week
Once daily or three times a week. Gg p1015

83. TMP SMX is not indicated in the following


conditions:

A. Local MRSA infection


B. Traveler’s diarrhea
C. Some UTI
D. None of the above

85. What drug is given as an intermittent prophylaxis for


pregnant woman living in malaria endemic places?
84. “Before whom in all time, Surgery was agony” is
A. Doxycycline
written on whose epitaph?
B. Chloroquine
A. William T.G. Morton C. Sulfadoxine Pyrimethamine
B. John Snow D. Mefloquine
Only Chloroquine is safe in pregnancy
C. Frances Burney Drugs safe in pregnancy:
D. Joseph Priestly ● Proguanil
● Quinine
Anes ppt
● Sulfadoxine - pyrimethamine - but contraindicated in lactating mother
John Snow used chloroform to deliver the last 2 children of Queen Victoria
and infants <2 mons
Frances Burney given alcohol and held by 7 men while undergoing a ● Chloroquine & Hydrochloroquine - preferred agents
mastectomy ● Artemether - lumefantrine - only in the 2nd and 3rd trimester
● Quinine - Clindamycin - only in the first tri (based on CPG)

86. Alice, 25 yo, non-pregnant with no comorbidities,


sought consult at the OPD due to dysuria. She has
no fever. Physical exam is unremarkable. Urinalysis:
WBC 10-15/hpf, RBC 0-5/hpf, Epith cells- few,
bacteria- moderate. Which of the following is the
next best thing to do?

A. Do urine culture and sensitivity test


B. Repeat urinalysis
C. Start Cefuroxime 250mg twice daily for 7
days
D. Start Co-amoxiclav 625 mg twice daily for 3
days

BK23 | TABLE OF CONTENTS


88. Which of the following conditions CANNOT be
UTI CPG Part 1 2013 p7 treated with Metronidazole?
Diagnosis: Acute Uncomplicated Cystitis (AUC)

A. Giardiasis
B. Liver abscess
C. Toxoplasmosis
D. Trichomoniasis
GG p991

The combination of pyrimethamine and sulfadiazine is the treatment of choice for


toxoplasmosis. (GG P.1013)

89. A 30-year-old mentally-ill man was brought to the


emergency room. He allegedly ingested 30 tablets of
his maintenance antipsychotic medication half an
hour ago. Which treatment is/are correct in the
emergency stabilization of the patient?

A. Intubate the patient and attach to


mechanical ventilator if breathing is
compromised
B. Intravenous administration of thiamine if the
patient is seizing
C. Give naloxone if the patient is comatose
D. All of the above
GG pg. 60 *Not sure*
Naloxone is used to rapidly reverse opioid overdose.
87. Which of the following conditions is Mefloquine When toxicity is expected or occurs, priorities of poisoning treatment are to
Maintain vital physiologic function; Reduce or prevent absorption and enhance
contraindicated? elimination to minimize the tissue concentration of the poison; Combat the
toxicological effects
A. History of Quinine allergy
B. Ongoing treatment for depression 90. Mr Chui, 65 y.o with heart failure is on chronic
C. Heart failure with cardiac conduction prolonged QT prolongation on ECG. He developed
abnormalities UTI. Which of the quinolones is the safest for the
D. A and C patient?
E. All of the above
GG page 979 A. Moxifloxacin
B. Ciprofloxacin
C. Levofloxacin
D. Gatifloxacin
GG. page 1017 under other adverse effects
Moxifloxacin does not accumulate in the urine and is not recommended
for UTI & it also carries the highest risk for QT interval prolongation &
torsades de pointes.
Gemifloxacin, levofloxacin, & ofloxacin have lower risk.
Ciprofloxacin has the lowest risk.

BK23 | TABLE OF CONTENTS


91. What can occur with prolonged oral intake of 94. Mrs Sy, 40 yo, was started on nitrofurantoin for
Eflornitine? recurrent UTI. Within 24 hrs, she then developed
fever, chills and cough. What would be the next best
A. Cinchonism thing to do?
B. Hearing loss
C. Peripheral neuropathy A. Give anti-histamine
D. Optic atrophy B. Give antibiotic
GG 13th Ed page 990 C. Discontinue nitrofurantoin
D. All of the above
GG 13th Ed page 1018

92. Atracurium undergoes metabolism through

A. Plasma cholinesterase
B. Pseudocholinesterase
C. CYP450 95. Your Filipino friend is planning to do volunteer work
D. Hoffman degradation in Africa, what is the best anti-malarial
GG 13th Ed page 990 chemoprophylaxis to prescribe?

A. Artesunate
B. Chloroquine
C. Mefloquine
D. Quinine
Goodman & Gillman’s, 13th ed., page 981

93. The following is the correct order of the physiologic


stages of general anaesthesia:

A. Surgical, Overdose, Sedation, Excitation


B. Sedation, Overdose, Excitation, Surgical
C. Sedation, Excitation, Surgical, Overdose
D. Excitation, Sedation, Surgical, Overdose

96. Approved as treatment for patent ductus arteriosus


in premature infants:

A. Oral indomethacin
B. IV indomethacin
C. IV and oral has proven same beneficial
effects
D. None of the above
Goodman & Gillman’s 13th ed., page 697

BK23 | TABLE OF CONTENTS


Philippine Clinical Practice Guidelines on the Diagnosis and Management of
Urinary Tract Infections in Adults 2015 Update: Part 2

97. Mrs A, 55 yo, diabetic with poor sugar control went


to her doctor for her annual lab work up. She has no 99. Which of the following NSAIDS is largely excreted by
current complains. PE is unremarkable except for the kidneys?
HR of 110 bpm.
CBC A. Diclofenac
Wbc – 20,000/mm3 B. Salicylates
Segmenters – 80% C. Ibuprofen
Lymphocytes – 20% D. Naproxen
Plt 250,000 Naproxen StatPearls NCBI
Hgb 11
Hct 33
Urine culture – gm (-) bacilli >100,000 CFU/ml
Urinalysis: WBC 5-10/hpf, RBC 0-1 /hpf, epithelial cells
100. Which of the following defines an asymptomatic
– many, bacteria, moderate
bacteriuria?

What is the diagnosis?


A. In women, bacteriuria in 2 consecutive urine
samples with >100,000 CFU/ml colony count
A. Asymptomatic bacteriuria of 2 different isolates
B. Acute uncomplicated cystitis B. In men, a single catheter derived urine
C. Acute pyelonephritis sample with bacterial isolate >100
D. Complicated UTI CFU/ml
Patient is not having any symptoms which makes this case more likely to be an
asymptomatic bacteriuria C. In women, a clean catch voided urine with
bacterial isolate of >100,000 CFU/ml
D. All of the above
Philippine Clinical Practice Guidelines on the Diagnosis and Management
of Urinary Tract Infections in Adults 2015 Update: Part 2

98. Which of the following statement is TRUE?

A. Asymptomatic bacteriuria prior to an elective


urologic procedure may not be treated
B. In emergency procedures, antibiotic may not
be started if bacteriuria is noted
C. If asymptomatic bacteriuria is noted prior to
an elective urologic procedure, defer
procedure temporarily and repeat urinalysis
and culture
D. Bacteremia develops in 80% of patients
with traumatic urologic procedure if no
treatment is done

BK23 | TABLE OF CONTENTS


Pharmacology 4_1 4. Virtually all drugs administered to the mother can
traverse the placenta and appear in the fetal
circulation. The following factors affect the placental
1. Which of the following does NOT have an active role drug transfer EXCEPT:
in the management of depression and anxiety?
A. Rate at which the drug crosses the placenta
A. GABA
B. Duration of drug exposure
B. Dopamine C. Effects of drugs used in combination
C. Norepinephrine D. Presence of placental transport proteins
D. Serotonin
Rationale: Doc Camomot’s ppt on Perinatal and pedia
pharmacology

Rationale: Doc Bueno’s PPT on depression& anxiety

2. One of the statements below is NOT a good practice


of drug compliance in children

A. The parents should obtain a calibrated medicine


spoon or syringe from the pharmacy 5. Core symptoms of Major Depressive Disorder:
B. The parents must be told whether or not to wake
the infant for its every-6-hour dose day or night A. Depressed mood and anhedonia
C. Convenient dosage forms and dosing schedules B. Motor excitation or retardation
should be chosen C. Change in appetite and sleep
D. Children should choose the medication they D. Thoughts of death or suicide
like
Rationale: Dr. Bueno PPT
3. Precursor of serotonin

A. Tyramine
B. Monoamine
C. Tryptophan 6. Maximum allowable dose of metformin:
D. Tyrosine
A. 2000 mg/day
Rationale: B. 2550 mg/day
C. 3500 mg/day
D. 1500 mg/day
Rationale: Brunton, et. al - Goodman & Gilman’s 13th ed, pg. 875

BK23 | TABLE OF CONTENTS


9. Which of the following is considered an exogenous
amine that is linked to the development of
hypertensive crisis in a patient taking MAOI?
7. Which of the following statements regarding
diabetes screening is correct? A. Carotenoids
B. Eicosanoids
A. Routine testing for GDM is recommended at C. Tyramine
3rd trimester of pregnancy D. Tryptophan
B. Universal lab screening is recommended
C. Consider earlier testing if patient has BMI of Rationale:
24
D. All individuals seen in the clinic should
be screened annually

Rationale:

Doc Bueno’s PPT

10. A female suffering from psychosis, injected with


fluphenazine now complains of sudden onset of high
8. Which of the following promotes secretion of insulin?
grade fever, muscle rigidity and altered sensorium.
The diagnosis is:
A. Glucose
B. Ketone bodies
A. Malignant hyperthermia
C. Both
B. Tardive dyskinesia
D. Neither
C. Akathisia
D. Neuroleptic malignant syndrome
Rationale: Goodman and Gilman

BK23 | TABLE OF CONTENTS


Rationale:

14. The relative safety of the majority of commonly used


anti-depressants and anxiolytics notwithstanding
their optimal use requires a clear understanding of
their

Neuroleptic Malignant Syndrome. The rare NMS resembles a severe form of A. Mechanism of action
parkinsonism, with signs of autonomic instability (hyperthermia and labile pulse, B. Differential diagnosis of psychiatric illnesses
blood pressure, and respiration rate), stupor, elevation of creatine kinase in
serum, and sometimes myoglobinemia with potential nephrotoxicity. At its most C. Pharmacokinetics
severe, this syndrome may persist for more than a week after the offending agent D. All of the above
is discontinued and is associated with mortality. (GG 13th ed. p. 290-291)
Rationale:
11. Low cost and sedation are its advantages however
they could cause cardiovascular toxicity

A. MAOIs
B. SSRIs 15. Least extrapyramidal side effects are seen with:
C. TCAs
D. SNRIs A. Clozapine
B. Fluphenazine
C. Haloperidol
D. Chlorpromazine

Rationale:
Dr. Bueno’s PPT (Antipsychotics)
Clozapine
• Extrapyramidal side effects are minimal

16. Mario 32 yo diabetic but non hypertensive. He has


Doc Bueno’s PPT elevated blood cholesterol levels on his annual
testing. Creatinine and liver enzymes are at normal
levels. He claims to be compliant to his metformin
12. Which of the following is an ideal treatment for plus sitagliptin. You started him on atorvastatin
tardive dyskinesia? 20mg once daily and advised to repeat testing after
2-3months. What would be you LDL target?
A. Drug discontinuation
B. Valbenazine A. 70 mg/dl
C. Diphehydramine B. 100 mg/dl
D. Amantadine C. 150 mg/dl
D. 200 mg/dl

Rationale:
Diabetes CPG
Doc’s ppt
In individuals without overt CVD, the primary
13. What type of drug transfer does ketamine belongs goal is an LDL cholesterol <100 mg/dl (2.6
to? mmol/l). (A) In individuals with overt CVD, a
lower LDL cholesterol goal of <70 mg/dl (1.8
A. Complete transfer mmol/l), using a high dose of a statin, is an
B. Exceeding transfer option. (B).
C. Incomplete transfer

BK23 | TABLE OF CONTENTS


17. Drugs that are low in extrapyramidal side effects and 20. Mr A was rushed to the emergency room due to
at least equal in efficacy for positive symptoms and unresponsiveness. He is a known diabetic and on
better for negative symptoms insulin therapy. Hgt at ER was 45 mg/dl. You
ordered glucagon to be given to the patient. What is
A. Olanzapine the ideal route of administration of glucagon?
B. Chlorpromazine
C. Haloperidol A. Per NGT
D. Biperiden B. IV
Rationale: C. SC
The newer antipsychotic drug, classified as “atypical” and D. IM
characterized byt heir low propensity for inducing
extrapyramidal side effects, also includes agentss that are
potent muscarinic receptor antagonists. In particular
Rationale:
CLOZAPINE binds to human brain muscarinic receptor with IV, IM, SC are the available routes for glucagon.
high affinity and OLANZAPINE also is a potent muscarinic IM is preferred for emergencies.
receptor antagonist, xerostomia is a prominent side effect of p. 883 GG 13th Ed
this drug.
21. Which of the following statements is
18. What is the route of elimination of Bupropion? pharmacologically correct?

A. Hematologic A. Tricyclic antidepressant is the first line drug


B. Hepatic for treating depression.
C. Renal B. Atypical antipsychotic in combination
D. A and B with SSRI has been used to treat
E. B and C treatment resistant major depression
Rationale: C. Monoamine oxidase inhibitors are commonly
Bupropion elimination has a t 1/2 of 21 h and involves both hepatic and used because of its safety and efficacy
renal routes. Patients with severe hepatic cirrhosis should receive a
profile in the management of depression.
maximum dose of 150 mg every other day; consideration for a
decreased dose should also be made in cases of renal impairment.
D. Selective Serotonin Reuptake Inhibitor
(GG, p.272) inhibits both SERT and NET

19. Which of the following best describes the Rationale:


hyperprolactinemia as an adverse effect of anti-
psychotic medications?

A. Causes breast engorgement and


infertility in women only
B. This condition is reversed after 3 months of
drug discontinuation - immediately reversed
C. Dose reduction is not effective in reversing
the effects of the medication - effective
D. Atypical antipsychotic such as paliperidone
has minimal effect on prolactin secretion -
significant effect
Rationale: 22. What is the side effect associated with the use of
Hyperprolactinemia can directly induce breast engorgement and Olanzapine?
galactorrhea and can cause amenorrhea in women and sexual
dysfunction or infertility in women and men. (GG, p.291)
A. Blood dycrasias
With the exception of risperidone and paliperidone, atypical B. Hypersensitivity reaction
antipsychotic agents show limited effects (asenapine, iloperidone, C. Hypertensive crisis
olanzapine, quetiapine, ziprasidone) to almost no effects (clozapine, D. Metabolic syndrome
aripiprazole, brexpiprazole, cariprazine) on prolactin secretion.:
Rationale:
olanzapine is an atypical antipsychotic drug. The major risks of these agents are
weight gain and metabolic syndrome, a greater problem for quetiapine and
olanzapine than for aripiprazole

BK23 | TABLE OF CONTENTS


23. Which of the following is NOT a function of the 26. Which of the following is considered a side effect of
placenta? SSRIs?

A. Site of gas exchange and transfer of A. Dry mouth


nutrients B. Cardiac conduction abnormalities
B. Transfer of immunity C. Insomnia
C. Secretes oxytocin during parturition D. Sedation
D. Facilitate supply of oxygen to the fetus Rationale:
The human placenta is a complex organ that acts as the interface between the Adverse side effects of SSRIs from excessive stimulation of brain 5HT2 receptors
mother and the fetus. Its functions are: may result in insomnia, increased anxiety, irritability, and decreased libido,
1.Gas exchange and the transfer of nutrients and waste products between effectively worsening prominent depressive symptoms. G&G p.273
maternal and fetal plasma.
2.Transfer of immunity by transfer of immunoglobulins from the mother to the
fetus. 27. Signs and symptoms of hypoglycemia will start to
3. Secretion of hormones which are important for fetal growth and development. occur at what glucose level?

24. BB, 28 yo, sought consult in your clinic due to A. <50 mg/dl
gradual weight loss. No fever nor coughing. Appetite B. <70mg/dl
was fair. He has a family history for both C. <90mg/dl
hypertension and DM. Vital signs are normal with D. <100mg/dl
BMI of 22. Other PE findings are normal. Labs were Symptoms of hypoglycemia are seen at glucose level of 60-70 mg/dL or 3.3-3.9
mM and the symptoms are sweating, hunger, paresthesias, palpitations, tremor,
normal except for FBS of 110 mg/d and triglycerides
and anxiety.
of 300 mg/dl. What is your diagnosis at this time?
28. What is considered the “therapeutic lag” of an
A. Diabetes insipidus antidepressant once initiated to a patient?
B. DM type 1
C. DM type 2
A. 1 to 2 weeks
D. Impaired Glucose Tolerance
B. 2 to 3 weeks
C. 3 to 4 weeks
D. 4 to 5 weeks
Antidepressants target our DNA, specifically genes that encode serotonin
transporter. They make these genes less active which results in the delay.

29. Mr Chiu 50 yo with no known comorbids went to


your clinic for check-up. No unusual signs and
symptoms noted except for increased daytime
sleepiness. BP 120/90, BMI 25. Patient don’t want to
fast thus ordered HBA1C which was 11% (DCCT
25. Newer ‘atypical’ anti-psychotic drugs are less certified). ECG – normal sinus rhythm. What would
inclined to produce these effects - possible due to be the ideal regimen for the patient?
their greater affinity for this pathway over the striatal
areas of the brain A. Metformin, being the 1st line of treatment
B. Insulin therapy with or without oral
A. Mesolimbic pathway hypoglycemic agents
B. Mesocortical pathway C. Metformin + SGLT2 inhibitor combination
C. Nigrostriatal pathway D. Metformin + DPP4 inhibitor combination
D. Tuberoinfundibular pathway Rationale: This regimen is better than metformin alone since the HBA1C
Rationale: page 279 Goodman and Gilman’s 13th edition of the patient is 11%
- The discovery of clozapine’s unique clinical features and binding
profile stimulated development of second-generation antipsychotics
that potently antag-onize the 5HT2A receptor while possessing less
affinity for D2 receptors than typical antipsychotic agents, resulting in
antipsychotic efficacy with lower potential for extrapyramidal side
effects.

- The DA model of antipsychotic action has limitations: It does not


explain the psychotomimetic effects of LSD (e.g., a potent 5HT2A
receptor agonist) or the effects of phencyclidine and ketamine,
antagonists of the NMDA glutamate receptor. However, phencyclidine
and ketamine indirectly act to stimulate DA availability by decreasing
the glutamate-mediated tonic inhibition of DA release in the
mesolimbic DA pathway.

BK23 | TABLE OF CONTENTS


30. Which of the following insulin has the longest 35. Adverse event when MAOI is taken with cheese:
duration of action?
A. Hypertensive crisis
A. Glulisine B. Hypoglycemia
B. Detemir C. Hypotensive crisis
C. Aspart D. Hyperglycemia
D. Degludec Rationale: MAOI fell out of fame due to “cheese reaction” - dietary
Duration of action: tyramine metabolism inhibited by MAOI leading on to hypertensive
Glulisine = 3-4 hrs (short acting) crisis. - Doc Bueno’s ppt
Detemir = 12-20 hrs (long acting)
Aspart = 3-4 hrs (short acting)
Degludec = 24-42 hrs (long acting) 36. This is an alcoholic solution in which the drug
molecules are dissolved and evenly distributed and
31. Which of the following is/are insulin secretagogues? therefore no shaking is required
A. Suspension
A. Empagliflozin B. Elixir
B. Metformin Rationale: powerpoint by Doc Camomot
C. Sitagliptin
D. Pioglitazone
Rationale: Sitagliptin is a DPP4 inhibitors which inhibits degradation of
incretin leading to increased insulin secretion

32. Drug known to have a very narrow therapeutic index

A. Carbamazepine
B. Lamotrigine
C. Lithium
D. Divalproex
Rationale: pls refer to GG 13th edition pg 295

33. Neurotransmitter that influences the brain’s function 37. Which of the following drugs has an effect on
on sleep, appetite and sexual behavior: dopamine transmission?

A. Norepinephrine A. Amoxapine
B. Dopamine B. Bupropion
C. Serotonin C. S-Citalopram
D. All of the above D. Venlafaxine
Rationale: Serotonin influences a wide variety of brain functions:
mood, sleep, cognition, sensory perception, temperature regulation, Rationale:
nociception, appetite and sexual behavior. - Doc Bueno’s ppt

34. Treatment of choice for drug-resistant


Schizophrenia:

A. Olanzapine
B. Clozapine
C. Risperidone
D. Chlorpromazine
Rationale: Aside from clozapine, which is uniquely efficacious in
refractory schizophrenia, atypical antipsychotics are not more effective
than typical agents but offer a better neurological side-effect profile
than typical antipsychotic drugs. - GG p.282

BK23 | TABLE OF CONTENTS


38. Somatic symptoms of Major Depressive Disorder Rationale: GG page 879
includes:

A. Poor concentration
B. Feelings of guilt
C. Change in appetite
D. Suicidal thoughts

42. Which of the following statements regarding


thiazolinediones is TRUE?

A. GI symptoms are common side effects


B. May be given in patients with renal
insufficiency
C. Decreases FBS by 50%
D. Increases pancreatic insulin secretion
Rationale: GG page 876
39. Which of the following antidepressants is classified
as a Serotonin - Norepinephrine Reuptake Inhibitor?

A. Amitryptylline
B. Doxepin
C. Escitalopram
D. Venlafaxine

43. Which of the following differentiates type 1 vs. type 2


DM?

40. Which of the following statements regarding A. Low C peptide level


diabetes is correct? B. Normal C peptide level
C. Weight loss
A. Risk of developing over diabetes D. Exogenous insulin use
increases by 6 fold if both parents are E. A and D
diabetics Rationale: PPD on Diabetes
B. All type 1 DM occurs at a younger age
C. Low BMI is present in 100% of Asian
diabetics
D. All of the above
Rationale: GG pg 868 20th ed
The etiology of type 2 diabetes has a strong genetic component. It is a heritable
condition with a relative 4-fold increased risk of disease for persons having a
diabetic parent or sibling, increasing to 6-fold if both parents have type 2
diabetes.

41. Which of the following oral hypoglycemic agent


offers no cardiovascular benefits to diabetic
individuals?

A. SGLT 2 inhibitors
B. DPP4 inhibitors
C. GLP 1 agonist
D. Biguanides

BK23 | TABLE OF CONTENTS


44. Which of the following drugs will reduce lithium
levels in the body?

A. Hydrochlorothiazide
B. Spironolactone
C. Furosemide
D. Indomethacin 47. Neurotransmitter that modulates the brain’s cognitive
Rationale: GG page 297 and motor function, aggression and drive.
A. Norepinephrine
B. Dopamine
C. Serotonin
D. All of the above

45. Ideal route of administration of GLP 1 agonist: 48. Which of the following statements about depression
A. SC is NOT correct?
B. Oral A. The lifetime risk of depression is 15%.
C. IM B. Females are commonly affected than males
D. IV C. Increased libido is one of its
manifestations
D. It can occur secondary to a chronic illness

49. Which of the following statements is TRUE about


perinatal pharmacology?
A. Highly ionized drugs cross the placenta
readily
B. Water soluble drugs diffuse readily across
the placenta and enter the fetal circulation
C. Drugs with molecular weight >1000 cross
very poorly
D. Warfarin is the anticoagulant of choice in
pregnant patients since the drug molecule is
large and polar

Rationale: Perinatal PPT

46. Glucose crosses the placenta to the fetal circulation


using this mechanism
A. Passive diffusion
B. Facilitated diffusion
C. Active transport
D. Pinocytosis

BK23 | TABLE OF CONTENTS


50. When is the best time to administer lithium?
A. After breakfast
B. After lunch
C. After dinner
D. At bedtime

BK23 | TABLE OF CONTENTS


Pharmacology BME 4

1. What agent is best given to a patient suffering from


postoperative ileus?

A. Atropine
B. Bethanechol
C. Pilocarpine
D. Trospium
GG 13th ed. page 159

4. Which of the following statements is TRUE for


Ipratropium?

A. It selectively blocks M1 and M3 receptors


B. It has a long half-life of 24 hours
C. It is approved for perennial rhinitis
D. It is the mainstay of treatment for bronchial
asthma

5. Which of the following conditions is associated with


hypothyroidism?
2. What is the most toxic organophosphate AChE
inhibitor?
A. Autoimmune disease
B. Partial removal of the thyroid gland
A. Parathion C. Radiation damage
B. Malathion D. A and C
C. Ecothiophate E. B and C
D. Metrifonate Autoimmune- Hashimoto
Katzung, Chapter 7, p. 117 Partial removal still produce thyroxine
Radiation-induced hypothyroidism is a well know side effect of radiation
therapy in most cancer treatment

6. The following are effects of your Benzodiazepines,


EXCEPT:

A. Anxiolysis, sedation
B. Anticonvulsant activity
C. Anterograde amnesia
3. Which of the following drugs would affect thyroid D. Retrograde amnesia
GG 13th ed page 341
hormone metabolism?

A. Carbamazepine
B. Phenobarbital
C. Rifampicin
D. All of the above
E. None
GG 13th, p 796

BK23 | TABLE OF CONTENTS


7. Which of the following lab tests is diganostic of
Diabetes Mellitus?

A. FBS of 100 mg/dl


B. HbA1C of 6.7%
C. Capillary blood sugar of 250 mg/dl
D. Random blood sugar of 15 mmol/L
p.13 United for Diabetes Philippines - Philippine Practice Guidelines

10. The following statement regarding a Preganglionic


neuron is true

A. Small, unmyelinated type B fibers


B. Sympathetic: short myelinated type B
8. A farmer was accidentally exposed to a high Fibers
concentration of an agricultural organophosphate C. Parasympathetic: long type C fiber
insecticide. If untreated, the cause of his death D. Small, myelinated Type C fibers
would be? E. Parasympathetic: short, Type C fibers

A. Cardiac arrhythmia
B. Hypoxia
C. Heart failure
D. Gastrointestinal bleeding
GG 13th ed, chapter 10, page 170
Organophosphates are anticholinesterase agents. Toxicities of such are
seen below.

11. A 28-year old resident doctor in a public hospital is


having an acute asthmatic attack after receiving the
first dose of Sinovac vaccine. Epinephrine is give SC
route. Through which of the following adrenoceptors
does epinephrine act to dilate bronchial smooth
muscle?

9. What muscarinic receptor type is predominantly A. ά1-adrenoceptors


located in the heart? B. ά2-adrenoceptors
C. ß1-adrenoceptors
A. M1 D. ß2-adrenoceptors
GG 13th ed. page 121
B. M2
C. M3
D. M4

BK23 | TABLE OF CONTENTS


12. Which of the following statement/s is TRUE of febrile
seizures?

A. May give anti-seizure prophylaxis - due to


increased risk of developing epilepsy or
other neurological sequelae
B. May not be associated with febrile illness -
2-4% is associated with febrile illness
C. Phenobarbital is the drug of choice -
Diazepam, rectal
D. 10% of patients with family history of seizure
will develop prolonged epilepsy - risk
factors: preexisting neurological disorder or 15. Muscarinic receptors are located in:
developmental delay, family history of
epilepsy, or complicated febrile seizure = A. Autonomic ganglia
10% risk of developing epilepsy B. Skeletal muscle neuromuscular junctions
Goodman and Gilman, p. 322 C. Autonomic effector cells
D. Sensory carotid sinus baroreceptor zone
13. Which of the following agents is a potent competitive
antagonist at both ά1 and ά2 receptors?

A. Phenoxybenzamine
B. Phentolamine
C. Prazosin
D. Phenylpropanolamine 16. In the biosynthesis and metabolism of acetylcholine,
Rationale: GG 13th p 208 & Pubmed
what ion triggers the release of acetylcholine from
the synaptic vesicles?

A. Calcium
B. Potassium
C. Sodium
D. Magnesium

*A & B are both nonselective antagonists but according to pubmed,


phenoxybenzamine antagonizes both a1 and a2 receptors. 17. A 30-year-old patient has moderately severe new
*Prazosin is an a1 selective onset asthma, and you prescribe a highly selective
*Phenylpropanolamine is a sympathomimetic antagonist
β2 agonist inhaler to be used when needed. In
considering the possible drug effects in this patient,
14. Which of the following agents is devoid of nicotinic you would note that β2 stimulants frequently cause:
activity?
A. Direct stimulation of renin release
A. Acetylcholine B. Hypoglycemia
B. Bethanechol C. Itching due to increased cGMP (cyclic
C. Carbachol guanine monophosphate) in mast cells
D. Methacholine D. Skeletal muscle tremor
E. Vasodilation in the skin

BK23 | TABLE OF CONTENTS


20. A 32-year old female ingested 30ml of Propoxur
(Baygon) after a fight with her husband. Upon
examination, which is NOT an expected finding
following intake of this drug?

A. Dry mouth- should be salivating


B. Small, pinpoint pupils
C. Difficulty of breathing
AE: tremors; tachycardia; increase concentration of glucose, lactate D. Tearing
and FFA in plasma; decrease K concentration; myocardial necrosis in
lab animals

18. Full activation of the parasympathetic nervous


system is likely to produce which of the following
effects?

A. Bronchodilation
B. Decreased intestinal motility
C. Increased thermoregulatory sweating
D. Increased pupillary constrictor tone
(miosis)
E. Increased heart rate (tachycardia)

19. A ß blocker was prescribed for hypertension in a


female asthmatic patient. About after a week, the
asthma attacks got worse and she decide to stop
taking the drug. What ß blocker would you shift to
less likely worsen her asthma?
21. What is the best thyroid hormone preparation to
administer to a patient suffering from myxedema
A. Propanolol- non selective coma?
B. Metoprolol- b1 selective blocker
C. Labetalol- non selective b blocker
A. Levothyroxine
D. Carvedilol- non selective b blocker
All other options are non-selective beta blockers
B. Liothyronine
C. Iodine
D. Propylthiouracil

BK23 | TABLE OF CONTENTS


24. Acetylcholine is NOT a specific neurotransmitter at:

A. Sympathetic ganglia
B. Sympathetic postganglionic nerve endings
C. Parasympathetic ganglia
D. Parasympathetic postganglionic nerve
endings

GG, 13th ed page 795

22. Loss of anti convulsant activity of prolonged


clonazepam use is observed:
Dr. Vatanagul’s PPT
A. Within 1 month
B. Within 3 months 25. Which of the following drug groups decreases the
C. Within 6 months rate of glucose reclemation in the proximal tubule?
D. Within 2 years
A. DPP4 inhibitors
B. SGLT2 inhibitors
C. GLP1 receptor agonist
D. Alpha glucosidase inhibitor

GG, 13th ed page 312

23. Which of the following drugs or classes of drugs will


GG, 13th ed page 879
be useful in treating the symptoms of myasthenia
gravis?
26. Folate supplementation for epileptic women of child
A. Nicotinic antagonists bearing age:
B. Muscarinic agonists
C. Muscarinic antagonists A. 1 mg/d
D. Anticholinesterase agents B. 5mg/d
C. 0.4mg/d
D. 4.0 mg/d

GG, 13th ed page 322

27. What ß-blocker agent is safer to prescribe to treat a


hypertensive patient with a known asthma of
history?

A. Propanolol
Dr. Maratas’ PPT
B. Atenolol
C. Pindolol
D. Nadolol

BK23 | TABLE OF CONTENTS


Muscarinic agonists are currently used in the treatment of urinary bladder
disorders and xerostomia and in the diagnosis of bronchial hyperreactivity. They
are also used in ophthalmology as miotic agents and for the treatment of
glaucoma. There is growing interest in the use of M1 agonists in treating the
cognitive impairment associated with Alzheimer disease. Other receptor
subtypes, including M2 and M5, also appear to be involved GG page 152

32. Epinephrine increases blood pressure by which of


the following mechanisms?

A. Indirect myocardial stimulation that


increases the strength of ventricular
contraction
B. An increased heart rate
C. Vasodilation in many vascular beds,
especially in the precapillary resistance
Katzung 14th ed. Page 170 vessels of skin, mucosa, and kidney
D. Decrease in the cutaneous blood flow,
28. A farmer ingests a plant seed and presents with constricting precapillary small venules
signs of an indirect-acting cholinomimetic that readily The mechanism of the rise in blood pressure due to EPI is a triad of effects:
• a direct myocardial stimulation that increases the strength of ventricular
enters the CNS. Which of the following tends to be contraction (positive inotropic action);
the active, toxic ingredient in the seed? • an increased heart rate (positive chronotropic action); and
• vasoconstriction in many vascular beds—especially in the precapillary
resistance vessels of skin, mucosa, and kidney—along with marked
A. Neostigmine constriction
B. Physostigmine of the veins.
C. Muscarine GG 13th ed page 194

D. Edrophonium
Physostigmine is a tertiary amine which rapidly enters the CNS and is more toxic 33. Clinical uses of ά -adrenergic blockers include which
than quaternary amines. Neostigmine and edrophonium are quaternary amines. of the following?

29. Acetylcholine is NOT widely used in clinical practice A. Treatment of hypotension in anaphylactic
because it is: shock
B. Treatment of benign prostatic
A. Very toxic hyperplasia
B. Requires very high doses C. Treatment of asthma
C. Rapidly hydrolyzed D. Treatment to reduce urinary frequency
D. Expensive ● treatment of essential hypertension
When ACh is administered systemically, it can potentially act at all of these ● treatment of congestive heart failure
sites; however, as a quaternary ammonium compound, its penetration to the ● decrease the incidence of digital vasospasm in patients
CNS is limited, and the amount of ACh that reaches peripheral areas with low with Raynaud disease
blood flow is limited due to hydrolysis by plasma butyrylcholinesterase. GG 13ed ● treatment of patients with mitral or aortic valvular
insufficiency
page 149
● α1-Selective antagonists have efficacy in
● BPH owing to relaxation of smooth muscle in the bladder
30. Idaho plate method provides this much calories: neck, prostate capsule, and prostatic urethra.

A. 1000 cal 34. Which of the following conditions responds to the


B. 1200 cal administration of Benztroine mesylate?
C. 2000 cal
D. 2500 cal A. Extrapyramidal symptoms
Idaho plate method: It helps the patient visualize how different food can be B. Insomnia
proportionally arranged on a plate for different meals. This method provides C. Parkinson’s Disease
1200-1500 calories. Diabetes cpg
D. A and B
E. A and C
31. Which of the following conditions would be the Dy-Ybanez Trans Chapter 9
therapeutic use of muscarinic agonists?

A. Mydriatic agent
B. Bronchial asthma treatment
C. Prostatic hypertrophy
D. Xerostomia

BK23 | TABLE OF CONTENTS


35. Which of the following is the most sensitive to 39. Which of the following best describes insulin
Acetylcholinesterase? degludec?

A. Bethanechol A. Has less severe hypoglycemia compared to


B. Carbachol glargine
C. Methacholine B. Binds well with albumin
D. Pilocarpine C. May be given every other day
Dy-Ybanez Trans Chapter 9 D. A combination of short and long acting
insulin
E. A and B
GG 13th ed page 872
Degludec, which is active at a physiologic pH, forms multihexamers after injection
subcutaneously. It has less severe hypoglycemia than glargine.
When degludec is injected subcutaneously, it forms multihexameric complexes
that slow absorption; degludec also binds well to albumin

40. Phenoxybenzamine blocks the following receptors


except which of the following receptors?

36. When is the crucial time to start treatment for A. ά receptor


cretinism? B. ß receptor
C. Serotonine receptor
A. Anytime D. Acetylcholine
B. First 2 weeks postnatal Phenoxybenzamine binds covalently to alpha receptors, causing irreversible
blockade of long duration (14-48 hrs or longer). It also blocks Histamine (H1),
C. First year of life acetylcholine, and serotonin receptors. p. 159 Katzung 14th ed
D. Up to 6 months postnatal
GG 13th ed page 792
41. The symptoms of mushroom poisoning include all of
the following EXCEPT:

A. Salivation, lacrimation, nausea, vomiting


B. Dryness of mouth, hyperpyrexia,
37. This is an example of type 3 drugs or incomplete
hallucination
transfer of drugs across the placenta
C. Headache, abdominal colic
D. Bradycardia, hypotension and shock
A. Ketamine
B. Thiopental
C. Succinylcholine
D. Warfarin
Katzung 14th ed page 1047
Highly ionized drugs such as succinylcholine and tubocurarine, also used for
cesarean sections, cross the placenta slowly and achieve very low
concentrations in the fetus.

38. Which of the following is NOT a characteristic of


Methimazole?

A. It is the drug of choice for Graves Disease


B. It has a long half life 42. The Na+-independent transport system involved in
C. It is concentrated in the thyroid the uptake of choline is inhibited by this drug
D. It inhibits peripheral conversion of T4 to
T3 A. Botulinum toxin
GG 13th ed page 796
propylthiouracil partially inhibits the peripheral deiodination of T4 to T3 . B. Syntaxin
Methimazole does not have this effect; this provides a rationale for the choice of C. Hemicholinium
propylthiouracil over other antithyroid drugs.
D. Curare
E. Neostigmine

BK23 | TABLE OF CONTENTS


What is the most likely impression of the patient?

A. Typhoid psychosis
B. Schizophrenia
C. Bipolar disorder
D. HIV psychosis
E. Delirium

43. To differentiate cholinergic (from insufficient therapy)


from myasthenic (excessive therapy) crisis, a
physician should do a test by giving which drug? 46. Which of the following is the most common form of
generalized epilepsy?
A. Edrophonium
B. Neostigmine A. Dravet Syndrome
C. Physostigmine B. Generalized Tonic Clonic Seizure
D. Ambenonium C. Absence seizure
Although the diagnosis of autoimmune myasthenia gravis usually can be made D. Juvenile Myoclonic Epilepsy
from the history, signs, and symptoms, its differentiation from certain
The most common generalized epilepsy is referred to as juvenile myoclonic
neurasthenic, infectious, endocrine, congenital, neoplastic, and degenerative
epilepsy (JME), accounting for about 10% of all epilepsy syndromes. GG p. 304
neuromuscular diseases can be challenging. However, in autoimmune
myasthenia gravis, the aforementioned deficiencies and enhancement of muscle
strength can be improved dramatically by antiChE medication. The edrophonium 47. Immunoglobulins are transported and crosses the
test for initial diagnosis relies on thes responses. The edrophonium test is
performed by rapid intravenous injection of 2 mg of edrophonium chloride,
placenta in this way
followed 45 sec later by an additional 8 mg if the first dose is without effect. A
positive response consists of brief improvement in strength, unaccompanied by A. Passive diffusion
lingual fasciculation (which generally occurs in nonmyasthenic patients).
B. Facilitated diffusion
C. Active transport
44. Which of the following agents is described as a long D. Pinocytosis
acting ß2 agonist?

A. Albuterol
B. Salbutamol
C. Formoterol
D. Isoproterenol
A. Albuterol - 3-4 h
B. Salbutamol - >12 h
C. Formoterol - 12 h
D. Isoproterenol

45. An 18 year old male developed an acute onset of 48. Which of the following drugs can be given as
moderate grade fever without chills or rigors along migraine headache prophylaxis?
with generalised weakness after a trip from a
province. On the second day of his illness, he began A. Pregabalin
behaving abnormally by remaining aloof from his B. Gabapentin
colleagues, refusing to participate in training C. Topiramate
activities, displaying delusions of persecution and D. Lamotrigine
passing urine and stools in his clothes without any E. C and D
evidence of social embarrassment. He was admitted
febrile and disoriented. There was
hepatosplenomegaly. Bone marrow culture showed
growth of S. typhi and Widal test showed S. typhi O
titer 1:480.

BK23 | TABLE OF CONTENTS


49. Which of the following patient groups is/are 53. The pharmacological agents used in the treatment of
candidates for a 75 gm OGTT as the preferred initial mania
screening test for DM?
A. Antipsychotics
A. FBS of greater than 100 but less than 126 B. Anticonvusants
mg/dl C. Lithium
B. High risk of cardiovascular disease D. A and C
C. Patients with metabolic syndrome E. All of the above
D. All of the above Goodman and Gilman 13th ed. p. 294-295
Doc Frankie’s slide
OGTT done on pxs with history of IFG, metabolic syndrome or diagnosed with
CV events

50. Which of the following GLP1 RA can be given once


weekly?

A. Albiglutide
B. Liraglutide
C. Lixisenatide
D. Exenatide
Page 874 Goodman and Gilman’s 13th edition

51. These properties of a drug enables the drug to cross


the placenta readily EXCEPT

A. Lipophilic
54. The presence of porphyria is a contraindication for
B. Unionized
use of this hypnotic-sedative
C. MW 250
D. Presence of P-glycoprotein transporters
P-gp and BCRP are the two major ABC efflux transporters in the placenta that limit A. Diazepam
fetal exposure to drugs B. Phenytoin
C. Propofol
52. Which of the following is a direct-acting D. Meprobamate
cholinomimetic that is lipid-soluble and is used to Goodman and Gilman 13th ed. p. 349 ; https://www.uofmhealth.org/health-
library/d00288a1
facilitate smoking cessation?

A. Acetylcholine
B. Bethanechol
C. Neostigmine
D. Physostigmine
E. Varenicline

BK23 | TABLE OF CONTENTS


57. Which of the following statements regarding the
autonomic nervous system characteristics is
INCORRECT?

A. The parasympathetic sacral outflow consists


of axons that arise from cells in the second,
third and fourth segments of the sacral cord
55. Which of the following agents is a mixed proceed to form the nervi erigentes
adrenoceptor antagonist? B. The motor nerves and interneurons of the
enteric nervous system are principally
A. Metoprolol organized into nerve plexuses: the
B. Propanolol myenteric (Auerbach’s) plexus and the
C. Esmolol submucosal (Meissner’s) Plexus
D. Carvedilol C. The efferent nerves of the involuntary
Katzung 14th ed. p. 159 system supply all innervated structures of
the body except skeletal muscle which is
served by somatic nerves
D. Cranial visceral sensory information
enters the CNS by four cranial nerves:
the oculomotor (III), trigeminal (V), facial
(VII) and Vagus (X)
E. The visceral afferents from cranial visceral
sensory afferents terminate topographically
in the solitary tract nucleus (STN)
Goodman and Gilman 13th ed. p. 115
Cranial visceral sensory information enters the CNS by four cranial
nerves: the trigeminal (V), facial (VII), glossopharyngeal (IX), and
vagus (X) nerves.

58. Ephedrine is an example of what type of adrenergic


agonist?

56. Which of the following is NOT an indication for


A. Selective direct acting agonist
phenobarbital use?
B. Indirect acting releasing agents
C. Mixed acting agonist
A. Absence seizure D. Non-selective indirect acting agonist
B. Generalized Tonic Clonic seizure of Goodman and Gilman 13th ed. p. 193
unknown onset
C. Focal seizures
D. None of the above
Goodman and Gilman 13th ed. p. 312

BK23 | TABLE OF CONTENTS


59. Women with no risk factors for DM should undergo 65. Which adrenoceptor antagonist may be used
lab screening for DM at: commonly as topical treatment of glaucoma?

A. 12-15 weeks A. Atropine


B. 20-24 weeks B. Timolol
C. 24-28 weeks C. Tropicamide
D. 28-30 weeks D. Scopolamine
Philippine Practice Guidelines on the Diagnosis and Management of Diabetes GG p.157
Mellitus Effects limited to the eye are obtained by topical administration of
muscarinic receptor antagonists to produce mydriasis and cycloplegia.
Cycloplegia is not attainable without mydriasis and requires higher
concentrations or more prolonged application of a given agent.
Mydriasis often is necessary for thorough examination of the retina and
optic disc and in the therapy of iridocyclitis and keratitis. Homatropine
hydrobromide, a semisynthetic derivative of atropine, cyclopentolate,
60. Nicotinic receptors are found at the surface of the hydrochloride, and tropicamide are agents used in ophthalmological
post ganglion plates in the: practice. These agents are preferred to topical atropine or scopolamine
because of their shorter duration of action.
A. Sympathetic nervous system chain
B. Parasympathetic nervous system chain 66. This is NOT part of the management of barbiturate
C. Sympathetic ganglion of the adrenal medulla poisoning
D. Somatic nervous system
A. Alkalinization of the urine
B. Osmotic diuresis
61. ß -adrenoceptor blockade may produce which of the C. Fluid replacement
following effects? D. Administration of CNS stimulant
GG p 349 The treatment of acute barbiturate intoxication is based on
general supportive measures, which are applicable in most respects to
A. Increase blood pressure
poisoning by an CNS depressant. The use of CNS stimulants is
B. Increase airway resistance contraindicated. If renal and cardiac functions are satisfactory and the
C. Increase intraocular pressure patient is hydrated, forced diuresis and alkalinization of urine will
D. Increase level of HDL cholesterol hasten the excretion of phenobarbital.

62. The primary agent for management of trigeminal 67. Which of the following mode of administration is best
neuralgia: for Oxybutinin?

A. Carbamazipine A. Oral extended release tablets


B. Gabapentin B. Sublingual
C. Vigabatrin C. Transdermal patch
D. Tiagabine D. IV bolus
GG p.157
Oxybutynin, the oldest of the antimuscarinics currently used to treat
63. The amino acid precursor of Norepinephrine,
overactive bladder disorders, is associated with a high incidence of
Epinephrine and Dopamine antimuscarinic side effects, particularly xerostomia. In an attempt to
increase patient acceptance, oxybutynin is marketed as a transdermal
A. Serotonin system that is associated with a lower incidence of side effects than
B. Tyrosine the oral immediate- and extended-release formulations; a topical gel
C. Tryptophan formulation of oxybutynin also appears to offer a more favorable side-
effect profile. Because of the extensive metabolism of oral oxybutynin
D. Alanine by enteric and hepatic CYP3A4, higher doses are used in oral than
E. Phenylalanine transdermal administration; the dose may need to be reduced in
patients taking drugs that inhibit CYP3A4.
64. Choline together with this ion is transported back
into the presynaptic membrane by which ion? 68. A drug used to reverse effects of benzodiazepine
and treat Z compound overdose
A. Sodium
B. Potassium A. Flumazenil
C. Magnesium B. Amobarbital
D. Calcium C. Methohexital
D. Flunitrazepam

BK23 | TABLE OF CONTENTS


71. Coarsening of the facial features is a side effect of
GG p. 346 which anti-seizure medication:
The primary indications for the use of flumazenil are the management
of suspected benzodiazepine overdose and the reversal of sedative
effects produced by benzodiazepines administered during general
A. Carbamazepine
anesthesia and diagnostic or therapeutic procedures. B. Gabapentin
C. Phenytoin
69. What is the function of thyroid peroxidase in thyroid D. Phenobarbital
hormone synthesis?

A. Deiodination
B. Iodide uptake
C. Oxidation and Coupling
D. Resorption of thyroglobulin
GG p.788
Oxidation and Iodination
Transport of iodine from the thyroid follicular cell to the colloid is
facilitated by the apical transporter pendrin. The oxidation of iodide to
its active form is accomplished by thyroid peroxidase. The reaction
results in the formation of MIT and DIT residues in thyroglobulin, a
process referred to as organification of iodine, just prior to its
extracellular storage in the lumen of the thyroid follicle.

72. The following are effects of Asenapine EXCEPT:


70. What muscarinic receptor is located in the brain?
A. Weight reduction
A. M1
B. No effect on lipid metabolism
B. M2
C. No effect on glucose metabolism
C. M5
D. Reduced neurologic risk
D. A and B
E. All of the above

73. What antidote would you administer for cholinergic


toxicity from Clitocybe dealbata mushroom
poisoning?

A. Atropine
B. Benzodiazepine
C. Epinephrine
D. None

BK23 | TABLE OF CONTENTS


74. A 65-year old female patient started a new 76. Indicate the location of M2 cholinoreceptor type:
antihypertensive drug maintenance. Blood pressure
seems to be under control but she complains of A. Heart
fatigue, drowsiness and fainting when she gets up in B. Glands
bed. Which of the following drugs is she currently C. Smooth muscle
taking? D. Endothelium

A. Metoprolol
B. Propanolol
C. Prazosin
D. Clonidine

77. Which of the following is an indication for tapering


anti-seizure maintenance medication?

A. Normal MRI scan


B. Normal electrolytes
C. Normal PE on the last check up
D. Seizure free for 2 years
E. All of the above
(See page 321 GG 13th Ed)
75. Which of the following is DOES NOT describe
levetiracetam?

A. Given as twice daily


B. Has good oral absorption
C. Adjunctive treatment for refractory
78. Actions and clinical uses of muscarinic cholinoceptor
generalized myoclonic seizure
agonists include which one of the following?
D. Can be used as monotherapy for
generalized tonic clonic seizure
A. Bronchodilation (treatment of asthma)
B. Miosis (treatment of glaucoma)
C. Decreased gastrointestinal motility
(treatment of diarrhea)
D. Decreased neuromuscular transmission and
relaxation of skeletal muscle (during surgical
anesthesia)
E. Increased sweating (treatment of fever)See
page 152 GG 13th Ed

BK23 | TABLE OF CONTENTS


79. Women who are planning pregnancy should take 82. What is the dose of atropine that would cause the
folic acid at least _______ months prior a planned patient to complain of headache, hot skin and
pregnancy to prevent neural tube defect: difficulty in urinating?

A. 1 month A. 0.5 mg
B. 2 months B. 1 mg
C. 3 months C. 2 mg
D. 4 months D. 5 mg
Doc Camomot’s ppt on Perinatal and Pedia Pharmacology E. >10mg
See page 153 GG 13th Ed)

80. What drug inhibits iodine transport?

A. Carbimazole
B. Lithium 83. The following describes the actions of
C. Perchlorate catecholamines except which of the following
D. 131 Radioactive iodine
See page 789 GG 13th Ed
statements?

A. Peripheral excitatory action of smooth


muscles of the blood vessels supplying the
kidneys
B. Peripheral inhibitory action of certain other
types of smooth muscles in the walls of the
gut
C. Cardiac excitatory action that increases the
heart rate and force of contraction
D. Metabolic functions such as a decrease
in the rate of glycogenolysis in the liver.
(Increase)
81. Which of the following is NOT considered an effect
of acetylcholine upon muscarinic receptor
activation?

A. Bronchoconstriction
B. Detrusor muscle contraction
C. Miosis
D. Vasoconstriction
Muscarinic receptor activation will cause vasodilation, bronchoconstriction,
detrusor muscle contraction and miosis. (See pp. 150-151 GG 13th Ed)

BK23 | TABLE OF CONTENTS


84. Patients with one lifetime episode of MDD who The first decision to make is whether and when to initiate treatment (French and
Pedley, 2008). For example, it may not be necessary to initiate therapy after an
achieve remission on antidepressants should isolated tonic-clonic seizure in a healthy young adult who lacks a family history of
continue to take them for another _____ months. epilepsy and who has a normal neurological exam, a normal EEG, and a normal
brain MRI scan.

A. 6-12 Duration of Therapy once initiated, ASDs are typically continued for at least 2
B. 24-36 years. Tapering and discontinuing therapy should be considered if the patient is
seizure free after 2 years; tapering should be done slowly over several months.
C. 18-36
D. 60 Factors associated with high risk for recurrent seizures following discontinuation
of therapy include EEG abnormalities, known structural lesions, abnormalities on
neurological exam, and history of frequent seizures or medically refractory
seizures prior to control.

(GG chap17, pg320-321)

87. The following drugs are antagonists of the N-methyl-


D.aspartate (NMDA) ionotrophic Glutamate
Receptor EXCEPT

A. Ketamine
B. Riluzole
C. Chick Pea toxin
85. Nicotinic receptor sites do not include which one of D. Dextromethorphan
the following sites? E. Memantine
not sure please correct if wrong (T_T) sorry guys ; but accdg to gg and internet:
Ketamine, Riluzole, Dextromethorphan, and Memantine are NMDA glutamate
A. Bronchial smooth muscle antagonist

B. Adrenal medullary cells


C. Parasympathetic ganglia 88. What are conditions that would impair the absorption
D. Skeletal muscle end plates of levothyroxine?
E. Sympathetic ganglia
A. Amniodarone administration
B. Concomitant administration with Sertraline
C. Intake with food
D. Pregnancy

86. Fabian is a 25 yo medical student sought ff up at the


OPD. He was seen last month due to generalized
tonic clonic seizure. Patient denies recurrence of
seizure. MRI and EEG were unremarkable. PE at
the clinic was also unremarkable. What is the next
best thing to do?

A. Continue anti-seizure prophylaxis ?


B. Observe for recurrence
C. Rpt initial MRI and EEG for confirmation
D. Give anti-seizure medication as needed only

BK23 | TABLE OF CONTENTS


89. Which of the following agents is BEST used for the
treatment of xerostomia after head & neck radiation?

A. Cevimeline
B. Carbachol
C. Methacholine
D. Pilocarpine
GG 13th Ed. Page 152

92. The neurotransmitter of all preganglionic autonomic


fibers, most postganglionic parasympathetic fibers, and
a few postganglionic sympathetic fibers
90. A 4-year-old child has been admitted to the
A. Acetylcholine
emergency department having swallowed the contents
B. Dopamine
of 2 bottles of a nasal decongestant. The active
C. Epinephrine
ingredient of the medication is a potent, selective α- D. Glutamate
adrenoceptor agonist drug. Which of the following is a E. Norepinephrine
sign of α-receptor activation that may occur in this Goodman and Gillman Pharma 13th ed., page 116
patient?

A. Bronchodilation
B. Cardiac acceleration (tachycardia)
C. Pupillary dilation (mydriasis)
D. Renin release from the kidneys
E. Vasodilation of the blood vessels of the skin
GG 13th Ed. Page 121

93. Which of the following is an accepted therapeutic


indication for the use of anti-muscarinic drugs?

91. A 3-year old child is admitted to the emergency A. Atrial fibrillation


department after taking a drug from her parents’ B. Botulinum poisoning
medicine cabinet. The signs suggest that the drug is an C. Chronic obstructive pulmonary disease
indirect-acting cholinomimetic with little or no CNS effect (COPD)
and a duration of action of 2-4 hours would be caused D. Glaucoma
by? E. Postoperative urinary retention
GG 13th ed p. 156-158: Therapeutic Uses of Muscarinic Receptor
Antagonists
A. Pyridostigmine
B. Physostigmine
C. Pilocarpine
D. Acetylcholine

BK23 | TABLE OF CONTENTS


94. Which of the following is NOT a side effect of
zonisamide?

A. Dizziness
B. Spontaneous abortion
C. Bicarbonate overload
D. Kidney stones
GG 13th ed p. 320

96. Botulinum toxin blocks the release of acetylcholine


from cholinergic nerve terminals. Which of the following
is a possible effect of the toxin?

A. Improvement of myasthenia gravis


symptoms
B. Skeletal muscle paralysis
C. Increased bronchial secretions
D. Reduced conduction velocity

97. A 34 year old cargo boy was seen at the emergency


room for changes in behaviour. He reported that he is
seeing his guardian angel anywhere he goes. History
revealed patient is a metamphitamine user. What is the
mechanism of the positive symptoms of this patient?

A. Increased postsynaptic DA activity because


of increased presynaptic DA release
B. Increased postsynaptic DA activity
because of presynaptic DA reuptake
inhibition
95. Parasympathetic nerve stimulation and a slow C. Decreased the glutamate-mediated tonic
inhibition of DA release
infusion of bethanechol will each
D. A and B only
E. A, B and C are correct
A. Cause ganglion cell depolarization
B. Cause skeletal muscle end plate
depolarization
C. Cause vasodilation
D. Increase bladder tone
E. Increase heart rate

NCBI

BK23 | TABLE OF CONTENTS


98. What measures can be done in an event of
antimuscarinic toxicity?

A. Perform gastric lavage


B. Give benzodiazepine for sedation
C. Physostigmine administration
D. Control hyperthermia
E. All of the above

99. A 45 yo diabetic was rushed to the ER due to


drowsiness. CBG is noted to be – HIGH. Upon
examination the patient noted to have tachypnea and
acetone breath. Which of the following is NOT part of the
initial management of this patient? 100. Which of the following anti-seizure drugs can be
used for controlling absence seizure?
A. IV hydration with PNSS
B. Potassium replacement A. Ethosuximide
C. Long acting insulin B. Valproate
D. IV line D5W C. Carbamazepine
E. None of the above D. Phenobarbital
E. A and B

BK23 | TABLE OF CONTENTS

You might also like